finals2009.wikispaces.com · Web viewPleural effusion which is secondary to hypoalbunimaemia Q4....

296
1.2 Question 1 As an obstetric SHO, you are presented with a primigravida who is 41 weeks + 2 days pregnant. She has had an uneventful pregnancy but has been admitted to hospital for induction of labour. She is unsure how to prepare a feed of formula milk. She asks you for advice. List four pieces of practical advice you would give her on how to prepare a feed. Any four of the following; Hands must be washed and absolutely clean before handling the bottles and teats • Suitable bottles and teats must be thoroughly washed and sterilised before use •Water should be boiled before it is used to make a feed •Information on how much water and the number of scoops of formula powder needed for a feed for a baby of various ages can be found on the back of the manufacture’s formula milk container. •Water should be put into the bottle FIRST and accurately measured in the bottle by using the fluid ounces/mls guide at the side of bottle. •The correct number of scoops of formula powder should be put into the bottle AFTER the water has been added •A clean knife should be used to scrape off any excess powder in each scoop to ensure the correct amount of formula powder is in each scoop •The bottle filled with water and formula powder should be GENTLY shaken to ensure adequate mixing •The milk should be allowed to cool and tested on the back of the hand before giving it to the baby •The milk, once prepared can be stored in the fridge for up to 24 hours. •The best way to warm a refrigerated bottle is by placing it in a jug of warm water. Take great care if you heat the feed in a microwave oven because of the risk of hot spot burning. 1

Transcript of finals2009.wikispaces.com · Web viewPleural effusion which is secondary to hypoalbunimaemia Q4....

Page 1: finals2009.wikispaces.com · Web viewPleural effusion which is secondary to hypoalbunimaemia Q4. What initial investigations would be implemented to confirm this diagnosis? Urine

1.2

Question 1

As an obstetric SHO, you are presented with a primigravida who is 41 weeks + 2 days

pregnant. She has had an uneventful pregnancy but has been admitted to hospital for

induction of labour. She is unsure how to prepare a feed of formula milk. She asks

you for advice. List four pieces of practical advice you would give her on how to

prepare a feed.

Any four of the following;

•Hands must be washed and absolutely clean before handling the bottles and teats

• Suitable bottles and teats must be thoroughly washed and sterilised before use

•Water should be boiled before it is used to make a feed

•Information on how much water and the number of scoops of formula powder

needed for a feed for a baby of various ages can be found on the back of the

manufacture’s formula milk container.

•Water should be put into the bottle FIRST and accurately measured in the bottle by

using the fluid ounces/mls guide at the side of bottle.

•The correct number of scoops of formula powder should be put into the bottle

AFTER the water has been added

•A clean knife should be used to scrape off any excess powder in each scoop to

ensure the correct amount of formula powder is in each scoop

•The bottle filled with water and formula powder should be GENTLY shaken to

ensure adequate mixing

•The milk should be allowed to cool and tested on the back of the hand before giving

it to the baby

•The milk, once prepared can be stored in the fridge for up to 24 hours.

•The best way to warm a refrigerated bottle is by placing it in a jug of warm water.

Take great care if you heat the feed in a microwave oven because of the risk of hot

spot burning.

Question 2

A mother presents to the GP clinic and is concerned about when and how to wean her

20 week old baby.

a. What is the current recommended age for weaning a baby?

6 months (recommended by department of health)

1

Page 2: finals2009.wikispaces.com · Web viewPleural effusion which is secondary to hypoalbunimaemia Q4. What initial investigations would be implemented to confirm this diagnosis? Urine

b. List three signs are suggestive that a child is ready for weaning?

The baby is hungrier and cries more for food

The baby wakes up for a feed more time during the night than previously

The baby shows more interest in food

The baby starts to make chewing motions with their mouth

c. What types of foods can be initially introduced to the baby? List two.

Baby Rice cereal mixed in with breast or formula milk (recommended by the

department of health as a starter)

Pureed fruit (e.g. Banana)

Pureed cooked vegetables ( e.g potatoes, carrots broccoli, peas, parsnips)

d. List two types of food you would advice the mother to avoid in the first stages of

weaning

Gluten containing products (if weaned before 6 months)

Don't add sugar or salt to any foods

Don't feed hot spices in the first 12 months

Avoid (until 12 months) raw or soft eggs, patè, soft cheeses such as brie, honey( in

rare cases it can lead to infant botulism

Avoid peanut butter until 12 months or 3 years if allergies run in the family,

Avoid whole nuts until 6 years (due to the dangers of choking).

e. The mother asks you ‘What about milk, do I have to discontinue that now?’ What

will you reply? Give a reason for your answer.

You should reply ‘No’

The reason for this is that babies still rely on milk for most of their energy (calorie)

intake.

1.2 Feeding

By the end of the course students should be able to:

• make and give a feed

• give advice to parents about infant feeding and weaning

2

Page 3: finals2009.wikispaces.com · Web viewPleural effusion which is secondary to hypoalbunimaemia Q4. What initial investigations would be implemented to confirm this diagnosis? Urine

1.5

Question 1

A two-day-old term infant on a postnatal ward has had bilious vomits after every feed

since birth. The baby is hungry and feeds well but is yet to pass meconium. Clinical

examination reveals a dysmorphic child with a flat occiput, large tongue, low set ears

and single palmar crease. A plain abdominal film was taken and showed a double

bubble appearance in the stomach.

a. What is the ‘double bubble appearance’ indicative of?

Duodenal atresia

b. What genetic condition does this child have that accounts for his dysmorphic

features?

Down’s Syndrome

c. List three further investigations you would perform and give a reason for each

investigation stated

Karyotype: To confirm the diagnosis of chromosomal abnormality

Upper gastrointestinal (GI) contrast study: There has been no passage of meconium

for 48 hours

Echocardiogram: Congenital heart disease should be actively looked for in babies with

Down’s syndrome

d. Give three possible short term or long term complications of this condition that may

necessitate intervention.

Any three of the following:

Congenital heart disease

Duodenal atresia

Severe learning disability

Small stature

Recurrent respiratory infections

Hearing impairment from acute otitis media

Visual impairment from cataracts, squints

Increased risk of leukaemia

Atlanto-axial subluxation (rare)

3

Page 4: finals2009.wikispaces.com · Web viewPleural effusion which is secondary to hypoalbunimaemia Q4. What initial investigations would be implemented to confirm this diagnosis? Urine

Hypothyroidsm

Alzheimer’s disease

e. What information, support or advice can be given to the parents at the time of

diagnosis and thereafter?

Any one of the following:

Written explanation of condition and its cause

Information about the short term and long term implications of diagnosis

Information about national charities and self help groups e.g Down Syndrome

Association

Professional counselling to help deal with feelings of guilt, disappointment or

anger

Antenatal diagnosis for future pregnancies

Question 2

Ben aged two months is admitted with tachypnoea following an upper respiratory tract

infection. On examination there is a thrill, a loud (grade4/6) pansystolic murmur at the

left sternal edge and the liver is palpable 4 cm below the costal margin.

a. What is the most likely cause of his clinical condition?

Heart failure, probably precipitated by an upper respiratory tract infection

b. What is the most likely underlying diagnosis?

Ventricular septal defect (VSD)

c. A chest x-ray was performed. Name 3 signs on the x-ray that may be indicative of

your chosen diagnosis

Cardiomegaly,

Enlarged pulmonary arteries,

Increased pulmonary vascular markings

Pulmonary oedema

d. What single diagnostic investigation would you perform now?

Echocardiography

e. Outline 2 important aspects of management that may be considered in this

condition and give a reason for performing each intervention

1) Drug therapy – furosemide, thiazide, spironolactone, ACE inhibitors

Reason – To treat symptomatic heart failure

2) Surgery

4

Page 5: finals2009.wikispaces.com · Web viewPleural effusion which is secondary to hypoalbunimaemia Q4. What initial investigations would be implemented to confirm this diagnosis? Urine

Reason: Severe symptoms with failure to thrive OR

Reason: Pulmonary hypertension with possible progression to pulmonary vascular

disease and subsequent Eisenmenger’s syndrome.

3.) Monitoring and observation (symptoms, signs, CXR, echocardiography)

Reason: This is acceptable in asymptomatic patients. By the age of 2 years at

least 50% of small and moderate sized VSDs undergo sufficient partial or

complete spontaneous closure to make intervention unnecessary.

1.5 Congenital Abnormalities

By the end of Phase II , students should be able to:

• recognise and explain to parents the principles of management of the commoner

congenital abnormalities

5

Page 6: finals2009.wikispaces.com · Web viewPleural effusion which is secondary to hypoalbunimaemia Q4. What initial investigations would be implemented to confirm this diagnosis? Urine

1.6

1. Nick and Judith have been blessed with a bouncing baby boy, Gavin.

What three measurements are plotted on his centile growth chart?

Head circumference

Weight

Height

Being distinctly average, he follows the 50th centiles. What would be noted on his chart

to constitute mild failure to thrive?

Mild failure to thrive is a fall across two centile lines.

(Severe failure to thrive is a fall over three centile lines)

Name two Organic and two Non-Organic causes of failure to thrive.

Organic:

Inability to feed

Poor retention of food

Illness-induced anorexia

Impaired nutrient absorption

Increased energy requirements

Metabolic causes

Chromosomal disorders/syndromes

Congenital infection

Non-Organic:

Feeding problems

Maternal stress

Lack of stimulation and undernutrition

Munchausen’s syndrome by proxy

Three years later, Gavin’s parents tell you he has reached all his developmental

milestones at the correct age.

Name three of the five developmental areas that are measured.

6

Page 7: finals2009.wikispaces.com · Web viewPleural effusion which is secondary to hypoalbunimaemia Q4. What initial investigations would be implemented to confirm this diagnosis? Urine

Gross motor

Fine motor

Vision

Hearing and Speech

Social behaviour

At what median age are the following milestones achieved?

Crawls –

8-9 months

Has a mature pincer grip –

10 months

Smiles responsively -

6 weeks

What screening tool is used for testing hearing in infants?

The Distraction test

At what age would this tool be used?

6-9 months

2. Child X has cerebral palsy. Name two ways in which cerebral palsy might present.

Abnormal tone and posturing

Feeding difficulties

Delayed motor milestones

Abnormal gait when walking is achieved

Language and social developmental delay

The management of Child X’s cerebral palsy will involve a multidisciplinary team

approach.

Name four members who might be involved.

7

Page 8: finals2009.wikispaces.com · Web viewPleural effusion which is secondary to hypoalbunimaemia Q4. What initial investigations would be implemented to confirm this diagnosis? Urine

Paediatrician

General Practitioner

Eric’s Family

Physiotherapist

Occupational therapist

Speech and language therapist

Social worker

Health visitor

Surgeon (e.g. fundoplicaiton for recurrent oesophageal reflux)

1.6 Developmental child health

By the end of Phase II , students should be able to:

• recognise failure to thrive, its common causes and initiate management

• make a developmental assessment of the infant and toddler

• recognise delay in speech and in walking

• examine a child for hearing loss, including distraction testing

• examine a child for reduced visual acuity and squint

• outline to parents the facilities available for children with learning difficulties

• outline to parents the facilities available for children with mobility difficulties

• recognise short stature and refer appropriately

• recognise delayed puberty and discuss the causes and investigation with patients and

parents

8

Page 9: finals2009.wikispaces.com · Web viewPleural effusion which is secondary to hypoalbunimaemia Q4. What initial investigations would be implemented to confirm this diagnosis? Urine

1.7

JB is a child who has been diagnosed with cystic fibrosis. This requires daily

medication, frequent visits to hospital and several hospital admissions for respiratory

illness. When he is 3 he is admitted to hospital for 2 weeks. During this time his

parents are unable to visit.

1) What are the stages of the acute separation response in young children?

Protest

Despair

Detachment

2) What factors in family life can have a negative impact on the psychological

wellbeing of a child? Give 6 examples.

Parental mental illness

Inconsistent, unpredictable discipline

Divorce

Inappropriate expectations/responsibilities for age.

Abuse

Intrusive overprotection.

Bullying.

3) What are the common disorders of psychological development that could present in

any child at age <5 yrs, 5-10 years old and as an adolescent? Give 3 examples for each.

- Age <5 yrs

Meal refusal

Sleeeping problems (night terrors, nightmares, waking at night)

Breath-holding attacks

(Aggressive behaviour, Autism, Tantrums).

- 5-10 years old

Nocturnal enuresis

Hyperactivity

School refusal

(Anxiety, antisocial behaviour, Tics, Faecal soiling)

- Adolescence

Anorexia nervosa

9

Page 10: finals2009.wikispaces.com · Web viewPleural effusion which is secondary to hypoalbunimaemia Q4. What initial investigations would be implemented to confirm this diagnosis? Urine

Chronic fatigue syndrome

Deliberate self-harm

(Drug misuse, depression)

JB starts at nursery, and the nursery nurse expresses concerns that he may have

features of an autistic spectrum disorder.

4) What are the three main features of autism?

Poor social interaction

Lack of imagination

Poor communication skills.

5) What other behaviour patterns can be seen in autism? Give 2 examples.

Obsessive behaviours

Repetitive behaviours

When JB is 9 he starts refusing to go to school.

6) What are the two main causes of school refusal?

Separation anxiety persisting beyond the toddler years.

School phobia – anxiety provoked by an aspect of school.

7) Name 3 professional groups who could be involved when an older child refuses to

go to school.

Teachers

Educational psychologist

Educational welfare officer

8) Name 6 behavioural features that might suggest attention deficit disorder.

Fidgeting

Continually interrupts

Instructions never obeyed

Blurts out answers

Behind with schoolwork

Short attention span.

10

Page 11: finals2009.wikispaces.com · Web viewPleural effusion which is secondary to hypoalbunimaemia Q4. What initial investigations would be implemented to confirm this diagnosis? Urine

9) What are the social and emotional issues that JB might face when he becomes a

teenager?

Low self esteem

Problems gaining independence from parents

Rebelling against medication regime.

Problems conforming with peer group because of limitations of illness.

Depression

Anxiety

Alcohol and drug misuse

1.7          Child Psychiatry

By the end of Phase II , students should be able to:

• recognise the common presentations of psychiatric disorder in children and distinguish the

common causes

• recognise the interaction between child and family upon the psychological disorders of

childhood

• discuss attachment theory

• recognise the common disorders of childhood psychological development

• recognise the impact on children of physical/mental illnesses in a parent

• recognise the psychological aspects of chronic physical illness in children

• discuss school refusal with a child and its parents

• recognise the social and emotional issues in adolescents with chronic illness 

11

Page 12: finals2009.wikispaces.com · Web viewPleural effusion which is secondary to hypoalbunimaemia Q4. What initial investigations would be implemented to confirm this diagnosis? Urine

1.10

1) You are a house officer in A+E and you see an 8 month old child who has had

vomiting and diarrhoea for 2 days.

What signs would you look for to assess hydration? (6 marks)

Any 6 of:

Sunken fontanelle

Prolonged capillary refill time

Dry mucous membranes

Reduced skin turgor

Sunken eyes/tearless

Oliguria

Tachypnoea

Tachycardia

Reduced level of consciousness

Hypotension

How would you clinically assess severity of dehydration in terms of body weight loss?

(3 marks)

Mild dehydration < 5%

Moderate dehydration 5-10%

Severe dehydration >10%

Name some routes through which rehydration therapy be administered? (2 marks)

Oral

NG tube

Intravenous

Intra-osseous

Name a micro-organism that commonly causes gastroenteritis in children in developed

countries? (1 mark)

Any of:

12

Page 13: finals2009.wikispaces.com · Web viewPleural effusion which is secondary to hypoalbunimaemia Q4. What initial investigations would be implemented to confirm this diagnosis? Urine

Viruses –Rotavirus, Adenovirus, Coronavirus

Bacteria – Campylobactor, Shigella, E.Coli, Staphylococcus, Salmonella

2) A 2 year old boy is given the diagnosis of coeliac disease in outpatient clinic.

What symptoms and signs would suggest this diagnosis? (6 marks)

Any of:

Failure to thrive

Abnormal stools/diarrhoea – foul smelling, pale, frequent and bulky

Vomiting

Weight loss

Constipation

Short stature

Abdominal distension

Buttock wasting

Irritability

Delayed motor milestones

Glossitis, angular stomatitis

Dermatitis Herpetiformis

What serological investigations would you consider? (2 marks)

Any of:

Anti-gliadin antibodies

Anti-endomysial antibodies

Anti-tissue transglutaminase antibodies

Anti-reticulin antibodies

What advice would you give to parents about the long term management and

implications of coeliac disease? (2 marks)

13

Page 14: finals2009.wikispaces.com · Web viewPleural effusion which is secondary to hypoalbunimaemia Q4. What initial investigations would be implemented to confirm this diagnosis? Urine

Any of:

Removal of all products containing wheat, rye and barley from the diet for life

Referral to a dietician

Possibility of a gluten challenge later in childhood to reassess the susceptibility of the

jejunum to damage by gluten

Life long follow-up

Increased risk of complications in later life

Extended questions:

What other diseases are patients with coeliac disease more prone to in later life? (1

mark)

Any of:

Small bowel malignancy

Enteropathy associated T cell lymphoma

Increased risk of other autoimmune disorders – thyroid, adrenal, pancreas, SLE, RA

Osteoporosis

Ulcerative jejunoileitis

The gold standard investigation for diagnosis of coeliac disease is jejunal biopsy.

What histological features would you expect to see in the jejunum of a patient with

coeliac disease? (2 marks)

Any of:

Lymphocyte infiltration

Subtotal villous atrophy

Crypt atrophy

Increased intra-epithelial cells

3) A 4 year old child presents with constipation.

What are the symptoms which would suggest a diagnosis of constipation? (2 marks)

Any of:

Child passes <1 motion every 3 days

14

Page 15: finals2009.wikispaces.com · Web viewPleural effusion which is secondary to hypoalbunimaemia Q4. What initial investigations would be implemented to confirm this diagnosis? Urine

Passage of stools that are hard, small or pellet like

Stools that are difficult or painful to pass

What lifestyle changes would you suggest to parents to help manage the constipation?

(2 marks)

Any of:

Increase fluid intake

Diet high in fibre, fruit and vegetables

Regular toileting

What is encopresis, and how can affect a child and their family? (3 marks)

Encopresis is the passage of stools in unacceptable places in a child over the age of

4 years old.

Any sort of bio-psycho-social answer eg. Increased washing of clothes,

teasing/bullying at school, increased risk of abuse,

Name two different types of laxatives available and give an example of each? (4 marks)

Any of:

Bulk forming laxatives – Ispaghula Husk (Fybogel, Fibrelief)

Stimulant laxatives – Senna (or Bisacodyl, Dantron, Sodium Picosulfate, Glycerol)

Faecal softeners – Liquid paraffin (or arachis oil)

Osmotic laxatives – Lactulose (or macrogols, magnesium salts, phosphates, sodium

citrate)

Bowel cleansing preparations – Picolax (or Klean-prep, Citramag)

Extended question:

Name some organic causes of constipation (2 marks)

Any of

Hypothyroidism

Hypercalcaemia

Hirschprungs disease

15

Page 16: finals2009.wikispaces.com · Web viewPleural effusion which is secondary to hypoalbunimaemia Q4. What initial investigations would be implemented to confirm this diagnosis? Urine

Urinary concentrating defect

Anorectal stenotic lesions

Anal Fissures

Spinal/Neuromuscular disease

1.10       Gastrointestinal Disease

By the end of Phase II , students should be able to:

• recognise the common causes of feeding difficulty and initiate management

• discuss feeding difficulties with parents

• recognise dehydration and the associated biochemical changes

• recognise the common causes of vomiting in infancy, childhood and adolescence and initiate

management

• recognise the common causes of acute and chronic diarrhoea in infancy, childhood and

adolescence and initiate management

• recognise the common causes of constipation in infancy, childhood and adolescence and

initiate management

• give advice to parents on gastroenteritis

• recognise and assess obesity

• make an initial assessment of a child with acute abdominal pain and refer appropriately

• recognise the common causes of chronic abdominal pain

• recognise the common causes of an abdominal mass in infancy, childhood and adolescence

16

Page 17: finals2009.wikispaces.com · Web viewPleural effusion which is secondary to hypoalbunimaemia Q4. What initial investigations would be implemented to confirm this diagnosis? Urine

1.11

A 4 year old Asian boy presents with a history of swelling around his eyes, most

noticeable in the morning for the preceding 5 days. His parents have noted that his

urine appears frothy and is reduced in volume.

Q1. What is the most likely diagnosis?

Nephrotic syndrome

Q2. Apart from peri-oribital and ankle swelling what other areas would you look for

swelling particularly if this was a severe case?

Swelling of scrotum

On examination there is dullness to percussion and reduced breath sounds at the right

base.

Q3. What are these clinical findings suggestive of and why has it happened?

Pleural effusion which is secondary to hypoalbunimaemia

Q4. What initial investigations would be implemented to confirm this diagnosis?

Urine dipstick

U+ E

albumin levels

microscopy and culture

Q5. What is the treatment for this patient and how long should the patient be treated

for?

Oral steroids – prednisolone 60mg/m2/day for a period of 10days

17

Page 18: finals2009.wikispaces.com · Web viewPleural effusion which is secondary to hypoalbunimaemia Q4. What initial investigations would be implemented to confirm this diagnosis? Urine

Q6. What are the side-effects of these drugs in children? Name 2.

Growth stunting and immunosuppresion

Q7. What are the serious complications which may occur with this condition? (2)

Hypovolaemia (intravascular compartment becomes depleted)

Thrombosis ( hypercoaguable state due to urinary loss of antithrombin and increased

blood viscocity from raised haematocrit predisposes to thrombosis).

Q8. What prophylaxis should be given in the acute phase of this condition and why?

Pencillin prophylaxis is given in acute phase to prevent pneumococcus infection due

to urinary loss of immunoglobulins increasing susceptibility to infection

1.11 Renal/urinary

By the end of Phase II , students should be able to:

• recognise oedema in infancy, childhood and adolescence, discuss its causes and

management with parents

• recognise a history of haematuria, and initiate investigations

• recognise polyuria and initiate investigations

• identify the commoner causes of bedwetting and give advice to parents

• fit a urine bag, obtain a specimen for testing

• explain the need for suprapubic aspiration to parents

• recognise and interpret the urine abnormalities in urinary infections

1.12

Qu. 1. Lucy is a 2 year old girl who presents to A & E. The parents are very worried,

they say that Lucy has been ill with a temperature for a few days and this morning just

after breakfast she had a ‘fit’ which lasted about quarter of an hour.

18

Page 19: finals2009.wikispaces.com · Web viewPleural effusion which is secondary to hypoalbunimaemia Q4. What initial investigations would be implemented to confirm this diagnosis? Urine

a) List three causes of Lucy’s fit in your differential diagnosis:

Febrile convulsion

Meningitis with a febrile convulsion.

Infantile spasm (3-12 months of age).

b) To help you distinguish which of your differentials is most likely list 6 things you

would specifically want to rule out on examination?

Photosensitivity,

Neck stiffness.

Fever,

Purpuric rash,

Signs of shock

Positive Brudzinski’s/Kernig signs

c) Your SHO performs a lumbar puncture on Lucy. What changes would you expect in

the CSF for both bacterial and viral meningitis?

1) Bacterial: Turbid CSF, increased (!!) polymorphs, increased proteins,

increased (!!) glucose.

2) Viral: Clear CSF, Lymphocytes increased, Normal or increased (!) protein,

normal or increased (!) glucose.

d) What are the main causative agents of meningitis?

1 month-6 years:

Neisseria meningitides

Streptococcus pneumoniae.

Haemophilus influenzae

e) What is the immediate management for meningitis? Step 1 is done for you.

1) ABC

Start antibiotics - Third generation Cephalosporin (Cefotaxime) plus either

Rifampicin or Vancomycin whilst waiting for CSF culture results..

CSF lumbar puncture (ideally CT first )

19

Page 20: finals2009.wikispaces.com · Web viewPleural effusion which is secondary to hypoalbunimaemia Q4. What initial investigations would be implemented to confirm this diagnosis? Urine

Check antibiotic regime against CSF results and change if necessary.

Establish if there are any contacts.

Reassess

f) List 4 complications of meningitis:

Hearing loss.

Local vasculitis - cranial nerve pathology.

Local cerebral infarction.- epilepsy.

Hydrocephalus.

Other: Cerebral abcess and Subdural effusion.

Extended: What antibiotic would you give to anyone Lucy may have had contact with

who is at risk?

Rifampicin.

Qu.2. Dave is a 15 year old boy who presents to his GP with his mum. Whilst shopping

his mum witnessed Dave having a ‘fit’. She explains that he suddenly dropped to the

floor and at first was still but then started to shake. When he came round he was

sleepy and spent the rest of the day in bed. His mum is very worried.

a) What extra information would be useful to know?

Was he incontinent?

Has it happened before?

Did he have any preceding aura or warning sign.

Is he on any medication?

Does he use recreational drugs?

Does he drink alcohol?

b) Name 5 different types of generalised epilepsies.

Absence seizures.

Myoclonic seizures.

Tonic seizures.

Tonic-clonic seizures.

Atonic seizures.

20

Page 21: finals2009.wikispaces.com · Web viewPleural effusion which is secondary to hypoalbunimaemia Q4. What initial investigations would be implemented to confirm this diagnosis? Urine

c) Dave admits that this is his fourth fit. His friends had seen two of the previous three

fits and had described the same fit in the same way as his mum. What investigations

are required?

Blood tests checking electrolytes.

Serum glucose.

EEG.

CT or MRI.

d) Name three anticonvulsant drugs that you could prescribe for generalised tonic-

clonic seizures?

Sodium valporate.

Carbamazepine.

Lamotrigine.

e) Dave explains that once has finished school he wants to join the army. Mention 5

things you would want to cover with Dave and his mum?

Provide written information (Leaflets) on epilepsy.

Give details of support groups.

3) Educate both Dave and his mum about the recovery position and how rectal

diazepam is useful and how it should be administered.

4) Explain that there are driving restrictions imposed by the DVLA regarding how

long he must be free of fits before he is allowed to drive. He should look these up

when he wants to learn to drive.

He will need to contact the army to see if he will be eligible to join.

1.12 Central Nervous System

By the end of Phase II , students should be able to:

• take a history of febrile fits

• take a history of epilepsy in infancy, childhood and adolescence

• outline to parents the nature and treatment of febrile fits and epilepsy

• examine a floppy baby and discuss the nature of the problem with parents

• recognise the possibility of meningitis and initiate management

• recognise and interpret the CSF abnormalities of meningitis

• recognise the common causes of abnormalities in gait in infancy, childhood and

21

Page 22: finals2009.wikispaces.com · Web viewPleural effusion which is secondary to hypoalbunimaemia Q4. What initial investigations would be implemented to confirm this diagnosis? Urine

adolescence

22

Page 23: finals2009.wikispaces.com · Web viewPleural effusion which is secondary to hypoalbunimaemia Q4. What initial investigations would be implemented to confirm this diagnosis? Urine

1.13

A 7 year old boy presents to A&E with after concerns from school about the extent of

continuous bruising on the buttocks and legs.

Q1. What would concern you about this pattern of bruising illustrated in Figure 1?

Unexplained injuries to protected parts of the body such as the buttock may indicate

the presence of non-accidental injury. The presence of bruises that have various

ages may signify multiple episodes of injury caused by ongoing physical abuse

Q2.What other areas of the body if bruised would raise your suspicions of a diagnosis

of bruising caused by non-accidental injury?

Thighs, torso, frenulum, ears and neck

Q3. How can you make a rough estimate the age of the bruising in this picture?

By the colour of the bruise. The picture shows yellow/brown colour indicating that

these bruises are older than a day. A bruise of within a day will be erythematous

Q4. What is the difficulty with determining the bruise age by colour?

Determining the bruise's age by its color must take into account the individual

patient's skin tone, the bruise's location (whether overlying soft tissue or bone), the

23

Page 24: finals2009.wikispaces.com · Web viewPleural effusion which is secondary to hypoalbunimaemia Q4. What initial investigations would be implemented to confirm this diagnosis? Urine

patient's relative nutritional state or other indeterminate factors.

Q5. What investigation may be indicated in the presence of suspicious bruising?

Platelet count and coagulation screen to exclude thrombocytopaenia and other

bleeding diatheses which often presents with multiple and excessive bruising.

Q6. How would you proceed with this case and whom should you inform?

If child abuse is suspected in the UK the Social Services Child Family department

must be informed. A decision must be made whether the child requires immediate

protection either hospital admission or placement in foster home. In most cases

further management will involve evaluation of the family by social workers and Child

Protection Conference.

Q7. Who should be present at the Child Protection Conference?

Paediatrician, GP, health visitor, social worker, police, nursing staff, representatives

from school and parents.

1.13 Haematology

By the end of Phase II , students should be able to:

• assess anaemia in infancy and childhood

• assess a child with bruising

24

Page 25: finals2009.wikispaces.com · Web viewPleural effusion which is secondary to hypoalbunimaemia Q4. What initial investigations would be implemented to confirm this diagnosis? Urine

1.14

SW is a 5-year-old male who presented to A&E with severe vomiting, abdominal pain,

and drowsiness. This had lasted for approximately 4 hours and was preceded by 1

week history of flu like illness. On examination he had signs of weight loss and

dehydration.

A) What would be your main differential in SW? Give two other examples of differential

diagnoses.

Diabetic Ketoacidosis

Gastroenteritis

Appendicitis

B) After initial ABC management, give three other management steps.

Rehydration with fluids (normal saline over 48-72hrs)

Insulin administration (sliding scale starting at 0.1units/kg/hr)

Potassium infusion.

(metabolic acidosis should correct itself following fluid administration).

C) SW was newly diagnosed with insulin dependent diabetes mellitus following this

incident. Give an example of how you would explain to his parents what this condition

is and how it needs to be managed.

“Your son has been diagnosed with diabetes which means that he is not producing

insulin within his pancreas, which we all need to regulate our blood sugar. This is due

to an autoimmune process, where there is destruction of the islet cells within the

pancreas which produce the insulin. Unfortunately he will need to be put on insulin,

and will need daily injections throughout his life. If his condition is managed

effectively then he will almost certainly lead a normal life.”

25

Page 26: finals2009.wikispaces.com · Web viewPleural effusion which is secondary to hypoalbunimaemia Q4. What initial investigations would be implemented to confirm this diagnosis? Urine

D) SW is responding well to treatment, give two regular investigations that his parents

could perform at home to monitor his diabetes.

Blood glucose.

Urine dipsticks (to check for protein and ketones).

E) List 3 key members of the diabetes team which will be essential in providing

continuing care throughout SW’s childhood.

-Consultant Paediatrician (with an interest in diabetes)

-Diabetic specialist nurse

-Paediatric dietician

F) SW needs to be placed upon an insulin regime. Give 2 examples of possible

regimes:

Twice-daily regimen – Twice daily short acting and intermediate acting insulin

Basal-bolus regimen – Three times daily short acting boluses and one or two

medium acting basal boluses of insulin.

1.14 Diabetes

By the end of Phase II , students should be able to:

• diagnose and assess diabetes

• outline to 'parents' in a role play situation the management of childhood diabetes

• perform ward-based blood and urine tests and interpret the results

26

Page 27: finals2009.wikispaces.com · Web viewPleural effusion which is secondary to hypoalbunimaemia Q4. What initial investigations would be implemented to confirm this diagnosis? Urine

1.15

A 1 year old boy, DS, is brought to the GP by his mother with an itchy rash, which is

diagnosed as atopic eczema.

1) What are the other possible diagnoses. Give 4.

Contact dermatitis

Pityriasis rosea

Fungal infection

Psoriasis

Urticaria

Chickenpox

2) What features would you expect to find on examination of this child’s rash. Give 4

points

present mainly on face and scalp. (Occurs mainly on flexure surfaces in older

children)

excoriation

weeping rash

crusted rash

erythematous rash

Dry skin.

3) What type of illnesses is this child at increased risk of developing?

Atopic disease – hayfever, asthma.

4) What is the underlying pathological reaction.

Type I Hypersensitivity

Increased IgE production in response to common environmental allergens.

27

Page 28: finals2009.wikispaces.com · Web viewPleural effusion which is secondary to hypoalbunimaemia Q4. What initial investigations would be implemented to confirm this diagnosis? Urine

5) What has caused this serious exacerbation of this child’s eczema?

Infection with Herpes Simplex Virus

6) What else could cause an exacerbation of this child’s eczema? Give 5 points

Staph aureus infection

Contact with an irritant/allergen

Psychological stress

Idiopathic

Environment (heat, humidity)

7) What simple advice could you give to DS’s mother for managing this child’s

eczema? Give 4 points.

28

Page 29: finals2009.wikispaces.com · Web viewPleural effusion which is secondary to hypoalbunimaemia Q4. What initial investigations would be implemented to confirm this diagnosis? Urine

Avoid soap and detergents

Avoid nylon and wool clothing next to the skin

Cut nails short (to reduce damage from scratching)

Avoid any precipitating allergens.

8) What medical treatments are available for the treatment of eczema? Give 5.

Emollients

Topical corticosteroids

Occlusive bandages

Antibiotics

H1 histamine antagonists

1.15        Skin disease

By the end of Phase II , students should be able to:

• recognise the common skin conditions in infancy, childhood and adolescence

• outline to parents the management strategies available 

29

Page 30: finals2009.wikispaces.com · Web viewPleural effusion which is secondary to hypoalbunimaemia Q4. What initial investigations would be implemented to confirm this diagnosis? Urine

1.16

PW is a 2 year old girl who presents to A&E with a painful swollen right arm.

Give 3 salient points that you would ask within the history.

-What was she doing at the time of injury?

-Any previous history of injury to the arm?

-Any other injuries?

What investigations would you order?

X-ray of site of injury.

Blood results (coagulation disorders, calcium and phosphate levels, copper deficiency)

What type of fracture is shown in the radiograph above?

Greenstick fracture of the right radius

30

Page 31: finals2009.wikispaces.com · Web viewPleural effusion which is secondary to hypoalbunimaemia Q4. What initial investigations would be implemented to confirm this diagnosis? Urine

A CxR taken at the same time is shown below:

What abnormality is shown by the arrows?

healed rib fractures

What other points in the history may aid a diagnosis of NAI?

Previous injuries in unlikely areas e.g. skull fractures, facial bruising, cigarette burns.

Inconsistent stories

Social circumstances e.g. underlying drug / alcohol abuse within the family.

If you do suspect non-accidental injury, who else would you involve in the

investigation and management? List three.

Social services

Health worker

GP

Community Paediatrician

Police in severe cases

Give one other important diagnosis which may produce the picture seen in

radiograph B, and what other examination findings may substantiate this

diagnosis.

31

Page 32: finals2009.wikispaces.com · Web viewPleural effusion which is secondary to hypoalbunimaemia Q4. What initial investigations would be implemented to confirm this diagnosis? Urine

Osteogenesis imperfecta

Blue sclera

Skeletal deformities

Short stature

Joint laxity

(Excellent question)

Give the pathophysiology of this condition.

Osteogenesis imperfecta is a condition resulting from abnormality in the type I

collagen. Type I collagen fibers are found in bones, organ capsules, fascia, cornea,

sclera, tendons, meninges, and dermis. Structurally, this protein is composed of a

left-handed helix formed by intertwining of pro-a1 and pro-a2 chains. Mutations in the

loci coding for these chains result in osteogenesis.

1.16 Fractures in Children

By the end of their teaching the students should be able to:

• recognise those fracture patterns that occur in children and their possible aetiological

factors

• order appropriate investigations

• be aware of non-accidental injury and know the outline strategy for the handling of non

accidental injuries

• act as part of the team in the immediate and definitive care

32

Page 33: finals2009.wikispaces.com · Web viewPleural effusion which is secondary to hypoalbunimaemia Q4. What initial investigations would be implemented to confirm this diagnosis? Urine

1.17

Question 1

Harry, a 7 year-old boy has had pain in the right leg since he woke up this morning. He

finds it difficult to walk on the leg due to pain, which has been getting worse

throughout the day. His teachers at school said that he was unable to join in his P.E

lesson because he was in too much discomfort and his mother was asked to pick him

up early from school since the pain was getting significantly worse. The pain is in the

groin, front of the thigh and radiates down to the knee; is exacerbated by movements

at the hip and weight bearing. Harry does not have any pain in any other joints, and he

otherwise feels fit and well. There is no history of trauma; Harry was playing happily

the previous day. He had a moderately severe viral upper respiratory tract infection 2

weeks ago; but has now fully recovered. All other past medical history was

unremarkable. Harry is afebrile.

◦ List 3 differential diagnoses on the basis of this history:

transient monoarticular synovitis (TMS)septic arthritis (SA)juvenile rheumatoid arthritis(others to consider; osteomyelitis, leukaemia and other malignancies)

◦ What findings on examination would you look for to aid your diagnosis?

Assess gait and ability to weight bear (TMS – able to walk despite

discomfort; SA – severe pain, tenderness to palpation, spasm, and refusal

to walk)

Examination of the hip: TMS – no signs of trauma; mild, or no effusion -

unlikely to be obviously swollen;

mild restriction of hip motion, especially abduction and internal rotation. (cf.

SA or bacterial infection – hip is held in a flexed, abducted and externally

rotated position and severely painful to move)

Vital signs: TMS – afebrile; SA – usually febrile (>38.50C)

◦ Are there any particular features of the history that give you a clue to the

diagnosis?

**Past history of non-specific viral upper respiratory infection – Approximately 70% of

involved children have a history of prodromal symptoms indicative of a viral URTI 7-

14 days before onset of TMS of the hip

33

Page 34: finals2009.wikispaces.com · Web viewPleural effusion which is secondary to hypoalbunimaemia Q4. What initial investigations would be implemented to confirm this diagnosis? Urine

Other features consistent with this diagnosis:

Sex: male-to-female ratio is 2:1; Age: mean age of onset is 6 years, most patients

are 3-8 years of age; distribution and character of symptoms: monoarticular, acute

onset, progressive pain exacerbated by active movement at the joint but still able to

weight bear; otherwise in good health, no systemic symptoms associated; no history

of trauma.

Harry’s mother persuaded him to rest as much as possible and gave him some pain

relief, which eased his discomfort. However, after two days, Harry felt able to mobilise

again and was keen to get back to school and play football in the playground with his

friends. After one day back at school, running around, Harry’s hip became extremely

painful once again and he was unable to weight bear on it by the evening. His mother

took him to their GP the following day for further review.

◦ Given the progression of symptoms, what now seems to be the most likely

diagnosis?

Transient monoarticular synovitis (hip)

◦ What investigations would you instigate?

1st line: Blood tests (FBC, U&E’s, LFT’s)

AP and lateral pelvic x-rays

Others to consider: Aspiration of fluid if moderate effusion present – may improve

symptoms and improve local blood flow

Bone scans may help to differentiate a septic process

◦ What advice would you give Harry and his mother meanwhile to help ease the

discomfort?

Bedrest and non-weight bearing until the pain resolves, followed by limited activities

(for 1-2 weeks)

Important to communicate that rest is essential even following resolution of pain – if

the child returns to normal activities too early, exacerbation of the symptoms can

occur

NSAID’s can help for symptomatic relief

Advised to return to the GP for further evaluation if there is lack of

improvement/worsening of symptoms despite adequate rest to eliminate other, more

serious disorders.

34

Page 35: finals2009.wikispaces.com · Web viewPleural effusion which is secondary to hypoalbunimaemia Q4. What initial investigations would be implemented to confirm this diagnosis? Urine

Question 2

John is 5 years old. His mother is concerned because she has noticed him walking

with an increasingly obvious limp for the last 3 weeks. He does not complain of

discomfort when running or walking but has reported intermittent “soreness” of the

leg from time to time. When asked where it hurts, he points to the front of his right

thigh. John is otherwise in good health, has no recent history of illness and all other

joints are non painful and normally active/non-restrictive. There is no history of trauma

and review of all systems was unremarkable – he is afebrile.

On examination John appeared short for his age, and walked with a limp – minimal

weight bearing on the right leg. There was no discrepancy of leg length, swelling, signs

of trauma or localised tenderness on palpation. There was decreased range of

movement at the hip – poor abduction and internal rotation and muscle spasms were

seen in the quadriceps of the right leg. Examination of the left leg and hip was normal.

Investigations showed John to have delayed bone age.

◦ What is the most likely diagnosis?

Perthes disease – idiopathic avascular necrosis (osteonecrosis) of the

capital femoral epiphysis (CFE) and the associated complications in a

growing child. Caused by interruption of the CFE blood supply.

◦ What issues should be discussed with John and his mother about problems that

could occur in later life given the suspected diagnosis?

The short-term prognosis concerns femoral head deformity at the completion of the

healing stage. Development of late degenerative arthritis occurs; the incidence is

essentially 100% in children who are 10 years of age or older at onset. This rate is in

contrast to a negligible risk in children 5 years or younger and a 38% risk when onset

occurs between 6 and 9 years of age. Therefore, discussion should focus on the

potential that there is a moderate risk but the age of onset is of good prognostic

value.

Long-term prognosis involves the potential for osteoarthritis of the hip in adulthood

◦ What features of the history suggest that there is a good prognosis?

Age – 5 years old

(The age related risk is associated with premature CFE closure. Early intervention to

prevent deformity of the femoral head prior to closure reduces the likelihood of

degenerative changes occurring, reducing the incidence of later complications/joint

disease and the associated morbidity in early adulthood.)

35

Page 36: finals2009.wikispaces.com · Web viewPleural effusion which is secondary to hypoalbunimaemia Q4. What initial investigations would be implemented to confirm this diagnosis? Urine

◦ Which specific investigation would help determine the severity of the condition?

AP and lateral pelvic x-ray – technique of choice for determining the extent of CFE

involvement, useful for following disease progression and identifying CFE collapse

and extrusion and assess response to treatment.

Occasionally additional procedures such as arthrography, bone scans, and MRI may

be useful. Bone scans and MRI are helpful in recognising early perthes disease but

are of limited value in assessing the extent of CFE involvement or following the

disease progression.

◦ What interventions could be considered (possibly not immediately) to improve

John’s prognosis and avoid complications in later life?

Surgical and non-surgical “containment” - the femoral head is contained within the

acetabulum, which acts as a mold for the reossifying CFE.

Perthes disease is a local, self-healing disorder – treatment focuses of prevention of

femoral head deformity and secondary osteoarthritis. Containment is indicated for

children 6 years of age or older in whom more than half the CFE is involved. This is

accomplished by non-surgical containment using abduction casts and orthoses or by

surgical containment with proximal femoral varus osteotomy – long term results are

85-90% satisfactory.

Objective 1.17

The Limping Child

By the end of their teaching the student should be able to:

• examine the child with the painful hip or knee

• consider the conditions of irritable hips, synovitis, systemic rheumatoid arthritis, slipped

upper femoral epipyhsis, Perthes disease, septic arthritis, osteomylitis and tumour

• initiate appropriate investigations

• outline treatment to the child and family

36

Page 37: finals2009.wikispaces.com · Web viewPleural effusion which is secondary to hypoalbunimaemia Q4. What initial investigations would be implemented to confirm this diagnosis? Urine

2.2

Miss AB is a 26 year old woman who has been to her GP with concerns that her period

is 3 weeks late.

1. Give two other symptoms which could be obtained form the history, and may

suggest that she is pregnant. (2 points)

breast tenderness

vomiting

2. There are several clinical signs which can be seen on examination of a

pregnant woman (later gestation), name 4. (4 points)

Striae gravidarum

linea nigra

areolae pigmentation

chloasma

montgomery’s tubercles (on the breast)

3. A urine pregnancy test is suggested. What substance does this investigation

detect, what is it produced by and what role does it play in pregnancy. (3

points)

ßhCG (human chorionic gonadotrophin)

produced by the trophoblast

it maintains the corpus luteum which secretes progesterone to maintain the

pregnancy

37

Page 38: finals2009.wikispaces.com · Web viewPleural effusion which is secondary to hypoalbunimaemia Q4. What initial investigations would be implemented to confirm this diagnosis? Urine

4. Routine ultrasound scan is performed at 11-14 weeks; suggest one use that

this has. (1 point)

confirm gestational age

identify multiple pregnancies

nuchal thickness in Down’s Syndrome

2.2 Diagnosis of pregnancy

By the end of Phase II , students should be able to:

• diagnose pregnancy on clinical grounds

• use tests of pregnancy appropriately

• recognise the role of ultrasonography in the diagnosis of pregnancy

38

Page 39: finals2009.wikispaces.com · Web viewPleural effusion which is secondary to hypoalbunimaemia Q4. What initial investigations would be implemented to confirm this diagnosis? Urine

2.3

Mrs J is a 30 year old lady who has come to ask about starting a family. She has no

medical problems but smokes 20 cigarettes a day.

1. State 2 factors which you would advise her about and for each give a reason why.

(4 points)

Smoking cessation: Any of; associated with IUGR, premature labour, miscarriages

Folic acid supplements: reduce the risk of neural tube defects

2. What important advice would you give if Mrs J had a pre-existing condition such as

diabetes or epilepsy? (1 point)

Ensure optimum management of pre-existing maternal disease prior to pregnancy if

possible and address potential teratogenic medications

(question not quite fit in: debate role of screening for genetic disease and be able to take a

family hx with reference to genetic disease)

2.3 Pre-pregnancy counselling and care

By the end of Phase II , students should be able to:

• recognise the importance of pre-pregnancy counselling in particular groups

• debate the role of screening for genetic disease

• take a simple family history with reference to genetic disease

39

Page 40: finals2009.wikispaces.com · Web viewPleural effusion which is secondary to hypoalbunimaemia Q4. What initial investigations would be implemented to confirm this diagnosis? Urine

2.4

A 35 year old lady presents to her GP saying that she has carried out a home

pregnancy testing kit and it has revealed a positive result. She wants to confirm

whether or not she is pregnant and how far she has progressed through her

pregnancy.

Name five likely symptoms of pregnancy noticed in the first trimester of pregnancy,

that this lady may mention.

Morning sickness.

Fatigue.

Frequency of micturition.

Heartburn.

Constipation.

Breast changes.

Emotional lability.

This lady tells you that the date of her last menstrual period (LMP) was on the 7 th

September 2005. Calculate using Naegle’s Rule the estimated date of delivery (EDD).

Estimated date of delivery (EDD) = Last menstrual period (LMP) + 9

mnths 7 dys

= 7th September + 9 mnths 7 dys

= 14th September 2005 + 9 mnths

= 14th June 2006

Give one reason directly associated with the menstrual cycle, that could affect the

accuracy of this calculated EDD.

Some women have longer than average length of menstrual cycles.

Variation in the length of the proliferative phase of the menstrual cycle.

List three other confounding factors could affect EDD accuracy.

Assumption of 40 weeks gestation (280 days).

Assumption of conception occurring two weeks after first day of LMP

(14 days into cycle ovulation occurs, assumption of conception

40

Page 41: finals2009.wikispaces.com · Web viewPleural effusion which is secondary to hypoalbunimaemia Q4. What initial investigations would be implemented to confirm this diagnosis? Urine

occurring on this day, but could have occurred any time between

then and next date of expected menstrual period).

Break through bleeding could confuse the date of LMP.

Bleeding from recent IUCD use could confuse date of LMP.

Break through bleeding and irregularity of cycles after cessation of

oral contraceptive pill could confuse date of LMP.

Describe the basis of a standard pregnancy test.

B human chorionic gonadotrophin (B hCG) secreted from placenta

and trophoblastic tissue is excreted in the maternal urine. This is

detected by monoclonal antibodies to it, tagged to latex or red cells.

A visible colour change in the detection probe is noticed by this

reaction which confirms the pregnancy.

What visible changes may be noticed on speculum examination in a pregnant woman?

Bluish tinge to the vagina and cervix.

Give three ultrasound parameters that may be noted with respect to the foetus, to

verify pregnancy dates.

Presence and size of gestation sac.

Foetal crown-rump length.

Foetal bipareital diameter.

Foetal head circumference.

Foetal abdominal circumference.

Foetal femur length.

List five potential risk factors for abnormality in pregnancy.

Maternal age.

Previous miscarriage.

Consanguineous marriage between parents.

Alcohol abuse.

Drug abuse.

Teratogen ingestion.

Maternal illness (including infections such as HIV).

41

Page 42: finals2009.wikispaces.com · Web viewPleural effusion which is secondary to hypoalbunimaemia Q4. What initial investigations would be implemented to confirm this diagnosis? Urine

Poor folic acid intake.

There is concern from an ultrasound scan that this woman’s child may have Down’s

Syndrome. A “triple test” is carried out on maternal serum. Name the three factors

tested, and indicate whether these would be high or low if the foetus does have Down’s

Syndrome.

Human chorionic gonadotrophin levels raised.

Decreased alpha fetoprotein levels.

Decreased unconjugated oestradiol levels.

Name and give a brief description of three investigations used to detect foetal

abnormalities (other than the “triple test” and ultrasound scanning).

1 Amniocentesis – fine needle used to aspirate amniotic fluid from

the mother’s uterus, under ultrasound real time guidance. Cells

shed from foetal gut and skin can then be used for further tests such

as karyotyping.

2 Chorionic villus sampling – sample of actively dividing trophoblast

cells allows rapid karyotyping. Done in 1st trimester by

transabdominal/transcervical route.

3 Placental biopsy – sample of actively dividing trophoblast cells

allows rapid karyotyping. Done in 2nd/3rd trimester by transabdominal

route.

4 Foetal Tissue Sampling – done under real time ultrasound

guidance, involving skin or liver or other organ tissue extraction

depending on indication..

5 Antenatal foetal blood sampling – done under USS guidance

transabdominally (placental cord, foetal intrahepatic vessels, foetal

heart).

Name a foetal abnormality that could be responsible for raised alpha fetoprotein levels

in maternal serum.

42

Page 43: finals2009.wikispaces.com · Web viewPleural effusion which is secondary to hypoalbunimaemia Q4. What initial investigations would be implemented to confirm this diagnosis? Urine

Neural tube defect (anencephaly, spina bifida).

Give three other reasons for raised alpha fetoprotein in maternal serum.

Exomphalos.

Gastroschisis.

Polycystic kidneys.

Foetal death (tissue autolysis).

Foetal teratoma (AFP secreted from tumour).

Duodenal/oesophageal atresia.

A younger pregnant woman mentions a long term problem with acne vulgaris, for

which she had roaccutane treatment. What sort of drug is this and why is this of

relevance with respect to her unborn child?

It is an isotretinoin medication derived from vitamin A and is highly

teratogenic.

2.4 Ante natal care

By the end of Phase II , students should be able to:

• assess pregnancy by a detailed obstetric and general medical history

• calculate the expected date of delivery and allow for the possible confounding effects of

irregular menstruation and the effects of the oral contraception

• identify normal pregnancy

• identify the possibility of abnormality in pregnancy

43

Page 44: finals2009.wikispaces.com · Web viewPleural effusion which is secondary to hypoalbunimaemia Q4. What initial investigations would be implemented to confirm this diagnosis? Urine

2.5

A PRHO clerks a 16 year old girl who presented to the A&E department with acute

abdominal pain.

The girl gave a history of generalised lower abdominal pain for 3 days, worsening over

the past 24 hours and being more localised over right iliac fossa.

On examination she has rebound abdominal tenderness in right iliac fossa.

1) Give four likely differential diagnoses.

Gastrointestinal – appendicitis, mesenteric adenitis, Meckel’s diverticulum,

inflammatory bowel disease, gastroenteritis, peritonitis.

Reproductive – Ectopic pregnancy, salpigitis/pelvic inflammatory disease,

endometriosis, mittelschmerz.

Renal – UTI, ureteric colic(stones).

Pain from adjacent areas

2) The PRHO then performs a pregnancy test:

a) What hormone is measured to test for pregnancy?

Beta - Human chorionic gonadotrophin.

b) Specifically which cells secrete it?

Trophoblast cells.

c) When is it first measurable in the urine?

2 weeks after implantation.

3) The hormone level is >1000 IU/L and the uterus is empty on ultrasound scan.

Give two possible explanations for this and underline the most serious one.

Early intrauterine pregnancy (<5weeks).

Ectopic pregnancy.

44

Page 45: finals2009.wikispaces.com · Web viewPleural effusion which is secondary to hypoalbunimaemia Q4. What initial investigations would be implemented to confirm this diagnosis? Urine

4) With regards to this serious problem, what factors may predispose to this?

Give two.

Previous tubal surgery.

Fertility treatment (IVF, GIFT).

Previous Ectopic pregnancy.

Congenital abnormalities (diverticula).

IUCD.

Progesterone only pill.

Emergency contraceptive.

5) Give one medical and one surgical treatment options.

Medical –methotrexate either systemically or direct injection into fallopian tube

under USS guidance.

Surgical – laparoscopy and salpingectomy or salpingostomy.

6) How would you follow this patient up?

Beta-HCG and TV USS every 1-2 days to ensure termination.

Give all rhesus negative women anti-D.

7) In a 16 year old girl specifically, what advise would you give regarding

contraception?

2.5 Major problems of pregnancy

By the end of Phase II , students should be able to:

• recognise, and initiate management of

• spontaneous miscarriage

• ectopic pregnancy

• trophoblastic disease

• abruptio placentae

• placenta praeva

• unclassified antepartal haemorrhage

• recognise pre-eclampsia

45

Page 46: finals2009.wikispaces.com · Web viewPleural effusion which is secondary to hypoalbunimaemia Q4. What initial investigations would be implemented to confirm this diagnosis? Urine

• initiate investigation of gestational hypertension

• explain the consequences of pre-eclampsia

• outline to a patient the management of pre-eclampsia and gestational hypertension

46

Page 47: finals2009.wikispaces.com · Web viewPleural effusion which is secondary to hypoalbunimaemia Q4. What initial investigations would be implemented to confirm this diagnosis? Urine

2.6

You are a GP at your local practice. An extremely anxious 40-year-old woman comes to

see you after reading a news paper article about the risks of Downs Syndrome in older

women. You know this patient well. You diagnosed her pregnancy last month after she

missed her last two periods. The pregnancy test was positive and her last menstrual

period (LMP) was 10 weeks ago. You and the patient discuss prenatal screening with

particular reference to Downs Syndrome.

What investigation can reveal the gestational age of the fetus more reliably than a

history of her LMP?

Ultrasound

What four main types of prenatal screening are available to detect Downs Syndrome?

Maternal serum screening (blood) or Triple Test.

Ultrasound

Amniocentesis

Chronic Villus Sampling

Which three markers are assessed by the Triple Test.

AFP (alpha fetoprotein)

Oestriol

HCG.

Explain why the cut off for high vs low risk of Downs Syndrome via the Triple Test is

set at 1 in 250?

If the risk is 1 in 250 or less, the risk of having a baby with Down syndrome is greater

than the risk of having a miscarriage from the amniocentesis.

If the risk is more than 1 in 250, the risk of having a miscarriage from amniocentesis

is greater than the risk of having a baby with Down syndrome.

47

Page 48: finals2009.wikispaces.com · Web viewPleural effusion which is secondary to hypoalbunimaemia Q4. What initial investigations would be implemented to confirm this diagnosis? Urine

What abnormalities may be found on Ultrasound to indicate Downs Syndrome? List

3.

Any three of….

Nuchal translucency

Choroid plexus cyst

Echogenic bowel

Echogenic intracardiac focus

Dilatation of kidneys

The fetus was determined to be high risk of Downs Syndrome by Triple Test. What two

further investigations can give a more reliable assessment and at what gestational age

can they be performed? Note: They both allow fetal chromosomal analysis.

Amniocentesis at 14-18 weeks, and Chronic Villus Sampling at 10-12 weeks.

List 5 side effects/ risks for these investigations?

Cramps

Bleeding

Infection

Leak amniotic fluid

Increased risk of misscariage.

This patient asks you if medication can be harmful during pregnancy. Below are six

common drugs. List one major side effect to the fetus in each.

Warfarin: Neonatal haemorrhage

Tetracyclines: Abnormal bone growth or Teeth discoloration

ACE inhibitors: Renal damage

Benzodiazepines: Neonatal withdrawal

NSAID: Delayed closure of ductus arterious

Phenytoin: Cleft palate or cardiac abnormalities

Amiodarone: Neonatal goitre.

48

Page 49: finals2009.wikispaces.com · Web viewPleural effusion which is secondary to hypoalbunimaemia Q4. What initial investigations would be implemented to confirm this diagnosis? Urine

2.6 Fetal growth, development and disease

By the end of Phase II , students should be able to:

• recognise and interpret discrepancy in uterine size for gestational age

• outline to patients the role of blood tests, ultrasound amniocentesis and chorionic villus

sampling in the establishment of fetal health

• describe the role and methods of fetal assessment in pregnancy

• recognise the potential adverse effects of drugs in pregnancy

• debate the benefits and costs of prenatal screening

49

Page 50: finals2009.wikispaces.com · Web viewPleural effusion which is secondary to hypoalbunimaemia Q4. What initial investigations would be implemented to confirm this diagnosis? Urine

2.7

A 32-year-old multiparous woman presents at 10 weeks’ gestation to the antenatal

clinic. She has a blood pressure of 140/90 mmHg.

1.) Name 3 causes for this BP (3 marks)

Essential hypertension

Chronic hypertension

Secondary hypertension

2.) What examinations should be performed? (2 marks)

Repeat BP measurements

Fundoscopy – assesses severity of hypertension

3.) What investigations should be performed? (4 marks)

FBCs

U+Es

LFTs

24-hour urinary protein concentration

4.) How should this condition be managed? (2 marks)

Treatment should be aimed at controlling the BP. Drugs that have been used

include methyldopa, labetolol and nifedipine. These drugs can be used as

single agents or in combination. (Important to remember that labetolol and

nifedipine have been known to cause fetal and neonatal bradycardias.

Methyldopa may cause abnormal LFTs and depression).

50

Page 51: finals2009.wikispaces.com · Web viewPleural effusion which is secondary to hypoalbunimaemia Q4. What initial investigations would be implemented to confirm this diagnosis? Urine

A 26-year-old woman who is now 37 weeks pregnant has been attending the antenatal

clinic since she was 14 weeks pregnant. A routine FBC performed by the midwife at 36

weeks shows the following abnormal results:

Hb = 8.2 g/dl

MCV = 68.4 fl

WBC = 9.0 x 109

1.) What is the diagnosis? (1 mark)

Microcytic anaemia

2.) What are the reasons for this diagnosis? (2 marks)

Low Hb, low MCV

3.) What other symptoms may she present with? (3 marks)

Breathlessness

easy fatiguability

asymptomatic

palpitations

tinnitus

symptoms from underlying cause. e.g. GI bleed- malaena, rectal bleed

4.) What further investigation will you perform on her? (2 marks)

Serum ferritin

5.) Assume she has iron deficient anaemia what 2 treatment options may you

recommend for her at this stage?

(2 marks)

51

Page 52: finals2009.wikispaces.com · Web viewPleural effusion which is secondary to hypoalbunimaemia Q4. What initial investigations would be implemented to confirm this diagnosis? Urine

Parenteral iron, blood transfusion

A glucose tolerance test was performed on a pregnant woman who previously had a

stillbirth at 39 weeks. The results are shown below:

Fasting = 5.5 mmol/l

60 minutes = 12.4 mmol/l

120 minutes = 10.5 mmol/l

1.) What is the diagnosis? (1 mark)

Impaired glucose tolerance

2.) What treatment will you recommend for her? (2 marks)

Dietary control in the first instance, if unsuccessful, insulin should be started

3.) What are the complications of the condition during pregnancy? (3 marks)

Fetal macrosomia, unexplained intrauterine death, polyhydramnios

4.) When is the baby most likely to be delivered? (1 mark)

At 38 weeks gestation

5.) If the mother is placed on insulin, what must be done after delivery?

(1 mark)

Stop the insulin and return her to her pre-pregnancy status; without any

treatment)

6.) What advice will you give this woman about the long-term implications of this

condition? (2 marks)

She has a higher risk of developing adult-onset diabetes mellitus and

therefore will require frequent screening

52

Page 53: finals2009.wikispaces.com · Web viewPleural effusion which is secondary to hypoalbunimaemia Q4. What initial investigations would be implemented to confirm this diagnosis? Urine

Mrs PB attended for routine antenatal care at 28 weeks gestation when she was found

to have a + protein in her urine. She had no symptoms. An MSU grew E. Coli for which

she was treated successfully. On 2 subsequent visits she was found to have

proteinuria and an MSU showed significant showed significant bacterial growth but

she remained asymptomatic.

1.) What is this condition called? (2 marks)

Asymptomatic significant bacteruria

2.) What is the sequelae if this is untreated? (1 mark)

Symptomatic UTI

3.) What 2 causes of this condition need to be excluded? (2 marks)

Urinary tract abnormalities

kidney stones

pyelonephritis

4.) What treatment would this patient benefit from? (2 mark)

Antibiotics with either trimethoprim, ampicillin or nitrofurantoins

5.) What further investigations need to be performed on her? (2 marks)

Ultrasound scan of kidneys, intravenous urogram 3-4 months after

delivery

2.7          Medical problems in pregnancy

By the end of Phase II , students should be able to:

• initiate management of the common self-limiting problems in pregnancy

53

Page 54: finals2009.wikispaces.com · Web viewPleural effusion which is secondary to hypoalbunimaemia Q4. What initial investigations would be implemented to confirm this diagnosis? Urine

• recognise the potential adverse effects of intercurrent disorders such as anaemia, asthma,

cardiac disease, diabetes, hypertension, renal and thyroid disease

• initiate appropriate investigations for these disorders

• appreciate the risk of thromboembolic disease in pregnancy

• participate in the long-term management of these conditions 

2.8

Mrs G has conceived after her third IVF attempt. She has just had an USS at 6 weeks

and has been told she has triplets.

a. What three possible first trimester complications may she have?

Miscarriage

Hyperemesis gravidarum

Vanishing triplet syndrome.

b. What prophylactic drugs must she have during pregnancy? Name 2.

Iron tablets

folic acid.

c. What would be the most common complication in the second half of

pregnancy?

Preterm labour

d. What three other maternal complications may arise?

Anaemia

Hypertension

Preeclapsia

pressure symptoms.

54

Page 55: finals2009.wikispaces.com · Web viewPleural effusion which is secondary to hypoalbunimaemia Q4. What initial investigations would be implemented to confirm this diagnosis? Urine

e. What precautions might be taken to reduce risk and complications of

prematurity?

Admission into hospital for bed rest.

Antenatal steroids to reduce the severity of respiratory distress syndrome in the

babies.

f. How may the babies be delivered?

The most common way is by C-section but vaginal delivery is possible.

2.8 Multiple pregnancy

By the end of Phase II , students should be able to:

• recognise the likelihood and make the diagnosis of multiple pregnancy

• recognise the complications of multiple pregnancy during pregnancy, parturition and the

puerperium

• outline to patients the possibility of preterm labour and the early symptoms

55

Page 56: finals2009.wikispaces.com · Web viewPleural effusion which is secondary to hypoalbunimaemia Q4. What initial investigations would be implemented to confirm this diagnosis? Urine

2.9

A woman was admitted to the delivery suite at 38 weeks gestation with absent fetal

movements for 24 hours and fetal heart was not picked up on CTG.

a) How would you confirm the diagnosis of intrauterine fetal death? (1 mark)

Ultrasound scan

b) How would you counsel the mother? (3 marks)

Explain the diagnosis to the mother in an empathic way

explain need to deliver the fetus

explain need to investigate the cause of intrauterine death

c) Name 4 investigations you would perform on the mother. (4 marks)

Coagulation profile

thyroid function test

HbA1c

Kleihauer test

lupus anticoagulant and anticardiolipin antibodies

infection screen

d) Name 2 investigations you would perform on the fetus. (2 marks)

Swabs from nose, throat ear and mouth

Skin biopsy for karyotype

Autopsy

2.9          Intrauterine and neonatal deaths

By the end of Phase II , students should be able to:

56

Page 57: finals2009.wikispaces.com · Web viewPleural effusion which is secondary to hypoalbunimaemia Q4. What initial investigations would be implemented to confirm this diagnosis? Urine

• recognise the antecedents and methods of prevention of intrauterine and neonatal death

• recognise on clinical grounds the possibility of intrauterine death and initiate appropriate

investigations

• communicate with patients in a role play situation about the diagnosis of intrauterine death

• participate as a team member in the counselling of parents sustaining an intrauterine or

neonatal death

57

Page 58: finals2009.wikispaces.com · Web viewPleural effusion which is secondary to hypoalbunimaemia Q4. What initial investigations would be implemented to confirm this diagnosis? Urine

2.10

A gravida 0, aged 37 women, comes to see you for advice as she is planning to start a

family. She has heard that there are risks associated with starting a family later in life.

a) List 2 maternal risks of elderly primigravida. (2 marks)

Any of:

Increased risk of ectopic pregnancy

Increased spontaneous loss

Molar pregnancy

Thromboembolism

Mortality.

b) Give 2 obstetric risks associated with increased age at first delivery. (2 marks)

2 of:

abruptio placenta (HTN, fribroids)

placenta previa

increased rate of c-section

uterine infection (postpartum endometriosis).

c) What are the foetal risks? Give 2. (2 marks)

Increased incidence of twinning

Iatrogenic preterm delivery

Aneuploidy

Non-chromosomal anomalies/ autosomal dominant mutations (due

to increased paternal age).

d) What is the aim of prenatal diagnosis? (2 marks)

Identification of abnormalities to prepare the parents

Offer termination

Identify conditions which may influence the timing of delivery

Enable adequate paediatric support to be available

Identify foetuses that may benefit from in utero treatment.

e) Name 2 prenatal tests and what they are used for. (2 marks)

58

Page 59: finals2009.wikispaces.com · Web viewPleural effusion which is secondary to hypoalbunimaemia Q4. What initial investigations would be implemented to confirm this diagnosis? Urine

Maternal serum -feto protein – neural tube defects

Triple test (HCG, MSAFP, oestriol) – aneuploidy

Ultrasound scan – soft markers (nuchal translucenecy in Down’s,

echogenic bowel in CF), structural abnormalities (congenital heart

disease, abdominal wall defects).

Amniocentesis: tissue sampling and karyotyping

Chorionic villus sampling: tissue sampling and karyotyping.

A 13 year old girl comes to see you with a 8 week history of amenorrhoea.

f) What are the possible diagnoses?

Pregnancy

Emotional disturbances (stress, exams)

Weight loss/athleticism

PCOS

Drugs (OCP)

Hyper/hypothyroidism

Hyperadrenalism

g) What first line investigation would you do?

Pregnancy test

h) What are the issues associated with an adolescent pregnancy? (2 marks)

Lack of concordance with prenatal care.

Nutritional issues – growth demands of adolescence

Vitamin deficiency

Iron deficiency (poor diet, heavy erratic periods)

Alcohol and drugs

Body image and dieting

Infections - UTI

STD

Other - lack of social support (partner, home, school)

Infant disadvantage.

A 34-year-old lady comes to see you as she has a positive pregnancy test. She has 7

children.

59

Page 60: finals2009.wikispaces.com · Web viewPleural effusion which is secondary to hypoalbunimaemia Q4. What initial investigations would be implemented to confirm this diagnosis? Urine

i) Define grand multiparity. (1 mark)

5 or more pregnancies carried to viability.

j) What risks are associated with this?

Abruptio placenta (due to age/HTN)

Placenta previa (due to previous scarring)

Post partum haemorrhage (due to atony and therefore lack of

contraction)

Precipitate delivery

k) What social issues need to be considered?

Childcare during and after pregnancy

Socio-economic factors

Contraceptive advice.

Mrs J, a 25 year old G2 P0 is referred to you in the antenatal clinic. She is a known to

have a problem with alcohol abuse and her midwife is concerned that she is continuing

to drink throughout her pregnancy.

l) List 3 facial features of Foetal Alcohol syndrome.

Absent/hypoplastic philtrum

Broad upper lip

Flattened nasal bridge

Micrognathia

Micropthalmia

Short nose

Short palpebral fissures

m) List 5 other features of Foetal Alcohol Syndrome.

Hyperactivity/irritability

Developmental delay

Growth deficiency

Mental retardation

Poor coordination

Congenital heart and joint defects

60

Page 61: finals2009.wikispaces.com · Web viewPleural effusion which is secondary to hypoalbunimaemia Q4. What initial investigations would be implemented to confirm this diagnosis? Urine

n) Give one other effect of drinking during pregnancy.

Increased spontaneous abortion

Restricted foetal growth

Preterm birth

Ms H G1 P0 is pregnant. She works as a nurse and had a needle-stick injury 1 year

ago. Having never been vaccinated, she contracted acute hepatitis B and has recently

had a blood test showing presence of Hepatitis B E antigen.

o) Is her foetus at risk of vertical transmission?

Yes (up to 90% if HBeAg present)

p) By what routes may Hep B be transmitted to Ms H’s baby?

Intrapartum exposure

Haematogenous

Transplacental

Breastfeeding.

q) What management plan would you follow?

Advise to abstain from alcohol/hepatotoxins

Notify staff on labour ward

Ensure baby receives Hep B immunoglobulin and vaccine

LFT’s

?amniocentesis

?delivery route

discuss breastfeeding

Mrs K is HIV positive and pregnant with her first child.

r) What maternal factors affect vertical transmission?

Advanced maternal disease

Low CD4 count

High viral load

Genital tract HIV viral load

Genital tract infections

61

Page 62: finals2009.wikispaces.com · Web viewPleural effusion which is secondary to hypoalbunimaemia Q4. What initial investigations would be implemented to confirm this diagnosis? Urine

s) What obstetric factors effect transmission?

Mode of delivery (reduced in c-section)

Duration of membrane rupture

Prematurity

t) What interventions would you recommend?

Antiretroviral therapy

Elective c-section

Formula feeding

u) What antiretroviral therapy would you prescribe?

Triple therapy – zidovudine, lamivudine and nevirapine

v) How would you manage the neonate?

Treat with antiretroviral therapy (depending on mothers viral load)

Diagnostic testing: HIV PCR at day 1, 1 month, 3 months, 6 months,

12 mnth

HIV antibody at 18 months

BCG not given until after 18 months.

2.10 Special situations

By the end of Phase II , students should be able to:

• recognise that special obstetric risks are associated with grand multi-parity, elderly

primigravidae, adolescent pregnancies and pregnancies in drug and alcohol abusers

• recognise the particular risks to patients and staff in circumstances of Hepatitis B

and HIV infection

62

Page 63: finals2009.wikispaces.com · Web viewPleural effusion which is secondary to hypoalbunimaemia Q4. What initial investigations would be implemented to confirm this diagnosis? Urine

2.11

Ms K is a 25 year old primigravida of 29 weeks gestation who presented with an

abdominal ‘tightening’ sensation which were becoming more painful and more

frequently in nature.

Define premature labour.

Premature labour is defined as labour occurring after 24 weeks and before 37 weeks

gestation.

Describe five characteristics of a woman who is at increased risk of developing

premature labour.

Body mass index <19

Low social class

Extremes of reproductive age (under 20 and over 35)

Domestic violence

Previous preterm labour

Others to mention – unsupported, smoking, previous preterm labour, bacterial

vaginosis and chronic medical conditions.

Name five possible causes of Ms K’s premature labour

Infection eg. Chorioamnionitis, maternal pyelonephritis, appendicits.

Uteroplacental ischaemia eg. Abruption

Uterine overdistension eg. Polyhydramnios, multiple pregnancy.

Cervical incompetence

Foetal abnormaility

Upon initial examination of Ms K in the EAU department describe what you would do.

Assess mother to see if she is stable (? shocked from a large abruption)

Assess for signs of a precipitant of preterm labour e.g. sepsis, polyhydramnios, and

severe pre-eclampsia, obstetric cholestasis.

63

Page 64: finals2009.wikispaces.com · Web viewPleural effusion which is secondary to hypoalbunimaemia Q4. What initial investigations would be implemented to confirm this diagnosis? Urine

Determine frequency and regularity of contractions

Sterile speculum examination – check for dilatation of the cervix

Begin foetal monitoring – CTG

Ascertain foetal presentation (cephalic/breech)

Inform paediatricians

Ultrasound examination – abnormal lie and presentation common in preterm labour.

What investigations would you carry out?

Blood tests

MSU and urinalysis

High vaginal and endocervical swabs

Assess for liquor using nitrazine sticks (PH – sensitive) – turn black, if it is not

obvious. False positives from infected vaginal discharge, semen, blood and urine

It was discovered that Ms K membranes were not ruptured, cervix was approx 1 cm

dilated. There was no obvious precipitant or cause. What treatment should be initiated

at this stage?

Corticosteriods – betametasone or dexamethasone 12mg/12h IM twice promotes

foetal surfactant production, lowering mortality and complications of Respiratory

Distress syndrome. Use before 34 weeks only.

Antibiotics – to be used if membranes rupture or obvious infection

Consider tocolysis

Monitor maternal pulse, BP, temperature, contractions and foetal heart rate.

Tocolytic therapy was initiated. Name two known drugs and describe its mode of

action.

Ritodrine/salbutomol – beta agonist, act on beta – receptors in myometrium to cause

relaxation.

Nifedipine – calcium channel blocker – block calcium channels in the myometrium,

interrupting contraction.

64

Page 65: finals2009.wikispaces.com · Web viewPleural effusion which is secondary to hypoalbunimaemia Q4. What initial investigations would be implemented to confirm this diagnosis? Urine

Mention a side effect of each named drug.

Ritodrine – pulmonary oedema, tachycardia, hyperglycaemia and hypokalaemia.

Nifedipine – potential to alter uteroplacental bloodflow causing foetal compromise.

Maternal side-effects headache, flushing and tremor.

2.11 Preterm labour

By the end of Phase II , students should be able to:

• recognise the circumstances when preterm labour may occur

• outline management to the patient

65

Page 66: finals2009.wikispaces.com · Web viewPleural effusion which is secondary to hypoalbunimaemia Q4. What initial investigations would be implemented to confirm this diagnosis? Urine

2.12

Mandy, aged 27, is expecting her first baby. No known complications are identified.

Early in her pregnancy, she discusses the delivery options with her midwife. Initially

Mandy is in favour of a home birth. But she later changes her mind, choosing to deliver

the baby in hospital.

A) Suggest two reasons why Mandy may have decided to opt for a hospital based birth.

(any 2 from below)

Primigravid women are more likely to experience failure to progress in the first or

second stages of labour than multigravid women. This would require hospital

transfer.

Regional anaesthesia,

Resuscitation of the newborn,

Management of postpartum complication are all more difficult at a home delivery,

compared with a hospital delivery.

In her 39th week of pregnancy Mandy attends the maternity unit. She is experiencing

contractions at a rate of one every 5-10 minutes and believes that she is in labour.

B) List 3 symptoms may Mandy have experienced, which herald the onset of labour?

Regular uterine contractions.

Rupture of membranes.

Show.

C) What is the definition of labour?

The occurrence of regular uterine contractions accompanied by cervical dilatation.

The midwife confirms that Mandy has started the first stage of labour.

D) Define the three stages of labour.

66

Page 67: finals2009.wikispaces.com · Web viewPleural effusion which is secondary to hypoalbunimaemia Q4. What initial investigations would be implemented to confirm this diagnosis? Urine

First stage: From the onset of labour until the cervix is fully dilated.

Second stage: From full cervical dilation to birth of the baby.

Third stage: From birth of the baby to delivery of the placenta.

The midwife initiates monitoring of the progress of Mandy’s labour using a partogram.

E) What four factors are recorded on a partogram?

Cervical effacement.

Cervical dilatation.

Descent of presenting part.

Uterine contractions

The wellbeing of Mandy and her foetus are also monitored.

F) List two parameters used to assess maternal wellbeing during labour, and two to

assess foetal wellbeing.

Maternal (any 2 of below):

Blood pressure

Pulse

Temperature

Pain relief

Fluid balance

Foetal (any 2 of below):

Foetal heart rate

Colour of liquor

Foetal scalp pH

A short cardiotocograph recording is taken on admission.

G) What parameters should be used to interpret a cardiotocograph recording?

Remember: “BRAVADO”

67

Page 68: finals2009.wikispaces.com · Web viewPleural effusion which is secondary to hypoalbunimaemia Q4. What initial investigations would be implemented to confirm this diagnosis? Urine

BR The baseline rate.

A Acceleration.

VA Variability.

D Decelerations.

Overall impression.

F) Indicate two findings on a cardiotocograph that may indicate foetal distress.

(any 2 of following)

The baseline rate <110 or >160 beats/minute.

Accelerations absent for over 20 minutes.

Variability <5 beats/minute.

Decelerations not associated with contractions.

Overall impression non-reassuring.

F) List 4 indications for continuous cardiotocograph monitoring.

(any 4 of following)

Preterm infant (<37 weeks completed gestation).

Foetuses that are or are suspected to be small for gestational age.

Multiple pregnancies.

Women with epidural analgesia.

Women with Syntocinon augmentation of labour.

Women who have been induced.

Women who are hypertensive.

Women with major medical disorders, including diabetes.

Women who develop meconium staining of the amniotic fluid during labour.

Women who undergo a trial of uterine scar.

If a foetal heart abnormality is recorded with the Pinard stethoscope/sonicaid.

G) Which drug is routinely given during for the third stage of labour?

When should it be administered?

Syndometrine.

68

Page 69: finals2009.wikispaces.com · Web viewPleural effusion which is secondary to hypoalbunimaemia Q4. What initial investigations would be implemented to confirm this diagnosis? Urine

Administered with the delivery of the anterior shoulder.

2.12        Normal labour

By the end of Phase II , students should be able to:

• describe the environments in which normal labour may be conducted and their relative

benefits and risks

• acknowledge the wishes of the mother in this decision

• recognise the onset of labour

• manage normal labour under supervision

• identify the drugs commonly used in labour and their benefits and risks

• participate in fetal surveillance in labour

• recognise the common signs of fetal distress 

69

Page 70: finals2009.wikispaces.com · Web viewPleural effusion which is secondary to hypoalbunimaemia Q4. What initial investigations would be implemented to confirm this diagnosis? Urine

2.14

Louise, a 25 year old woman, has successfully delivered her first baby, a healthy male,

by normal full term delivery. Normal third stage has completed, though she needed

repair of a third degree tear. She is wishing to breast feed but is experiencing some

difficulty.

Define the puerperium

6 week period from the completion of the 3rd stage of labour

What physiological changes take place during the puerperium?

e.g.s

Genital tract:

Uterine involution

Lochia

Endometrium regeneration and return of menstruation

CVS:

Fall in plasma volume and cardiac output

Return of BP to normal (if raised in pregnancy)

Blood:

Increased coagulability due to rise in platelets and clotting factors

U+E return to normal

Rise in HB and haematocrit with haemoconcentration

Urinary tract:

Reduction of physiological dilatation GFR fall to normal

Reduced bladder sensitivity to over-distension

List benefits of breast feeding

To mother

To baby

(from Lifespan;)

Benefits to mother:

70

Page 71: finals2009.wikispaces.com · Web viewPleural effusion which is secondary to hypoalbunimaemia Q4. What initial investigations would be implemented to confirm this diagnosis? Urine

post-partum recovery- uterine involution, less weight gained, figure

and physical fitness

psychological- bonding. Measure of success by society

Long-term health- lower risk osteoporosis, ovarian ca post-

menopausal, breast cancer

Benefits to baby:

Species specificity

Nutritional benefits

Emotional- bonding

Enhanced immunity- less infection

Lower risk cot death

Lower risk allergies

Coats gut- lower risk gastroenteritis

Cost

For what maternal related reasons may breast feeding be difficult?

Mastitis

Abscess formation

Postnatal depression/ psychosis

Multiple births

Exhaustion

What advise would you give to a mother such as Louise who is learning for the first

time to establish successful lactation?

Louise asks you about postnatal contraception. She has used the OCP previously, but

has heard that some drugs should not be given whilst breastfeeding, and wonders if

breastfeeding alone will be enough to protect her.

What contraceptive advice would you give her? What are her options and when should

any measures be started?

Lactation not adequate alone

Start contraception 4-6 wks post-delivery

OCP contraindicated if breastfeeding- suppresses lactation

Progesterone-only pill or depot is safe when breastfeeding

IUD safe- insert at end of 3rd stage or at 6 weeks (infection screen first).

71

Page 72: finals2009.wikispaces.com · Web viewPleural effusion which is secondary to hypoalbunimaemia Q4. What initial investigations would be implemented to confirm this diagnosis? Urine

drugs that should be avoided during breastfeeding, and their potential

effect on the baby

With your help, Louise successfully starts breastfeeding. She is managed on the ward

for a few days.

What routine assessments should be made daily postnatally? What common

problems in the puerperium should be identified?

Assess:

Uterine involution and lochia

Blood pressure

Temperature

Pulse

Perineal wound

Common problems:

Infection

Thrombo-embolism

Poor wound healing

Urinary incontinence

Urinary retention

Faecal incontinence

Feeding problems

Post-natal depression and psychosis

Louise develops pyrexia.

What are the most likely sources/causes? How would you assess and

manage it?

Sources/causes:

Genital tract infection- endometritis (± retained products of

conception)

Urinary tract infection

Breast- Mastitis

72

Page 73: finals2009.wikispaces.com · Web viewPleural effusion which is secondary to hypoalbunimaemia Q4. What initial investigations would be implemented to confirm this diagnosis? Urine

Wound infection

Lung infection

DVT

Examine- abdo, breasts, IV access sites, chest, legs

Cultures- Blood, urine, high vaginal, fetal

Broad spectrum antibiotics

Louise returns home with her new baby. A few days later, she is visited by her

midwife.

What symptoms might suggest post-natal depression? What are the risk

factors?

Tiredness

Guilt

Feelings of worthlessness

RF: Social/emotional isolation

Previous history

Pregnancy complications

What measures might you take in treatment

Social support

Psychotherapy

Antidepressants

2.14 Normal puerperium and lactation

By the end of Phase II , students should be able to:

• make routine assessments of patients to identify infection, thrombosis and feeding

problems and assess wound healing

• recognise the factors leading to the establishment of successful lactation

73

Page 74: finals2009.wikispaces.com · Web viewPleural effusion which is secondary to hypoalbunimaemia Q4. What initial investigations would be implemented to confirm this diagnosis? Urine

• advise patients on the use of drugs in lactation

• recognise the possibility of post-natal depression and psychosis

74

Page 75: finals2009.wikispaces.com · Web viewPleural effusion which is secondary to hypoalbunimaemia Q4. What initial investigations would be implemented to confirm this diagnosis? Urine

2.15

Mrs S is a 35 year old primigravida at 40+10 weeks gestation. She has been admitted

to the labour ward for induction of labor.

1) What is the rationale for induction at this timing?

Due to increased perinatal mortality rates/Caesarean section after 42 weeks.

1 mark

2) Give 3 circumstances where induction of labor would be contraindicated?

Previous caesarean section,

Multiple Pregnancy,

Maternal Cardiac disease,

Pre-eclampsia.

3 marks

The progress of labor was monitored using the Bishops score.

3) Name 4 of the criteria examined for a bishops score.

Dilation,

Consistency,

Length,

Station,

Position.

4 marks

Mrs S progressed through to the 2nd stage of labor. Her blood pressure (110/70), pulse

(75bpm), and temperature (37.1ºC) were all monitored. Contractions were 3 in every 10

minutes. During her contractions, fetal heart rate was monitored by CTG, and fetal

distress was noted.

4) List 3 signs of fetal distress that can be observed on a CTG.

Unprovoked decelerations,

75

Page 76: finals2009.wikispaces.com · Web viewPleural effusion which is secondary to hypoalbunimaemia Q4. What initial investigations would be implemented to confirm this diagnosis? Urine

Decreased variability,

Fetal bradycardia(<110bpm)/tachycardia(>175bpm),

Slow recovery of fetal heart rate following a deceleration.

3 marks

Due to the fetal distress and a prolonged 2nd stage, assisted ventouse delivery was

attempted.

5) Give two complications of a ventouse delivery, and 2 complications of a forceps

delivery.

Ventouse:

Cephalo/skull haematoma, Scalp laceration, Vaginal tears, Neonatal jaundice.

Forceps:

Vaginal tears, Ruptured uterus, Facial nerve palsy.

4 marks

Following successful delivery, Mrs S was noted to have lost 500ml of blood and was

continuing to lose blood.

6) Give 3 causes of post partum haemorrhage

Caesarean section,

Atonic uterus,

Retained placenta,

Trauma during delivery.

3 marks

7) What drug is commonly used at the end of the 2nd stage of labor to aid the progress

of the 3rd stage, and what is it a combination of?

Syntometrine, combination of Ergometrine and Oxytocin.

2 marks

76

Page 77: finals2009.wikispaces.com · Web viewPleural effusion which is secondary to hypoalbunimaemia Q4. What initial investigations would be implemented to confirm this diagnosis? Urine

2.15 Abnormal labour

By the end of Phase II , students should be able to:

• describe the role of induction of labour and the possible methods

• recognise common fetal and maternal problems in the first stage of labour

• participate in the management of delay in the first stage of labour including the use of

oxytocin and monitoring techniques

• recognise delay in the second stage of labour and the circumstances when it may occur

• recognise the common complications of the third stage of labour and describe how they

may be prevented

• recognise and participate in the immediate management of post-partum haemorrhage

• discuss with patients the need for operative delivery including Caesarean Section

• debate critically the role of Caesarean Section

77

Page 78: finals2009.wikispaces.com · Web viewPleural effusion which is secondary to hypoalbunimaemia Q4. What initial investigations would be implemented to confirm this diagnosis? Urine

3.2

Mr Phillips, a 68 year old retired gardener comes to see you his GP complaining of

difficulty passing water which has been blood stained.

After skilfully eliciting the history, you suspect the prostate to be the cause of Mr

Phillips’ troubles

a) Apart from examining Mr Phillips’ abdomen, which examination is indicated here?

Digital rectal examination

b) List four other indications of when to perform the examination mentioned in a.

Suspected lower GI

prostate

cauda equina/ back injury

Rectal bleeding

Severe constipation

Pain in the anal or rectal area

Suspected enlarged prostate gland

Suspected rectocele.

c) List four things you would normally inspect for whilst performing this examination

(in any case, not just for Mr Phillips).

Rectal prolapse

fistulo-in-ano

skin tags

anal fissure

anal warts

haemorrhoids

anal carcinoma

pruritus ani

excoriation from diarrhoea

78

Page 79: finals2009.wikispaces.com · Web viewPleural effusion which is secondary to hypoalbunimaemia Q4. What initial investigations would be implemented to confirm this diagnosis? Urine

d) Give two distinguishing features for each of prostatitis, prostatic carcinoma and

BPH upon palpation of the prostate gland.

Hard and nodular in prostatic carcinoma

boggy and tender in prostatitis

smooth and diffusely enlarged in BPH

e) Assessment of anal tone is an important aspect of digital rectal examination. Which

type of muscle comprise the internal and external sphincters of the anus and what is

their respective innervation?

Internal is involuntary smooth muscle, sympathetic innervation via the hypogastric

and pelvic plexus

External is striated voluntary muscle. The proximal aspect is in intimate relationship

with the puborectalis muscle. The pudendal nerve, specifically the inferior rectal

branch, innervates the external anal sphincter.

Question 2

Mrs Wilson is a 56 year old school teacher. She has two children and is overweight

with a BMI of 28. She has come to see you the HO in A&E with a 3 month history of

RUQ/epigastric pain, colicky in nature and is occasionally relieved by food. She also

has a previous history of a peptic ulcer.

a) List two likely differential diagnoses that fit with Mrs Wilson presenting complaints.

Chronic cholecystitis

Biliary colic

Duodenal/gastric ulcer

GORD

b) Name two treatments for Mrs Wilsons’ symptoms?

79

Page 80: finals2009.wikispaces.com · Web viewPleural effusion which is secondary to hypoalbunimaemia Q4. What initial investigations would be implemented to confirm this diagnosis? Urine

Lifestyle and diet modification – avoid spicy food, stop smoking,

weight loss

Simple antacids - gaviscon

Proton pump inhibitor – omeprazole

H2 Receptor antagonists – ranitidine

c) List two further investigations required at this stage to identify the cause for Mrs

Wilsons problems.

H-pylori testing: CLO test or Urea Breath test

Upper GI endoscopy

Ultrasound scan abdomen

d) List 4 indications for upper GI endoscopy apart from weight loss and age.

Symptoms > 4 weeks

Dysphagia/odynophagia

Persisting symptoms despite treatment

Relapsing symptoms

Persisting vomiting

GI bleeding

e) Outline 3 of the potential complications of upper GI endoscopy

Problems related to sedation/medication

Bleeding

Perforation

f) Following the upper GI endoscopy, Mrs Wilson became unwell with abdominal pain

made worse by coughing. What does her chest x-ray show and what is the commonest

cause of this condition.

80

Page 81: finals2009.wikispaces.com · Web viewPleural effusion which is secondary to hypoalbunimaemia Q4. What initial investigations would be implemented to confirm this diagnosis? Urine

X-ray shows pneumoperitoneum (gas shown under both sides

diaphragm)

Commonest cause is from perforated gastroduodenal ulcer.

81

Page 82: finals2009.wikispaces.com · Web viewPleural effusion which is secondary to hypoalbunimaemia Q4. What initial investigations would be implemented to confirm this diagnosis? Urine

Question 3

Mr Hughes, a 34 year old stock broker, attends A&E with a 11 day history of yellowing

of his skin and abdominal pain. He has noticed darkening of his urine and his motions

have become putty coloured in the last three days.

a) Give four likely differential diagnoses for this patient?

Pancreatitis

Alcoholic liver disease

Acute hepatitis

Choledocholithiasis (gallstone in CBD)

Drugs (paracetamol)

Ascending cholangitis

His liver function blood results are as follows

LFT:

Bilrirubin (conjugated) – 68 (3-17mol/L)

Alkaline Phosphatase – 300 (40-120u/L)

ALT – 40 (3-35u/L)

b) What do these blood results imply is the likely pathological process? And what

other biochemical parameters would you like to know?

Diagnosis - Obstructive/cholestatic jaundice

Biochemical parameters - Gamma GT & Serum Amylase

c) State one further investigation you would arrange to determine the cause.

Abdominal ultrasound scan

ERCP

d) Mr Hughes has an ERCP arranged by your registrar. Give two indications for this

test and state three complications that can occur.

82

Page 83: finals2009.wikispaces.com · Web viewPleural effusion which is secondary to hypoalbunimaemia Q4. What initial investigations would be implemented to confirm this diagnosis? Urine

Indications (diagnostic)

Cholangitis

Jaundice with intra hepatic duct dilatation

Jaundice with normal ducts and a non diagnostic biopsy

Recurrent pancreatitis

Post cholecystectomy pain

Indications (therapeutic)

Sphincterotomy for CBD stones

Stenting for malignant strictures

Complications

Acute pancreatitis

Bleeding

Cholangitis

Perforation

Extended Question

Briefly outline the normal metabolism of bilirubin.

Bilirubin is breakdown product of haemoglobin.

It is conjugated with glucuronic acid by hepatocytes and becomes water

soluble.

The conjugated bilirubin is secreted into bile and then passes in to the gut.

Some is taken up by the liver (enterohepatic circulation) with the remainder

being converted to urobilinogen by gut bacteria.

Urobilinogen is either reabsorbed or excreted by the kidneys, or converted

stercobilin which colours faeces brown

83

Page 84: finals2009.wikispaces.com · Web viewPleural effusion which is secondary to hypoalbunimaemia Q4. What initial investigations would be implemented to confirm this diagnosis? Urine

3.2 Investigations

By the end of Phase II students should be able to:

• perform and interpret digital rectal examination

• recognise the radiological features of intra-peritoneal air, obstructed bowel and correct

placement of naso-gastric tubes

• explain to a patient the reasons for conduct of and possible complications of upper

gastro-intestinal endoscopy, colonoscopy ERCP and contrast radiography

• request abdominal ultrasound and plain abdominal radiographs and chest radiographs

appropriately seeking advice where necessary

• interpret liver function tests and routine liver serological and immunological

investigations

relate the results to the underlying pathology

84

Page 85: finals2009.wikispaces.com · Web viewPleural effusion which is secondary to hypoalbunimaemia Q4. What initial investigations would be implemented to confirm this diagnosis? Urine

A 68 year old gentleman presents with a 3 month history of nocturia, poor stream,

increased frequency, urgency and hesitancy.

The SHO suspects Benign Prostatic Hyperplasia and feels she needs to perform a

digital rectal examination (DRE).

How would you perform a digital rectal examination whilst maintaining dignity for the

patient?

Fully explain the procedure to the patient.

Make sure you will not be interrupted

Ensure the patient is kept as comfortable and warm as possible.

Lie patient on their left side with knees pulled to chest.

Use gloves and lubricant.

Part the buttocks.

Ask the patient to take a deep breath and gently insert your finger into the anus.

Feel with finger pad and twist with finger tip.

When finished, wipe anus.

Inspect glove for faeces, blood.

You palpate the prostate gland. Typically, how would you differentiate between a

patient with BPH and a patient with Prostate Cancer?

A normal prostate is the size of a walnut.

In BPH-the gland may still feel smooth, yet is enlarged.

In prostate cancer, the gland may show asymmetrical, nodular enlargement. It may

feel characteristically stony hard and irregular with obliteration of the median sulcus.

Name 5 differential diagnoses which would require a DRE to be performed?

BPH

Prostate Cancer

Cauda Equina Syndrome- assess anal tone by asking patient to squeeze your finger.

Bowel Cancer- feel for masses and inspect glove for blood

Chronic Constipation- feel for impacted faeces and inspect glove for faeces.

.

Objective: Digital rectal examination

85

Page 86: finals2009.wikispaces.com · Web viewPleural effusion which is secondary to hypoalbunimaemia Q4. What initial investigations would be implemented to confirm this diagnosis? Urine

3.4

1. A 35 year old gentleman presents to his GP complaining of recent onset dyspepsia

a) Name 5 likely differentials to consider

Duodenal ulcer

Gastro-oesophageal reflux

Functional dyspepsia MI

Gastric ulcer

Gastric malignancy

Gallstones

Gastritis

Oesophagitis

Pneumonia

b) What do you understand by the term dyspepsia?

A non-specific group of symptoms related to the upper GI tract including epigastric

pain related to hunger, eating specific foods or time of day; associated bloating;

heartburn which may be associated with acid reflux.

c) On further history taking he revealed he suffered with epigastric pain which was

worse before meals and at night and relieved by eating and drinking milk; he denied

any ‘alarm’ symptoms. What is the most likely diagnosis?

Duodenal ulcer (but don’t rely on history alone)

d) What are the main risk factors involved?

Helicobacter pylori*

Smoking

Increased gastric acid secretion

Drugs (aspirin, NSAIDs, steroids)*

Blood group O

Increased gastric emptying

*Major factors

86

Page 87: finals2009.wikispaces.com · Web viewPleural effusion which is secondary to hypoalbunimaemia Q4. What initial investigations would be implemented to confirm this diagnosis? Urine

e) Under what circumstances is referral for urgent endoscopy necessary?

Patients of any age with dyspepsia when presenting with any of the following:

Chronic GI bleeding

Progressive unintentional weight loss

Progressive difficulties swallowing

Persistent vomiting

Iron deficiency anaemia

Epigastric mass

Suspicious barium meal

Also patients aged 55 years and over with unexplained and persistent (4-6/52) recent

onset dyspepsia alone.

f) Explain possible initial pharmacological therapies for this patient (mention 3).

Initial strategy proton pump inhibitor (PPI) full dose for a month

Testing for and treating H.pylori. H.pylori can be initially detected using either a

carbon-13 urea breath test or a stool antigen test. If positive start eradication

programme ‘triple therapy’, a 7 day twice daily course of: full dose PPI (e.g.

lansoprazole 30mg bd) + metronidazole 400mg + clarithromycin 250mg; or full dose

PPI + amoxicillin 1g and clarithromycin 500mg (Insufficient evidence to guide which

should be offered first, NICE guidelines 2004)

If no response to above try H2 receptor antagonists (ranitidine or cimetidine) or a

prokinetic;

No response to above treatments refer for 2nd opinion

g) Before beginning pharmacological therapies what initial advice may you offer?

Review medications for possible causes of dyspepsia (steroids, NSAIDs, Ca

antagonists, nitrates, theophyllines, bisphosphonates). Use of simple antacids

Stop smoking

Reduce alcohol intake

Avoid food that worsens symptoms

Weight reduction

87

Page 88: finals2009.wikispaces.com · Web viewPleural effusion which is secondary to hypoalbunimaemia Q4. What initial investigations would be implemented to confirm this diagnosis? Urine

h) Peptic ulcer surgery (including antrectomy with vagotomy and subtotal gastrectomy

with Roux en Y) is nowadays rarely performed due to the success of medical therapy.

However you may encounter patients who are now experiencing the long-term

complications of ulcer surgery. Briefly describe 3 of these long-term complications.

Dumping syndrome – fainting, vertigo, sweating, which may be due to the osmotic

effect of rapid transit of food from the stomach into the small intestine. Fluid is

reabsorbed into the jejunum causing temporary hypovolaemia.

Anaemia – may be due to iron deficiency because hydrochloric acid is needed for

iron absorption or due to vitamin B12 deficiency because intrinsic factor is required.

Both hydrochloric acid and intrinsic factor are absent after partial gastrectomy.

Steatorrhoea and diarrhoea

Bile reflux and vomiting

Small stomach syndrome

Recurrent ulceration and malignancy in gastric remnant

Osteomalacia

i) What do you understand by the term ‘non-dysfunctional dyspepsia’?

Functional means symptoms occur in the absence of any demonstrable

abnormalities. Functional dyspepsia can present with a range of symptoms including

upper abdominal pain/discomfort, fullness/early satiety, bloating and nausea.

2. A 67 year old gentleman presents to his GP complaining of a 3-4 month history of

epigastric pain/discomfort and unintentional weight loss.

a) What other features in the history may suggest a gastric carcinoma, mention 3?

Anorexia

Dysphagia

Upper GI bleed

Metastatic disease

Vomiting

Anaemia

88

Page 89: finals2009.wikispaces.com · Web viewPleural effusion which is secondary to hypoalbunimaemia Q4. What initial investigations would be implemented to confirm this diagnosis? Urine

Mass in abdomen

b) What clinical signs may be evident, name 5?

Epigastric mass

Hepatomegaly

Jaundice

Ascites

Troissier’s sign (Virchow’s node)

Acanthosis nigricans

c) What risk factors are associated with gastric carcinoma, name 5?

Diet – nitrosamines, pickling methods

Blood group A

Pernicious anaemia

Smoking

Alcohol

H.pylori infection

Atrophic gastritis

d) Name 3 types of tumour which can be found in the stomach

Adenocarcinoma (commonest)

Lymphoma

Leiomyomas

Leiomyosarcomas

neurofibromas

GI stromal tumours (GISTs)

e) Describe what you may find on macroscopic examination?

Polypoid, ulcerating or infiltrative (Borrmann classification)

If widespread can cause linitis plastica (leatherbottle stomach)

f) How and where may gastric carcinomas spread?

Direct invasion

89

Page 90: finals2009.wikispaces.com · Web viewPleural effusion which is secondary to hypoalbunimaemia Q4. What initial investigations would be implemented to confirm this diagnosis? Urine

Lymphatic spread – Virchow’s node

Haematogenous spread – liver, lung, brain

Transcoelomic e.g. to ovaries (Krunkenberg tumour)

g) What investigations may this patient undergo, name 5?

Urgent upper GI endoscopy – diagnostic - multiquadrant biopsy of all gastric ulcers

Bloods – FBC (anaemia), LFTs (mets)

Barium meal

CT/MRI – staging

Bone scan – staging

Endoscopic ultrasound – assess depth of gastric invasion and local lymph node

involvement

Diagnostic laparoscopy – used to exclude undiagnosed peritoneal or liver

secondaries prior to consideration of resection.

3) A 45 year old lady presents to A&E with severe right upper quadrant pain and

vomiting?

a) What differential diagnoses would you consider?

Biliary colic

Acute cholecystitis

Pancreatitis

Peptic ulcer disease

Hepatitis

Ruptured aneurysm

b) On further history taking she reveals she has experienced similar episodes of pain

over the past few months but never as severe and as prolonged as the current pain.

The pain radiates to the right tip of the scapula and is precipitated by fatty foods. On

examination the patient is lying still with shallow breathing. What other signs may you

find on examination?

Tenderness

Guarding

Murphy’s sign

90

Page 91: finals2009.wikispaces.com · Web viewPleural effusion which is secondary to hypoalbunimaemia Q4. What initial investigations would be implemented to confirm this diagnosis? Urine

Tachycardia

Low grade pyrexia

Mass

Jaundice (if stone moves into CBD)

c) Name the most important investigation and likely findings.

Abdominal ultrasound scan - shows gallstones, thickened gallbladder wall, checks

dilatation of biliary tree indicative of obstruction.

d) How would you manage this patient?

NBM

IV fluids

Strong analgesia e.g. pethidine

Antibiotics e.g. cefuroxime and metronidazole

Surgery – controversial issue regarding timing of surgery for patients with acute

cholecystitis. OHCM states in suitable candidates do cholecystectomy within 72h,

Early surgery is associated with fewer complications. If delayed, relapse occurs in

18%. Otherwise cholecystectomy deferred for 6-12 weeks until the inflammation has

settled.

Percutaneous cholecystostomy - elderly or high risk patients unsuitable for surgery

e) Name 5 complications of gallstones.

Biliary Colic

Acute cholecystitis

Chronic cholecystitis

Empyema

Gangrene/perforation

Cholestatic jaundice

Ascending cholangitis

Acute pancreatitis

Gallstone ileus

4) A 58 year old gentleman presents to his GP complaining of retrosternal pain

91

Page 92: finals2009.wikispaces.com · Web viewPleural effusion which is secondary to hypoalbunimaemia Q4. What initial investigations would be implemented to confirm this diagnosis? Urine

exacerbated by lying down and drinking alcohol and associated with regurgitation of

gastric contents.

a) Name the most likely diagnosis to consider?

Gastro-oesophageal reflux disease (GORD)

b) Name 5 risk factors associated with this disorder.

Hiatus hernia

Obesity

Pregnancy

High caffeine intake

Alcohol

Smoking

Systemic sclerosis

Increased gastric volume (large meals, delayed gastric emptying)

Drugs (TCA, anti-cholinergics, Ca channel blockers, Nitrates)

c) Briefly describe the underlying mechanisms responsible for this disorder.

Normally reflux of acid is prevented by

the lower oesophageal sphincter (LOS)

angle of His

crural fibres of the diaphragm

prominent mucosal folds

positive intra-abdominal pressure acting on the LOS

Therefore failure of one or more of these mechanisms or if hiatus hernia present

allows gastric contents to reflux into the oesophagus causing oesophagitis.

d) Briefly describe 4 complications

Oesophagitis – ulceration of oesophageal mucosa (mild to severe)

Barrett’s oesophagus – a premalignant disorder, metaplastic change from normal

squamous to columnar epithelium

Barrett’s ulcer – may bleed or perforate

Iron deficiency anaemia – blood loss from severe oesophagitis

Stricture – fibrosis may result from healing oesophagitis

92

Page 93: finals2009.wikispaces.com · Web viewPleural effusion which is secondary to hypoalbunimaemia Q4. What initial investigations would be implemented to confirm this diagnosis? Urine

Oesophageal malignancy – adenocarcinoma

e) Name 3 investigations and possible findings.

Upper GI endoscopy – directly visualise, biopsy and cytological brushings to confirm

presence of oesophagitis and to grade its severity and exclude malignancy.

Barium swallow – can show structural abnormalities

24h oesophageal pH monitoring – significant reflux if pH <4 for >4.7% of time.

f) Briefly describe lifestyle advice?

Weight loss

Raise bed head

Small regular meals

Avoid hot drinks, alcohol and eating <3h before bed

Avoid drugs affecting oesophageal motility (nitrates, anticholinergics, TCA) or that

damage the mucosa (NSAIDs)

g) How would you further manage this patient?

Initially try antacids or alginates.

If symptoms persist for >4wks (or weight loss; dysphagia; excessive vomiting; GI

bleeding) refer for GI endoscopy.

Oesophagitis confirmed – try PPI

Pro-kinetic drugs

Surgery – Nissen fundoplication if symptoms are very severe and there is

radiological or pH-monitoring evidence of severe reflux.

3.4 Dyspepsia

By the end of Phase II students should be able to:

• recognise and distinguish between peptic ulcer disease, gastro-oesophageal

reflux disease and functional dyspepsia clinically and on investigation including

investigation for H. pylori infection

93

Page 94: finals2009.wikispaces.com · Web viewPleural effusion which is secondary to hypoalbunimaemia Q4. What initial investigations would be implemented to confirm this diagnosis? Urine

• initiate management for these conditions including treatment for H. pylori infection

• communicate to a patient the diagnosis of peptic ulcer disease with an explanation

of the management including life-style changes

• recognise the possibility of gastric cancer in older patients with upper abdominal

symptoms

• recognise the possibility of gall bladder disease in patients with upper abdominal

pain and to initiate appropriate investigations 

94

Page 95: finals2009.wikispaces.com · Web viewPleural effusion which is secondary to hypoalbunimaemia Q4. What initial investigations would be implemented to confirm this diagnosis? Urine

3.6

You are called to see a 53year old man in A & E who looks unwell with a large,

distended abdomen and jaundice. He is unkempt and smells strongly of alcohol.

a) At what plasma bilirubin level is jaundice visible? (1mark)

35mol/L

b) Give a brief description of bilirubin metabolism. (3marks)

Formed from breakdown of Hb

Conjugated with Glucoronic acid by hepatocytes to make it water soluble

Conjugated bilirubin secreted into bile & out into gut

Some passes into enterohepatic circulation

Rest converted to Urobilinogen by gut bacteria

Urobilinogen either:

Reabsorbed and excreted by kidneys

Converted to stercobilin which colours faeces brown

Jaundice can be classified by the site that is causing the problem. State the 3 types

and give an example of each. (3marks) Pick one of the types and fill in the table

below to show what you would expect to find when testing LFT’s – State if the level

would be increased, decreased or within the normal range. (2marks)

Pre-hepatic Hepatocellula

r

Post-

Hepatic

Bilirubin ↑ ↑ ↑

AST N ↑ ↑

ALP N ↑ or N ↑

GGT N ↑ ↑

PRE-HEPATIC

Physiological = neonatal

Haemolysis

Glucuronyl transferase deficiency = Gilbert’s syndrome, Crigler-Najar syndrome

LFT’s:

95

Page 96: finals2009.wikispaces.com · Web viewPleural effusion which is secondary to hypoalbunimaemia Q4. What initial investigations would be implemented to confirm this diagnosis? Urine

Bilirubin (micromoles/l) 50-150 (normal range 3-17)

AST I.U. < 35 (normal range <35)

ALP I.U. <250 (normal range <250)

Gamma GT I.U. 15-40 (normal range 15-40)

HEPATOCELLULAR

Viruses = Hep. A, B, C, CMV, EBV

Drugs = Paracetamol overdose, Anti-TB (RIP), Sodium Valporate etc.

Alcoholic hepatitis

Cirrhosis

Hepatic Metastases

Haemochromatosis

Wilson’s Disease

Budd-Chiari syndrome

Dubin-Johnson syndrome

1-antitrypsin deficiency

Toxins = carbon tetrachloride

LFT’s:

Bilirubin (micromoles/l) 50-250 (normal range 3-17)

AST I.U. 300-3000 (normal range <35)

ALP I.U. <250-700 (normal range <250)

Gamma GT I.U. 15-200 (normal range 15-40)

CHOLESTATIC (OBSTRUCTIVE)

Gallstones in common bile duct

Pancreatic cancer

Drugs = Antibiotics (flucloxacillin, fusidic acid, co-amoxiclav), Oral contraceptives,

Clorpromazine etc.

Cholangiocarcinoma

Sclerosing cholangitis

Primary biliary cirrhosis

Biliary atresia

LFT’s:

Bilirubin (micromoles/l) 100-500 (normal range 3-17)

AST I.U. 35-400 (normal range <35)

ALP I.U. >500 (normal range <250)

Gamma GT I.U. 30-50 (normal range 15-40)

96

Page 97: finals2009.wikispaces.com · Web viewPleural effusion which is secondary to hypoalbunimaemia Q4. What initial investigations would be implemented to confirm this diagnosis? Urine

You want to take a history from this man. List 4 pertinent questions that you would

like to ask. (4marks)

Blood transfusions

IV Drug Use

Body piercing/tattoos

Sexual Hx

Travel abroad

Alcohol consumption

FHx of jaundice

Medications

Contact with other jaundiced people

From the history you discover that this man has been drinking heavily for the past

10years. State the most likely diagnosis and two things might you expect to find on

examination of this man. (2marks)

Diagnosis = Alcoholic liver disease causing cirrhosis/hepatic failure.

Palmar Erythema

Spider naevi

Gynaecomastia

Bruising

Scratch marks

Dupytren’s contracture

Clubbing

Leuconychia

Testicular atrophy

Hepatomegally

Ascites

Based upon your most likely diagnosis from (e), what investigations would you like

to carry out for this patient? (3marks)

Bloods

FBC

Folate & B12

LFT’s – Alk. Phos., ALT, AST, Bilirubin

GGT

97

Page 98: finals2009.wikispaces.com · Web viewPleural effusion which is secondary to hypoalbunimaemia Q4. What initial investigations would be implemented to confirm this diagnosis? Urine

Albumin

PT/INR

WCC

Platelets

Virology – rule out infective cause

Ascitic Tap

Liver USS

You diagnose alcoholic liver cirrhosis. How would you manage this patient’s

ascites? (2marks)

Fluid restriction

Spironolactone (100mg/24h PO, every 48rs to 400mg/24h)

Add Furosemide (<120mg/24h PO)

Monitor U & E’s, Cr

Chart weight loss = aim for 0.5kg/day

You hear your consultant talking about portal hypertension and porto-systemic

anastomoses.

What is portal hypertension and how does it occur? (1mark)

Damage to liver causes a raised pressure in the venous portal system.

Raised pressure stimulates expansion of collaterals between the portal and

systemic system

Name the four sites of porto-systemic anastomoses. (2marks)

Oesophagus = Haematemesis

Umbilical = Caput medusa

Anus = Haemorrhoids (rectal bleeding)

Retro-peritoneal

3.6 Jaundice & hepatomegally

By the end of Phase II students should be able to:

- distinguish pre-hepatic, hepatic, post-hepatic jaundice on clinical & biochemical

grounds

- distinguish between infectious and mechanical causes of biliary obstruction

- initiate appropriate investigations

- distinguish the common causes of hepatomegally on clinical grounds

- initiate investigations for hepatomegally

98

Page 99: finals2009.wikispaces.com · Web viewPleural effusion which is secondary to hypoalbunimaemia Q4. What initial investigations would be implemented to confirm this diagnosis? Urine

- recognise the manifestations of chronic liver disease including encephalopathy and

portal hypertension

- recognise the situations associated with acute hepatic failure, the signs of hepatic

failure and initiate immediate management

99

Page 100: finals2009.wikispaces.com · Web viewPleural effusion which is secondary to hypoalbunimaemia Q4. What initial investigations would be implemented to confirm this diagnosis? Urine

Mr Best, who admits to an alcohol intake of 20 cans of lager per day, has a 3 month history of jaundice, abdominal swelling, shortness of breath and itching. Looking through his previous admission notes you see that cirrhosis of the liver was diagnosed.

What is cirrhosis

Loss of normal liver architecture with diffuse fibrosis and nodular degeneration which is usually irreversible

In late cirrhosis, a small liver can be found. However, earlier on in the disease a patient may have hepatomegaly. Apart from cirrhosis name 3 causes of hepatomegaly.

CCFInfectious causes e.g. acute hepatitis, weils disease, Epstein barr virus Carcinoma, neoplasmCholangitisHaematological -Leukaemia, lymphoma, thalassaemiaVascular – e.g. amyloidReidel’s lobeReye’s syndromeAbcess, cystTricuspid incompetence (pulsatile)

As well as alcohol abuse, name three causes of cirrhosisInfection Hepatitis – viral, autoimmune, fungal

Autoimmune Primary sclerosing cholangitis Autoimmune liver disease

Genetic causes alpha 1 antitrypsin deficiency Haemochromatosis Copper deposition (wilson’s disease)

100

Page 101: finals2009.wikispaces.com · Web viewPleural effusion which is secondary to hypoalbunimaemia Q4. What initial investigations would be implemented to confirm this diagnosis? Urine

drugs – e.g. amiodarone, methyldopa, methotrexatehepatic venous outflow obstruction: venoocclusive disease, Budd Chiari "cardiac cirrhosis" from constrictive pericarditis

What abnormalities in blood results (FBC, LFT etc) would you expect in someone with chronic liver disease associated with alcohol abuse(values not required)

FBC –mean cell volume increaseDecreased platelets and WCCIncreased gamma GTIncreased AP and ASTDecreased albumin Increased bilirubin (jaundice)INR/PT increase

State the test you would perform to confirm your suspicion of ascites on examination

One of:Shifting dullnessFluid thrill(‘Dipping’ may also be done to look for masses underlying ascites)

What other investigations would you carry out in suspected ascites

UltrasoundDiagnostic paracentesis

Name three ways in which ascites is managed?

Bed rest

101

Page 102: finals2009.wikispaces.com · Web viewPleural effusion which is secondary to hypoalbunimaemia Q4. What initial investigations would be implemented to confirm this diagnosis? Urine

Fluid restrictionLow salt dietSpironolactone (frusemide added if no response)Observation: daily weights, U&E, creatinine (stop diuretics if Na↓ or creatinine ↑)Paracentesis (with albumin infusion)

Mr Best has signs of encephalopathy and portal hypertension. Name two presentations for each.

Encephalopathy:

Liver flapConfusionRestlessnessSlurred speech, incoherent speechCoordination problemsDrowsiness, stuporComaFoetor hepaticushyperreflexia

Portal hypertension:

ascites - with low plasma albumin porto-systemic shunts - e.g. caput Medusae, oesophageal varices haematemesis or melaena - due to rupture of gastro - oesophageal varices venous hum of abdomenhaemorrhoids peripheral oedema(increased risk of hepatocellular carcinoma)

Name three hand signs of liver disease

Palmar erythemaBrusing

102

Page 103: finals2009.wikispaces.com · Web viewPleural effusion which is secondary to hypoalbunimaemia Q4. What initial investigations would be implemented to confirm this diagnosis? Urine

Terry’s nailsDupytren’s contractureGynecomastiaClubbingAsterixisleuconychia

Objective: Chronic liver disease

103

Page 104: finals2009.wikispaces.com · Web viewPleural effusion which is secondary to hypoalbunimaemia Q4. What initial investigations would be implemented to confirm this diagnosis? Urine

3.7

Whilst working in A&E as a newly qualified F1 doctor you are presented with Ruddiger

a 52 year old male. From the history you ascertain that Ruddiger has had an alcohol

problem for the past 10 years following the loss of a jackpot winning lottery ticket. He

has recently been drinking even more following the loss of his trusted Labrador,

Wotsit. On examination the most striking finding is his distended abdomen which you

suspect is due to ascites.

List 2 clinical signs you would look for to demonstrate ascites?

Shifting dullness

Fluid thrill

Abdominal distension

List 2 signs associated with ascites that you would look for?

Abdominal wall venous distension

Ankle oedema

Distension of the neck veins

Divarifacation of the recti

Your examination has clearly demonstrated that Ruddiger has ascities. An inquisitive

medical student attached to your firm asks you what ascites is. What would be your

response?

Ascites is the presence of free fluid within the peritoneal cavity

What investigation would you order to determine the nature of the fluid within his

abdomen?

Diagnostic paracentesis / ascites tap

List 2 things that this fluid would be analysed for?

104

Page 105: finals2009.wikispaces.com · Web viewPleural effusion which is secondary to hypoalbunimaemia Q4. What initial investigations would be implemented to confirm this diagnosis? Urine

Protein

Malignant cells

Infection

Blood / white cells

The fluid can either be a transudate or an exudate. What is the difference and list 2

causes for each?

Transudate = protein <25 g/L

Exudate = protein > 25 g/L

Transudate – cardiac failure, liver failure, cirrhosis, hypoproteinaemia

Exudate – Intra-abdominal malignancy, infection, pancreatitis

Investigations show the fluid to be a transudate. You suspect the ascites is due to

damage to Ruddiger’s liver from the excessive alcohol. List 4 things that your

management would include to relieve the ascites?

Bedrest

Fluid restriction (<1.5 L/day)

Low salt diet

Diuretic – spironolactone

Diuretic – furosemide

Therapeutic paracentesis / ascites tap

Alcohol avoidance advice

3.7 Ascites

By the end of Phase II students should be able to:

• detect ascites clinically

• initiate appropriate investigation having regard to the likely causes

• initiate management of hepatic ascites

105

Page 106: finals2009.wikispaces.com · Web viewPleural effusion which is secondary to hypoalbunimaemia Q4. What initial investigations would be implemented to confirm this diagnosis? Urine

3.9

A 26 year old medical student comes to you complaining of abdominal pain and

explosive diarrhoea that has been getting worse for the past 7 days. She has noticed

some blood mixed in with her stool and is now very concerned. She reports that she

has been travelling around Thailand for the previous 3 weeks.

a) What is the likely cause of her bloody diarrhoea?

Any of : Campylobacter, Shigella, Salmonella, E.coli (1)

b) Pancreatitis, inflammatory bowel disease, antibiotics, malabsorption and

thyrotoxicosis are all causes of chronic diarrhoea, but can you name 2 other bacteria,

protazoa and viruses that cause acute diarrhoea (½ for each)?

Bacteria Clostridium difficile

Mycobacterium tuberculosis

vibro choleae. (1)

Protazoa Entamoeba Histolytica

Giardia lamblia (1)

Viruses Rotavirus (Grp A)

Astrovirus

Norwalk-like virus

enteric adenovirus

calicivirus (1)

c) What investigation and treatment would you recommend ?

Stool culture +/- parasite screen (1)

Rehydration using isotonic oral fluids containing electrolytes and glucose (1).

(Intravenous fluid replacement may be indicated in patients with persistent vomiting)

Ciprofloxacin 500mg bd or Erythromycin 500mg qds for 1 week is indicated as her

diarrhoea is persistent.(1)

Anti diarrhoeal drugs should be avoided here to avoid prolonging the infection.

d) She returns 2 months later complaining of severe constipation and she has not had

106

Page 107: finals2009.wikispaces.com · Web viewPleural effusion which is secondary to hypoalbunimaemia Q4. What initial investigations would be implemented to confirm this diagnosis? Urine

her bowels open for 10 days. Give 3 different drugs used in the treatment of

constipation and briefly mention how they exert their effects: (½ each 3max)

Lactulose/Magnesium sulphate - Osmotic laxative (1)

Senna/Docusate sodium - Motility agent (1)

Ispaghula Husk/Fybogel - Bulk forming laxative (1)

Extended Question – The defecation reflex occurs in response to distension of the

rectal wall mediated by mechanoreceptors, can you describe the stages involved ?

Contraction of the rectum

Relaxation of the internal and external sphincter

An initial contraction of the external anal sphincter

Increased peristaltic activity in the sigmoid colon

Relaxation of the external anal sphincter

Expulsion of faeces

(27 extra marks if you remember all this)

3.9          Diarrhoea and constipation

By the end of Phase II students should be able to:

• appreciate likely diagnoses in patients with acute and chronic diarrhoea and how they may

be distinguished on clinical grounds

• initiate appropriate investigation

• assess the physiological effects of severe diarrhoea

• explain the importance of oral rehydration solutions

• distinguish the common causes of constipation on clinical grounds

107

Page 108: finals2009.wikispaces.com · Web viewPleural effusion which is secondary to hypoalbunimaemia Q4. What initial investigations would be implemented to confirm this diagnosis? Urine

• initiate appropriate investigations for constipation

• manage constipation 

108

Page 109: finals2009.wikispaces.com · Web viewPleural effusion which is secondary to hypoalbunimaemia Q4. What initial investigations would be implemented to confirm this diagnosis? Urine

3.10

A 15 year old girl newly diagnosed with Inflammatory bowel disease.

Q1. How might she have presented?

Diarrhoea with blood / mucus

RIF mass

Abdo pain

Weight Loss

Signs of anaemia

Systemic signs E.g Skin and Eyes

Fever

Q2. How would you investigate her (she presented with diarrhoea)?

FBC – Anaemia, white cell count, raised platelets

LFT’s – albumin, alk phos

U/E’s – potassium

Inflammatory markers – CRP, ESR

Anti-endomysial antibodies – (celiac)

Stool sample – infective cause

Endoscopy – histology and macroscopic appearance

Q3. On histology how would you differentiate between UC and Crohns?

UC

Confined to Mucosa

Crypt abscess

Cell displasia

Crohns

Skip lesions with multiple

biopsies

Transmural

Granulomas and giant cells

Macrophages, lymphocytes and

plasma cells

Q4. What are the macroscopic differences?

UC Crohns

109

Page 110: finals2009.wikispaces.com · Web viewPleural effusion which is secondary to hypoalbunimaemia Q4. What initial investigations would be implemented to confirm this diagnosis? Urine

Rectum to iliocaecal valve

(occasionally backwash ileitis)

Superficial ulceration

Pseudopoylpos

Mouth to Anus

Skip lesions

Rose thorn fissures

Cobblestone appearance

Fistulae and Abscesses

After 5 years she has had repeated exacerbations and presents to A&E with bloody

diarrhoea occurring fifteen times a day and lower abdominal pain, for the last 3 weeks

She is very distressed and feels she has no quality of life

Q5. Give your acute management?

ABC

IV fluids

Admit

IV steroids

NBM

AXR – ? abscess

+/- antibiotics

ASA e.g sulphasalazine

Analgesia

After 1 week she is showing signs of improvement but still does not feel that she is

able to cope at home.

Q6. Discuss her options (for each disease)

UC

Medically – aziothioprine

steroids

ASA’s

Surgically – remove affected area

o Panproctocolectomy

Crohnes

If localised – can operate

Otherwise – aziothioprine

steroids

ASA’s

110

Page 111: finals2009.wikispaces.com · Web viewPleural effusion which is secondary to hypoalbunimaemia Q4. What initial investigations would be implemented to confirm this diagnosis? Urine

Q7. She opts for a surgical approach to discuss the issues

UC

Positives Negatives

Reduced colon cancer risk Major Surgery – anaesthetic risk

Symptom resolution Stoma – possibly life long

Long term incontinence post reversal

Crohns

Positives Negatives

Possible reduction in symptoms Recurrence of Disease

Reduced chance of perforation Adhesions

Fistula formation

Q8. When wanting to assess disease control other than GI symptoms what else might

you question her about (systemically)

Eyes – Uveitis

o Irritis

o Episcleritis

Skin - Rashes – erythema nodosum, pyoderma gangrenosm

Joints- Ankylosing spondylitis

o Sacroilitis

Liver- PSC

Gall Stones

Q9. (Extended)

Her brother suffers from Ankylosing spondylitis. What genetic factor might they

share?

111

Page 112: finals2009.wikispaces.com · Web viewPleural effusion which is secondary to hypoalbunimaemia Q4. What initial investigations would be implemented to confirm this diagnosis? Urine

HLA B27

3.10 Inflammatory bowel disease

By the end of Phase II students should be able to:

• recognise the possibility of inflammatory bowel disease in patients presenting with lower

gastrointestinal symptoms

• initiate appropriate investigations in a patient with a suspected inflammatory bowel disease

• initiate appropriate management in a patient with inflammatory bowel disease

• recognise the differences in presentation between Crohn's disease and ulcerative colitis, and

how these relate to the underlying pathology

• recognise the possibility of systemic symptoms associated with inflammatory bowel disease

• explain to patients the nature of and the rationale for maintenance treatment for inflammatory

bowel disease

112

Page 113: finals2009.wikispaces.com · Web viewPleural effusion which is secondary to hypoalbunimaemia Q4. What initial investigations would be implemented to confirm this diagnosis? Urine

3.12

A 42 year old man presents to his GP with a 4 day history of bloody

diarrhoea. The blood is fresh in appearance. The patient has been

diagnosed as suffering from FAP since his early 20's. The

patient also complains of abdominal pains which subside once he goes

to the toilet. He is mildly short of breath and looks pale in

appearance.

Question 1a

What is FAP and explain why you are worried this patient has cancer

of the large bowel?

People with Familial Adenomatous Polyposis, a rate autosomal dominant

disorder, almost always develop cancer of the intestine by age 35.

It is characterised by thousands of polyps growing in the large

intestine. It is a pre-malignant condition.

Question 1b

Explain the breathlessness.

The patient is likely anemic due to chronic blood loss.

Question 2

What are the 4 layers of the large bowel and for that matter the rest

of the GI system?

Mucosa, Submucosa, Muscularis Propria/Externa, Serosa

Question 3

Name 1 other symptom you might expect the patient to complain about

if he indeed has cancer of the large bowel.

113

Page 114: finals2009.wikispaces.com · Web viewPleural effusion which is secondary to hypoalbunimaemia Q4. What initial investigations would be implemented to confirm this diagnosis? Urine

Unexplained weight loss

Question 4

What is the most common colon cancer cell type?

Adenocarcinoma (which accounts for 95% of cases).

Question 5

Which classification is used to stage cancer in the large bowel?

Briefly explain each stage.

Duke's A: invaded submucosa and muscle layer of the bowel but

confined to the wall.

Duke's B: breached the muscle layer and bowel wall but no involvement

of local lymph nodes.

Duke's C1: spread to immediately draining pericolic lymph nodes.

Duke's C2: Spready to higher mesenteric lymph nodes.

Duke's D: Distant visceral metastases.

Extended Question 6

Name 5 factors which increase a person's risk of developing Colo-rectal

cancer.

Any 5 of:

Smoking

Diet high in fats

Diet high in carbohydrates

Diet low in fibre

Suffering from Inflammatory bowel diseases (i.e. Crohn's, UC) as these

conditions are pre-malignant.

114

Page 115: finals2009.wikispaces.com · Web viewPleural effusion which is secondary to hypoalbunimaemia Q4. What initial investigations would be implemented to confirm this diagnosis? Urine

Family Hx of Bowel Cancer

Age. The risk of developing colorectal cancer increases with age.

Most cases occur in the 60s and 70s, while cases before age 50 are

uncommon unless a family history of early colon cancer is present.

HNPCC (Hereditary Non-Polyposis Colorectal Cancer)

Extended Question 7

Name 2 investigations you would like to see carried out to assess

the extent of disease in this patient's bowel?

Barium Enema

Colonoscopy +/- Biopsy

3.12 Bowel cancer

By the end of Phase II students should be able to:

• recognise the possibility of colonic cancer in patients with

disturbed bowel habit, rectal bleeding or anaemia

• initiate appropriate investigations to confirm the diagnosis and

extent of disease

• suggest a prognosis based on staging and other factors

• outline to patients the possible management strategies for bowel cancer

• outline to a patient the management of a colostomy

115

Page 116: finals2009.wikispaces.com · Web viewPleural effusion which is secondary to hypoalbunimaemia Q4. What initial investigations would be implemented to confirm this diagnosis? Urine

3.13

Peter, a 58 year old gentleman presents to accident and emergency with a 6 hour

history of acute abdominal pain. As the surgical house officer you are called.

On arrival in A+E Peter has a pulse of 100bpm, is pyrexial and in obvious distress.

Q1

As you are walking to A+E what are your top 4 differential diagnoses for acute

abdominal pain.

Perforation

Bowel Obstruction

Acute Pancreatitis

Dissecting Aortic Aneurysm

Mesenteric Vascular Obstruction

Acute appendicitis

Acute Cholecystitis

Kidney Stones

Diverticulitis

Peptic Ulcer

On taking a history you find that he has a 6 hour history of severe epigastric pain,

waking him from his sleep, which is associated with nausea and vomiting. There is a

history of gallstones and hypertension. Peter attributes his symptoms to food at his

daughters wedding the day before.

Q2

What initial investigations would you carry out and why (be specific, not just ‘blood

test’)?

FBC: Raised WCC (infection)

U+E’s: Dehydration (N+V), General kidney function (management)

CRP, ESR: inflammatory markers

116

Page 117: finals2009.wikispaces.com · Web viewPleural effusion which is secondary to hypoalbunimaemia Q4. What initial investigations would be implemented to confirm this diagnosis? Urine

LFT’s: Raised AST secondary to Gallstones, Albumin for severity of ?pancreatitis

Amylase: May be raised in pancreatitis

Glucose: Severity of ?Pancreatitis

Bone (Ca): Severity of ?Pancreatitis

ABG: Severity of ?Pancreatitis

Erect CXR – Perforation? Air under the diaphragm?

AXR – Kidney stones? Sentinal Loop present? Psoas shadow?

ECG

Q3

Apart from gallstones, give 4 other common causes of pancreatitis? If gallstones are

not the cause, what do you think is the most likely cause in this case?

Alcohol

Drugs

Trauma

ERCP

Autoimmune

GET SMASHED – causes such as scorpion sting not common in this country!

Alcohol and diuretics most likely cause – due to daughters wedding the night before,

at which he may have been drinking, and history of hypertension, which may be

treated with diuretics.

Q4

In pancreatitis you would expect an increase in amylase, name 3 other causes of an

increased amylase.

Upper GI tract perforation

Leaking AAA

Mesenteric Ischaemia

Acute cholecystitis

Renal Failure

117

Page 118: finals2009.wikispaces.com · Web viewPleural effusion which is secondary to hypoalbunimaemia Q4. What initial investigations would be implemented to confirm this diagnosis? Urine

Q5

What parameters would you use to assess the severity of an individual such as Peter

who is suffering from pancreatitis? And what is the name for this scoring system?

Modified Glasgow Criteria

PaO2 < 8 kPa (PANCREAS)

Age >55 years

Neutrophils >15x109/L

Ca < 2mmol/L

uRea >16mmol/L

Enzymes – LDH >600iu/L, AST >200iu/L

Albumin<32g/L

Sugar (glucose) > 10 mmol/L

Peter scores 4 on this scale and your consultant admits him of ITU.

Q6

In your further management what complications would you look for?

ARDS

Diabetes Mellitus

Pseudocysts

Pleural effusions

Organ failure

Pancreatic abscess

Ascites

Sepsis

Extended Q

118

Page 119: finals2009.wikispaces.com · Web viewPleural effusion which is secondary to hypoalbunimaemia Q4. What initial investigations would be implemented to confirm this diagnosis? Urine

What signs would you look for on abdominal examination in severe pancreatitis and

what are they called?

Cullens sign – Peri-umbilical bruising

Grey-turners sign – Bruising of the flanks

3.13        Acute abdominal pain

By the end of Phase II students should be able to:

• recognise localised and generalised peritonitis and begin to distinguish the common causes

• appreciate the causes of a ruptured viscus and how they may  be begin to be distinguished

• investigate, assess and initiate management of acute pancreatitis

• appreciate the common causes of pain in the right iliac fossa and how they may be

distinguished

• initiate appropriate investigations in a patient with upper abdominal pain

• recognise the possibility of bowel obstruction

• initiate appropriate investigations and initiate intravenous fluid replacement and appropriate

pain relief 

119

Page 120: finals2009.wikispaces.com · Web viewPleural effusion which is secondary to hypoalbunimaemia Q4. What initial investigations would be implemented to confirm this diagnosis? Urine

A 52-year-old man presents to A&E with a four-day history of severe epigastric pain

radiating to the back. The patient describes the pain as a sharp constant pain present

in the epigastric region radiating to the back in a band like fashion. The pain was made

worse on eating and drinking alcohol and relieved slightly by sitting forward. The

patient described associated symptoms of vomiting and nausea.

Question 1: What are the main differentials for the patient’s presentation?

Acute pancreatitis

Peptic ulcer

Acute cholecystitis

GI obstruction

Question 2: What investigations would you order?

Bloods – FBC, U&E, Amylase, LFT’s, and CRP

Abdominal x-ray

Ultrasound of gallbladder

Question 3: After investigation the patient’s serum amylase was found to be 3045u/mL,

LFT’s were normal, WCC was 11.5 and CRP was 105. Nothing abnormal was detected

on Ultrasound and on abdominal x-ray; there was no psoas shadow. Given these

results, what is the most probable cause of the patient’s presentation?

Acute pancreatitis

Question 4: What are the commonest causes of acute pancreatitis?

G – Gallstones

E – Ethanol

T – Trauma

S – Steroids

M – Mumps

A – Autoimmune – PAN

S – Scorpion venom

H – Hyperlipideamia/Hypercalcaemia/Hypothermia

120

Page 121: finals2009.wikispaces.com · Web viewPleural effusion which is secondary to hypoalbunimaemia Q4. What initial investigations would be implemented to confirm this diagnosis? Urine

E – ERCP

D – Drugs – azathioprine, asparaginase

Question 5: What common signs may be found on examination with a patient with

acute pancreatitis?

Tachycardia

Fever

Jaundice

Shock

Generalised abdominal tenderness

Periumbilical discolouration – Cullen’s sign – around the umbilicus

Grey Turners sign – at the flanks

Question 6:What are the criteria for predicting the severity of acute pancreatitis?

P – PaO2 < 8 kPa

A – Age > 55

N – Neutrophils > 15*10power9/L

C – Calcium < 2 mmol/L

R – Renal function – Urea > 10mmol/L

E – Enzymes – LDH > 600iu/L and AST > 200iu/L

A – Albumin < 32 g/L

S – Sugar – Glucose > 10 mmol/L

Question 7: What would be the management plan for this patient?

Nil by mouth

Set up IVI and give plasma expanders and 0.9% saline until vital signs are

satisfactory and urine flow > 30mL/L. Check weight daily and insert a catheter.

Analgesia: Pethidine 75-100mg/4h IM, or morphine (may cause the sphincter of oddi

to constrict more)

Hourly pulse, BP and urine flow

Daily FBC, U&E, glucose, amylase, blood gas.

121

Page 122: finals2009.wikispaces.com · Web viewPleural effusion which is secondary to hypoalbunimaemia Q4. What initial investigations would be implemented to confirm this diagnosis? Urine

If worsening, take to ITU.

Question 8: What are some of the complications of acute pancreatitis?

Early:

Disseminated Intravascular Coagulation

Renal failure

Respiratory failure

Haemorrhage

Thrombosis – may occur in the splenic and gastroduodenal arteries, or in the

colic branches of the superior mesenteric artery, causing

bowel necrosis.

Late: > 1 week – P

Pancreatic necrosis

Pseudocyst

Objective: acute pancreatitis

122

Page 123: finals2009.wikispaces.com · Web viewPleural effusion which is secondary to hypoalbunimaemia Q4. What initial investigations would be implemented to confirm this diagnosis? Urine

3.14

Lindsay, a 23 year old previously healthy woman presents to her general practitioner

with a 3 month history of generalised abdominal pain.

On looking at her previous attendance and notes she was last seen for back pain 5

months ago.

Q1

List 6 further questions you would want to ask about her presenting complaint.

Change in bowel habits

Change in consistency

PR bleeding/mucus

Diet/relationship to food

Relieving/aggravating factors

Bloating/flatus

Tenesmus

PV bleeding/LMP/cyclical/discharge

On further questioning, Lindsay reports a change in bowel habits, primarily

constipation.

Q2

Give 4 causes of constipation which may be applicable in this case?

Decreased dietary fibre

Hypothyroidism

Hyperparathyroidism

Anxiety/depression

IBS

Drugs (codeine based – may have been used to treat previous back pain)

Other common causes are less likely due to the young age and previous good health.

123

Page 124: finals2009.wikispaces.com · Web viewPleural effusion which is secondary to hypoalbunimaemia Q4. What initial investigations would be implemented to confirm this diagnosis? Urine

On further questioning, she reports an associated bloating feeling. She also believes

she leads a healthy lifestyle.

On examination of Lindsay there is nothing remarkable found.

Q3

What investigations would you do, if any?

There are no real indications for any investigations, however if other symptoms were

present to indicate hypothyroidism or parathyroidism then the following would be

indicated:

Thyroid function tests

Plasma Ca

All tests come back normal.

Q4

Give steps to your management of Lindsay.

Description of IBS

Reassurance

Patient own management eg increase dietary fibre, increase exercise, decrease

stress

Medical therapy – osmotic laxative eg lactulose, anti spasmodic eg Mebeverine

Q5

Name one complication of this disorder?

Colonic diverticulae

124

Page 125: finals2009.wikispaces.com · Web viewPleural effusion which is secondary to hypoalbunimaemia Q4. What initial investigations would be implemented to confirm this diagnosis? Urine

3.14        Irritable bowel syndrome

By the end of Phase II students should be able to:

• recognise the presentations of irritable bowel syndrome

• appreciate other common causes of chronic abdominal pain and how they may be

distinguished

• explain to a patient the nature of irritable bowel syndrome and its relationship to precipitants

such as stress.

125

Page 126: finals2009.wikispaces.com · Web viewPleural effusion which is secondary to hypoalbunimaemia Q4. What initial investigations would be implemented to confirm this diagnosis? Urine

3.17

MP is a 35 year old man who was diagnosed with Type 1 diabetes mellitus at the age of

14 after presenting to his GP with polyuria, polydipsia and weight loss. Urine dipstick

demonstrated the presence of glucose and ketones, which is not diagnostic of

diabetes mellitus, but indicates the need for further investigation.

Name three blood tests used to diagnose diabetes mellitus and the values that indicate

presence of the disease.

Random blood glucose > 11.1 mmol/L

Fasting venous plasma glucose ≥ 7.0 mmol/L

2 hour oral glucose tolerance test > 11.1 mmol/L

NOTE: These values refer to venous plasma levels, rather then that of venous whole

blood.

Diabetes mellitus increases the risk of cardiovascular disease. Which other co-existent

cardiovascular risk factors can be screened / assessed in a diabetic patient? (Name 3)

Blood pressure

Cholesterol

Smoking history

Diabetic control (BMs, HbA1c)

When Mr P was initially diagnosed he was educated on his condition, advised about the

importance of diet, referred to a dietician, and started on insulin.

You are concerned that Mr P is non-compliant. You are worried that he is not following

your dietary advice with a large alcohol intake, and is not regularly taking his insulin.

Which investigation can be used to determine a patient’s glycaemic control? What is

the target value? What does this value indicate?

HbA1c < 7 %

Glycosylated haemoglobin levels relate to the mean glucose level over the previous

8 weeks (i.e. RBC half life)

Later that week Mr N was found at home drowsy and a little confused following a late

night where he consumed a lot of alcohol. Earlier that day he had complained of feeling

shaky, sweating, pallor, heart pounding, and irritability. What is the likely diagnosis?

126

Page 127: finals2009.wikispaces.com · Web viewPleural effusion which is secondary to hypoalbunimaemia Q4. What initial investigations would be implemented to confirm this diagnosis? Urine

Hypoglycaemia

Name two other ways that this condition may present.

Coma

Seizures

Temporary paralysis

You measure his blood glucose as 2.5. What management would you like to initiate?

Oral sugar e.g. fruit juice, sugar lump, followed by long acting starch e.g. toast

How would your management differ if he was unresponsive?

Call an ambulance

Glucose 25-50g IV or glucagon 0.5-1mg SC/IM – should see some improvement

following this.

Repeat after 20 minutes if no improvement

If prolonged hypoglycaemia, may want to give dexamethasone 4mg/4hr IV to combat

cerebral oedema.

3.17 Insulin dependent diabetes

By the end of Phase II students should be able to:

• identify patients with insulin-dependent diabetes

• screen patients for co-existent cardiovascular risk factors

• screen for diabetes-related complications

• initiate management of a patient with IDDM, including the appropriate use of long- and

short-acting insulins

• determine a patient's degree of metabolic control

• recognise diabetic ketoacidosis

• participate in the management of diabetic ketoacidosis

• recognise and manage hypoglycaemia

• outline to patients the dietary principles of the management of IDDM

127

Page 128: finals2009.wikispaces.com · Web viewPleural effusion which is secondary to hypoalbunimaemia Q4. What initial investigations would be implemented to confirm this diagnosis? Urine

3.19

A long standing insulin dependent diabetic Mr B, presents at your clinic having

recently been diagnosed with hypertension. You realize that the combination of long

standing diabetes and hypertension puts Mr B at greater risk of cardiovascular

complications; hence you decide to examine his cardiovascular system.

Name 6 signs or symptoms you might expect to find on Mr B?

Intermittent claudication

angina pain

reduced exercise tolerance

diminished peripheral pulses

reduced capillary refill, cold peripheries

ischaemic skin changes

poor healing and ulcers.

How would you treat Mr B’s hypertension in order of preventing the development

of cardiovascular complications?

Treatment is dependent upon the BP;

<145/80 and no microalbuminuria check BP every 6months

>140/80 an <160/100 and no microalbuminuria, treat with antihypertensives; ACE

inhibitor, beta-blocker or a thiazide

>140/80 with microalbuminuria should be treated with either an ACE inhibitor or

angiotensin 2 antagonist, with a target BP of 135/75

In addition to examining Mr B’s cardiovascular system you decide to perform

fundoscopy to detect any retinal changes of diabetic retinopathy.

Name the 4 stages of diabetic retinopathy with the key features of each?

Background, diabetic maculopathy, pre-proliferative and proliferative

128

Page 129: finals2009.wikispaces.com · Web viewPleural effusion which is secondary to hypoalbunimaemia Q4. What initial investigations would be implemented to confirm this diagnosis? Urine

Explain why these retinal changes develop in diabetic patients?

These changes are the result of microvascular disease, with an increase in the

thickness of the basement membrane, and increased permeability of the retinal

capillaries. The result is the formation of microaneurysms that haemorrhage and later

lead to new vessel development.

Diabetic retinopathy is one of the complications that can develop in a diabetic patient.

List 4 other complications of diabetes and whether they are microvascular or

macrovascular changes?

Neuropathy – microvascular

Nephropathy – microvascular

Peripheral vascular disease –macrovascular

Ischaemic heart disease –macrovascular

Stroke - macrovascular

What are the pathological differences between microvascular and macrovascular

changes?

Microvascular disease involves the small vessels in the retina, glomerulus and vasa

nervorum, whilst macrovascular involves the larger vessels.

In microvascular disease there is progressive narrowing of the vascular lumina due to

hyperglycaemia-induced microvascular hypertension, basement membrane thickening

and increased vascular permeability.

Macrovascular disease involves atherogenesis; the infiltration of lipid in the intimal

layer resulting in plaque formation and occlusion of the vessel.

Finally, you decide to manage Mr B with a multidisciplinary approach.

Name 4 health professionals/ agencies that could be involved in his care?

Endocrinologists/ GP/ diabetic nurse/ chiropodist/ophthalmologist/renal

physician/orthopaedic surgeon

3.19 Long term complications

By the end of Phase II students should be able to:

• recognise the long term cardiovascular complications of diabetes

129

Page 130: finals2009.wikispaces.com · Web viewPleural effusion which is secondary to hypoalbunimaemia Q4. What initial investigations would be implemented to confirm this diagnosis? Urine

• manage hypertension appropriately in diabetics

• recognise renal disease in diabetics and refer appropriately

• recognise diabetic neuropathy

• recognise autonomic neuropathy

• recognise the potential importance of skin lesions in diabetics

• recognise diabetic retinopathy and refer appropriately

• describe the mechanisms underlying these complications and the potential for their

prevention

130

Page 131: finals2009.wikispaces.com · Web viewPleural effusion which is secondary to hypoalbunimaemia Q4. What initial investigations would be implemented to confirm this diagnosis? Urine

3.23

Alice is a 66-year-old housewife who complains of weakness and episodes of fainting,

which makes her spend most day in bed. She also complains of nausea, vomiting and

abdominal pain. Alice also mentions in passing that her friends have been commenting

on her tan, even though she has not been on holiday.

a) what could be the diagnosis? (name 2)

Addison’s disease

Congenital adrenal hyperplasia

a) give two common causes for this condition?

Autoimmune

Tuberculosis

HIV/AIDS

Metastatic carcinoma

Bilateral adrenalectomy

b) what is the name of the test that will confirm your diagnosis and give a brief

explanation of the procedure?

Short ACTH Stimulation test (Synacthen Test)

Do plasma cortisol, inject tetracosactide IM (exogenous ACTH), plasma cortisol

after 30 minutes.

Cortisol remains low in Addison’s disease, but if above 600nmol/l then exclude

Addison’s disease

c) name two drugs that may be given to this patient?

Glucocorticoid replacement with either hydrocortisone or prednisolone

Mineralocorticoid replacement with fludrocortisone

131

Page 132: finals2009.wikispaces.com · Web viewPleural effusion which is secondary to hypoalbunimaemia Q4. What initial investigations would be implemented to confirm this diagnosis? Urine

Alice was given treatment. 2 years later she complained of being overweight, easy

bruising, excess hair growth and purple marks on her stomach.

d) what is the name of this condition?

Cushing’s syndrome

EXTENDED

Why has Alice got a ‘tanned’ appearance to her skin?

Lack of adrenal hormones causes feedback on the hypothalamo-pituitary axis to

increase CRH and therefore ACTH production. ACTH contains MSH-like

sequences that cause pigmentation due to stimulation of melanocytes.

OBJECTIVES

3.23        Adrenal gland hormones

By the end of Phase II  students should be able to:

• recognise the symptoms and signs of Addison's Disease

• confirm the diagnosis of Addison's Disease

• initiate the immediate management of Addisonian crisis

• recognise symptoms and signs of Cushing's Syndrome

• confirm the diagnosis of Cushing's Syndrome

• participate in the long term management of Addison's Disease and Cushing's Syndrome  

132

Page 133: finals2009.wikispaces.com · Web viewPleural effusion which is secondary to hypoalbunimaemia Q4. What initial investigations would be implemented to confirm this diagnosis? Urine

3.24

Dr Patel thrusts another rather large set of notes and asks you to see Mr PA in the side

room. On glancing at the GP letter you read:

‘This 45yr old gentleman has been complaining of long term headaches and visual field

disturbances suggestive of a pituitary lesion. The results of his glucose tolerance tests

(GTT) are consistent with acromegaly. I would be grateful if you could review his care’

Which type of visual field loss would you expect to find in this patient? (1 mark)

Bi-temporal hemianopia (tunnel vision)

List four other clinical features you would expect to find during a physical examination

on this patient other then those already mentioned. (2 marks)

Briefly describe how you would perform a glucose tolerance test? (2 marks)

Overnight fasting

Oral glucose 75mg taken at T 0 mins

Monitor BM and growth hormone at timed intervals, 0>30>60>90>120 mins

Absence of GH suppression suggests acromegaly.

Suggest two non-biochemical investigations you may carry out to assess the severity

of Mr PA’s main condition. (2 marks)

MRI brain: assess size and spread of pituitary lesion

X-ray feet and hands: looking for osteophytes and increased joint space

( Mr PA condition is a pit adenoma causing excessive levels of GHRH and thus GH

133

Prognathism, greasy skin, large tongue,spade like hands, features relating to carpel tunnel syndrome

prominent eyebrow ridgeincreased interdental space (diff from Prognathism)husky voice

enlarged feet

Page 134: finals2009.wikispaces.com · Web viewPleural effusion which is secondary to hypoalbunimaemia Q4. What initial investigations would be implemented to confirm this diagnosis? Urine

causing acromegaly. Carpel tunnel is not his main problem thus nerve conduction

tests are not investigations you would order yet. )

Dr Patel, being impressed with your answers so far, asks how you would treat this

patient with acromegaly. (Assume he is otherwise fit and well). (3 marks)

Surgery (hypophysectomy) is gold standard with adjunct radiotherapy to shrink the

lesion. (If unfit for surgery then somatostatin analogues (octeotide) and dopamine

agonists (bromocriptine) are effective)

Regular pituitary function tests are essential.

Extended question

Describe the clinical features you would expect to find if Mr PA’s condition was left

untreated (E3 marks)

Acromegaly > Hypopituitarism.

Hypothyroidism features

Androgen deficiency (loss libido, hypogonadism, loss of muscle bulk)

Corticotrophin deficiency (postural HT, weight loss, low sodium)

3.24 Pituitary gland hormones

By the end of Phase II students should be able to:

• recognise the circumstances when hypopituitarism may occur

• recognise the possibility of hypopituitarism in patients with 'non-specific' symptoms

• initiate investigations for posterior pituitary function in patients with polyuria

• initiate investigations of the cause of hypopituitarism

• recognise acromegaly

• initiate investigations for acromegaly

• outline to patients the possible treatments for acromegaly

134

Page 135: finals2009.wikispaces.com · Web viewPleural effusion which is secondary to hypoalbunimaemia Q4. What initial investigations would be implemented to confirm this diagnosis? Urine

1) A)-Mr A is a 49-year-old widower and has just joined your GP’s practice. The practice

nurse has referred him because during his enrolment check-up, a BP of 173/90 was

recorded. You repeat his BP and it is 176/101.

What is the difference between essential and secondary hypertension? (1 mark)

Essential hypertension has no specific underlying cause, lifestyle plays a large part,

and secondary hypertension is the result of a range of pathological processes.

B)-Give 4 possible causes of secondary hypertension: (4 marks)

(Any 4 of below)

Renal parenchymal disease eg chronic pyelonephritis, glomerulonephritis, polycystic

disease

Renal artery disease eg atherosclerosis, fibromuscular dysplasia, vasculitis

Obstructive uropathy eg hydronephrosis 2 to stone/tumour

Congenital eg coarctation of the aorta

Drugs eg combined oral contraceptive pill, NSAID’s

Endocrine eg phaeochromocytoma, hyperaldosteronism, Cushing’s syndrome,

acromegaly

C)-At this first consultation, you are struck by Mr A’s large facial features including his

jaw, nose, brow and large hands. You suspect that Mr A maybe suffering from

acromegaly. Name 4 other signs or symptoms of acromegaly other than the ones

mentioned above: (4 marks)

Interdental separation, headaches, excessive sweating, visual deterioration,

tiredness, wt gain, amenorrhoea/oligomenorrhoea, galactorrhoea, impotence, poor

libido, deep voice, hirsutism, goitre, breathlessness, tingling in hands/carpel tunnel

syndrome, polyuria/polydipsia, muscular weakness, joint pains, thick greasy skin,

tight rings.

D)-Describe one of the aetiologies for acromegaly: (1 mark)

Benign pituitary tumour secreting GH (commonest)

Pituitary carcinoma

Carcinoid tumours that secrete hypothalamic GH-releasing hormone.

135

Page 136: finals2009.wikispaces.com · Web viewPleural effusion which is secondary to hypoalbunimaemia Q4. What initial investigations would be implemented to confirm this diagnosis? Urine

E)-In the history Mr A went on to confirm that he had been experiencing headaches and

had noticed a change in his appearance but put it down to his age. In view of his

possible diagnosis, name 3 investigations would you like to do and why? (6 marks)

(3 of any, 1 mark for investigation, 1 mark for reason)

GH levels – excessive circulating levels of GH, although this is non-diagnostic on its

own because transient raised levels are normal during stress

IGF-1 levels – almost always raised in acromegaly as a result of increased GH levels

Glucose tolerance test – GH is measured during a GTT, in healthy individuals, GH is

undetectable during the test, because hyperglycaemia inhibits GH secretion.

MRI scan of pituitary – pituitary neoplasm is commonly the cause

Prolactin levels – frequently, the adenoma secretes both GH and prolactin

Assessment of visual fields – bitemporal hemianopia caused by pressure on the

optic chiasm by the tumour.

CXR and/or ECG – left ventricular hypertrophy secondary to hypertension

F)-A diagnosis of acromegaly is eventually made. What are the risks to Mr A if his

condition goes untreated? (2 marks)

Any hypertension related illness, eg coronary heart disease, heart failure,

cerebrovascular disease, PVD

Increased risk of neoplasia, especially large bowel tumours

G)-Discuss the treatment options available? (4 marks)

(Any 2; 1 mark for treatment, 1 mark for a relevant point)

Surgery – Trans-sphenoidal route is usually used, 90% of microadenomas are cured.

Complications include hypopituitarism, meningitis, intra-operative bleeding and death

Radiotherapy- Often used if attempts at surgery do not reduce GH levels.

Hypopituitarism may occur.

Medical therapies – Most effective is Octreotide, a somatostatin analogue, which

inhibits GH secretion in 60%. Bromocriptine, dopamine agonist reduces GH release,

less effective

Extended Question

136

Page 137: finals2009.wikispaces.com · Web viewPleural effusion which is secondary to hypoalbunimaemia Q4. What initial investigations would be implemented to confirm this diagnosis? Urine

H)-Explain the mode of secretion and physiological action of GH (4 marks)

GH is secreted by the anterior pituitary

GH acts directly on tissues, but most effects are mediated via IGF-1 (insulin-like

growth factor), produced mainly from the liver after stimulation from GH.

Stimulates cell growth and replication by accelerating the rate of protein synthesis

Increase uptake of amino acids, especially after a meal

Virtually every tissue responds, skeletal muscle and chrondrocytes especially

Counter regulatory to insulin, i.e., increases blood glucose via stimulating hepatic

gluconeogenesis and inhibiting glucose uptake by muscle

Secretion of GH occurs in bursts, in response to dual control by GHRH and GHRIH;

it is stimulated by low blood glucose and high blood amino acids, and during REM

sleep.

Objective: pituitary gland abnormalities

137

Page 138: finals2009.wikispaces.com · Web viewPleural effusion which is secondary to hypoalbunimaemia Q4. What initial investigations would be implemented to confirm this diagnosis? Urine

3.25

John is 18-years-old and is walking through the park when he sees some school

friends playing football. They ask if John would like to join in and hesitantly, because

he hasn’t done any exercise for years, he agrees to make up the numbers. Towards the

end of the game John is very sweaty, his hands are shaking and collapses on the floor.

At hospital John is diagnosed as being in a hypoglycaemic coma.

a) define hypoglycaemia in relation to plasma glucose levels?

<2.5mmol/l

b) give five causes of hypoglycaemia?(any 5)

Unexpected or unusual exercise

Alcohol

Insulin/ oral diabetes drugs overdose

Malabsorption

Addison’s disease

Hypopituitarism

Congenital adrenal hyperplasia

Liver failure

Alcohol abuse

Hypothermia

Insulinoma and other metastatic disease

c) what other signs and symptoms may his friends have noticed?

Sweating

Hunger

Drowsiness

Personality change

Fits

Anxiety

Confusion

Headache

138

Page 139: finals2009.wikispaces.com · Web viewPleural effusion which is secondary to hypoalbunimaemia Q4. What initial investigations would be implemented to confirm this diagnosis? Urine

Nausea

Tiredness

d) how would you treat such a patient?

ABC

50-100ml 50% dextrose IV fast followed by 0.9% saline flush as dextrose harms

veins

Glucagon 1mg IM if no venous access

EXTENDED

What is Whipple’s triad and which condition does it diagnose?

Diagnoses insulinomas by

symptoms are associated with fasting or exercise

hypoglycaemia is confirmed during these episodes

glucose relieves the symptoms

3.25        Hypoglycaemia

By the end of Phase II  students should be able to:

• recognise the possibility of hypoglycaemia and the circumstances under which it may occur

• initiate appropriate initial investigations

• initiate immediate management of hypoglycaemia 

139

Page 140: finals2009.wikispaces.com · Web viewPleural effusion which is secondary to hypoalbunimaemia Q4. What initial investigations would be implemented to confirm this diagnosis? Urine

A 21 year old female presents to A&E with a GCS of 9, abdominal pain and vomiting.

She is hyperventilating. She appears quite thin. Her friend tells you she has been

drinking heavily all evening but mostly soft drinks with little alcohol. The nurse calls

you over to show you she has an insulin pen in her jacket. You ask for a blood glucose

measurement which comes back at 26mmol/L.

1. Give the most likely cause of her symptoms [1 mark]

Diabetic ketoacidosis (DKA)

2. List 3 factors which may precipitate this condition [3 marks]

Infection

Poor compliance

Recent diagnosis

3. You decide to investigate further. List 5 investigations that would be suitable in

the acute phase. (be specific, don’t just put ‘blood test’)

[5 marks]

FBC

HbA1c

U+E

Glucose

Blood gases

Serum amylase

CXR

Ketones in urine or blood

4. Give 3 interventions to manage this patient [3 marks]

Insulin Sliding Scale

Fluid replacement

Potassium supplementation

Nil by mouth

Treat precipitating cause if any

140

Page 141: finals2009.wikispaces.com · Web viewPleural effusion which is secondary to hypoalbunimaemia Q4. What initial investigations would be implemented to confirm this diagnosis? Urine

5. Explain why the Type 1 diabetic with severe hyperglycaemia will enter a ketotic

state while the Type 2 diabetic will tend towards the hyperosmolar nonketotic

state. [5 marks]

Low insulin prevents the uptake of glucose into tissues.

This prevents cells using glucose as an energy source. Alternatives are required.

Energy is gained by producing fatty acids from lipolysis.

These are converted into ketones by the liver. These are present in the circulation

and produce a ketotic state

In Type 2 diabetics there is plenty of insulin present but there is resistance to it’s

action.

The blood glucose produces a hyper osmolar circulation which produces its clinical

effect due to osmotic pressures leading to cellular dehydration

Ojective: HONK and DKA

141

Page 142: finals2009.wikispaces.com · Web viewPleural effusion which is secondary to hypoalbunimaemia Q4. What initial investigations would be implemented to confirm this diagnosis? Urine

3.26

Sidney is a 55 year old bus driver. He presents to A+E with non-specific symptoms of

headache, confusion and drowsiness.

Bloods results: serum Na levels of 102mmol/l (130-145mmol/l)

Q1: Name 3 medical disorders causing ↓Na:

Hypothyroidism

Addisons’ disease

SIADH

↑ extracellular volume (heart failure, liver failure)

Q2: Treatment generally involves correcting the underlying cause. But how would you

manage this patient acutely?

Mild: Fluid restriction.

Moderate: IV saline and frusemide.

Severe (emergency): IV 0.9% saline or hypertonic 1.8% saline. Gradually ↑Na. can

combine with frusemide. Caution in heart failure.

Q3: What other investigations would you consider, to establish the cause?

Hypothyroidism (TFTs)

Addisons’ disease (blood plasma, synacthen test)

SIADH (plasma osmolality, urine osmolality, U&Es)

heart failure, liver failure ( BNP level, CXR, ECG, LFTs)

142

Page 143: finals2009.wikispaces.com · Web viewPleural effusion which is secondary to hypoalbunimaemia Q4. What initial investigations would be implemented to confirm this diagnosis? Urine

Deepak presents to his GP with a hx of polydipsia and polyuria.

Investigations show he has a ↑ serum Na level of 158mmol/l.

Q1: Name 4 causes of ↑Na:

Diabetes insipidus,

↓ Water intake,

↑ Fluid loss with less Na loss (vomiting, diarrhoea, burns)

Incorrect IV fluid replacement (excessive saline)

Primary aldosteronism

Renal disease (ATN)

Q2: how would you initially manage this (relating management to both hypovolaemia

and hypervolaemia):

Hypovolaemia: Give water orally, or 5% dextrose IV slowly (guided by urine output,

and plasma Na. Avoid hypotonic solutions!

Hypervolaemia: diuretics to remove Na and ↑ fluid loss.

Q3: He was later found to have Diabetes insipidus, what test confirms the diagnosis of

this condition? How is it performed, and how would you interpret the results?

Water deprivations

o Deprivations of water for ~9hours

o Injection/intranasal administration of ADH (Desmopressin)

o Measure urine osmolarity after hourly intervals (also can take serial plasma

samples)

Central DI: urine osmolarity increases

Nephrogenic DI: urine osmolarity changes very little

143

Page 144: finals2009.wikispaces.com · Web viewPleural effusion which is secondary to hypoalbunimaemia Q4. What initial investigations would be implemented to confirm this diagnosis? Urine

Rose has recently been diagnosed with type I diabetes mellitus. She presents to A+E

with signs and symptoms consistent with DKA.

Blood results show that her serum potassium is 6.7mmol/l.

Q1: Why do you get ↑K in DKA?

Metabolic Acidosis in DKA, causes redistribution of electrolytes to help minimise

acidosis. K:H exchanger (↑ extracellular K, ↑ intracellular H)

Q2: List 4 other causes of ↑K levels.

Renal failure: main cause, due to oliguria

Drugs: K sparing diuretic (main), ACEi, β-blockers, excess K therapy, massive blood

transfusion.

Redistribution to ECF: Acidosis-(K:H exchanger), cell lysis (crush injury, haemolysis,

strenuous exercise, rhabdomyolysis, burns).

Endocrine: hypoaldosteronism (Addison’s disease)

Artefact: haemolysis of RBC’s in collecting bottle, delayed analysis-leakage of K.

Q3: What ECG changes would you expect to see with increasing K levels?

Tall/tented T waves, small/flat P waves, wide QRS complex, VF.

Q4: List 5 options in treating K levels >6.5mmol/l?

10ml calcium gluconate. This provides cardio-protection, doesn’t lower K levels!!

Insulin + glucose: moves K into cells, via Na:K:

Nebulised salbutamol: moves K into cells.

Consider Calcium resonium PO or PR to bind K in the gut, SE: constipation.

Sodium bicarbonate: corrects acidosis.

High dose frusemide

Haemodialysis:

144

Page 145: finals2009.wikispaces.com · Web viewPleural effusion which is secondary to hypoalbunimaemia Q4. What initial investigations would be implemented to confirm this diagnosis? Urine

Jaswinder has a past hx of hypertension, controlled by use of a diuretic. On routine

bloods his serum potassium was 2.4mmol/l.

Q1: name some other causes of low serum K?

Drugs: diuretics (main cause), β-adrenergic stimulation (salbutamol)

Mineralcorticoid excess: Conn’s syndrome, Cushing’s syndrome, corticosteroids,

ectopic ACTH, hyperaldosteronism.

Redistribution: alkalosis (K:H exchanger), excessive insulin.

Vomiting (pyloric stenosis, bulimia nervosa), Diarrhoea

Q2: What ECG changes are associated with ↓ K?

Prolonged P-R interval, small or inverted T waves, prominent U waves (after T

wave), depressed ST segment.

Q3: How would you manage K levels <2.5mmol/l?

IV KCl not more than 20mmol/h (never give a fast bolus, peripheral vein damage)

3.26 Abnormalities of sodium and potassium

By the end of Phase II students should be able to:

• initiate investigation of hypo and hyper natraemia and hypo and hyper kalaemia

• initiate management of these conditions

145

Page 146: finals2009.wikispaces.com · Web viewPleural effusion which is secondary to hypoalbunimaemia Q4. What initial investigations would be implemented to confirm this diagnosis? Urine

4.3

You are a doctor working in an Emergency Department, you are called to see Miss HG

a 15 year old girl who has fallen from her horse. She has been triaged and sent for

investigations.

Question 1:

What is this investigation, what is it of and what does it show? [3]

X –Ray, Clavicle, Fracture of distal 1/3.

Question 2:

When presenting an X ray what are the important features? [4]

Patients name, age, penetration,type of X ray, eg PA view of shoulder

Question 3:

How may the injury in question 1 be managed? [3]

Pain relief, broad sling or collar and cuff, review in around 3 weeks. Will not usually

require any surgery.

While you are seeing Miss HG she complains of an odd feeling in her wrist, you

examine it.

Question 4:

What features are you looking for on examination of the wrist? [4]

146

Page 147: finals2009.wikispaces.com · Web viewPleural effusion which is secondary to hypoalbunimaemia Q4. What initial investigations would be implemented to confirm this diagnosis? Urine

Neurovascular compromise (wiggle your fingers, can you fell me touching this, distal

pulses). Any painful areas, deformity/swelling, open fractures, bruising.

Extended Question:

What are the stages of bone healing? [4]

Haematoma –Granulation Tissue – Callus Formation – Bony consolidation -

Remodelling -

Extra notes:

Other Fractures You May see:

Colles Fracture: Forearm fractures account for 55% of childrens fractures. May result in

dinner fork deformity.

147

Page 148: finals2009.wikispaces.com · Web viewPleural effusion which is secondary to hypoalbunimaemia Q4. What initial investigations would be implemented to confirm this diagnosis? Urine

Scaphoid fracture: If suspected (e.g. pain in anatomical snuff box) but not visible on initial X

ray then put in plaster and re Xray after 2 weeks

Ankle Fracture: High impact, will always require treatment, from plaster of paris backslab to

internal fixation.

148

Page 149: finals2009.wikispaces.com · Web viewPleural effusion which is secondary to hypoalbunimaemia Q4. What initial investigations would be implemented to confirm this diagnosis? Urine

Anterior Shoulder Dislocation: Trauma, pain, loss of movement and shoulder contour.

Objective 4.3 Investigations

By the end of Phase II students should be able to:

• recognise the radiological features of fractures in general and the specific features

of the commoner fractures seeking advice where necessary

• recognise the radiological features of subluxation and dislocation

149

Page 150: finals2009.wikispaces.com · Web viewPleural effusion which is secondary to hypoalbunimaemia Q4. What initial investigations would be implemented to confirm this diagnosis? Urine

4.4

1. A 25 year old man trips on a tree branch as he walks down a steep incline and falls

awkwardly. He immediately experiences severe pain in his left leg and is unable to

weight bear on standing.

a. State 4 clinical features of a fracture

Pain

Loss of function

Tenderness

Deformity

Swelling

Crepitus (grating of the broken ends)

b. You check his airway, breathing and circulation and find that he is not

shocked. You find that there are no abrasions to the skin overlying the

fracture. What would be your first 4 steps in the immediate first aid care of

this fracture?

Step 1 Immobilise the fracture

Step 2 Remove tight clothing such as boots, or anything that may

impede circulation

Step 3 Check circulation, sensation and motion beyond fracture site

Step 4 Give pain relief

c. What is the main investigation of choice?

Plain radiograph left leg

d. What 2 features are you looking for on this investigation of choice?

Alignment (displacement, angulation, rotation)

Bone pathology

( If open wound, look for trapped air and foreign body)

e. You find that there is a closed transverse fracture to the lower third of this

man’s left fibula. It is poorly aligned. What should be done to manage this

fracture?

Reduction (under anaesthesia)

Fixation

Immobilisation

150

Page 151: finals2009.wikispaces.com · Web viewPleural effusion which is secondary to hypoalbunimaemia Q4. What initial investigations would be implemented to confirm this diagnosis? Urine

f. ‘Fracture disease’ follows immobilization – what 3 clinical features are

associated with this condition?

Muscle atrophy

Osteoporosis

Stiff joints

g. Post-operatively this man is doing well and is about to be discharged home

when he experiences pain in his left calf. What are you concerned this may be

a symptom of?

Compartment syndrome

h. And signs would you look for to ascertain this diagnosis ? Name 3.

(3 of the following)

Redness

Skin mottling

Blisters

Swelling

Pain on passive muscle stretching

i. What is the pathological process behind this condition? (5 points) and what

is the surgical management for this condition?

Vicious cycle of pressure (from limbs own dead weight when immobile)

vascular occlusion hypoxia necrosis increased pressure

Surgical management: Fasciotomy

j. He recovered and was later discharged home. This very very unlucky man

suddenly starts to experience breathlessness 10 days after his accident.

What is ur differentials? (name 2)

Fat embolism

Pulmonary embolism

k. choose one of the above differentials you have mentioned, what other signs

might you be looking for? Name 3.

For fat embolism:

Dyspnoea

Confusion

151

Page 152: finals2009.wikispaces.com · Web viewPleural effusion which is secondary to hypoalbunimaemia Q4. What initial investigations would be implemented to confirm this diagnosis? Urine

seizures

Coma

pyrexia

Hypoxia

For pulmonary embolism:

Dypnoea

Tachycardia

cyanosis

Raised JVP

Pleural rub

hamoptysis

l. Thankfully, this man makes a full recovery and on follow up his check plain

radiograph shows a well healed fracture. What are the pathological steps that

lead to bone healing? 5 points

Haematoma formation (1) vascular granulation tissue subperiosteal

osteoblast stimulation bone matrix endochondral ossification

deformable woven bone (callus) (1) Replacement by lamellar bone (1)

Remodeling (1) Fracture union (trabecular) (1)

m. What 5 factors may delay or even arrest the repair of a fracture?

Movement

Interposed soft tissues

Gross misalignment

Infection

Pre-existing bone disease

152

Page 153: finals2009.wikispaces.com · Web viewPleural effusion which is secondary to hypoalbunimaemia Q4. What initial investigations would be implemented to confirm this diagnosis? Urine

4.4          Fractures-general

By the end of Phase II  students should be able to:

• manage the immediate first aid care of a fracture

• describe when the possibility of fracture should be suspected and request appropriate

radiological investigations

• apply a Plaster of Paris backslab to upper and lower limbs

• describe to a patient the management of the commoner fractures

• describe how the processes of healing may be promoted or retarded

•  recognise the possibility of acute compartment syndrome and fat embolism

 

153

Page 154: finals2009.wikispaces.com · Web viewPleural effusion which is secondary to hypoalbunimaemia Q4. What initial investigations would be implemented to confirm this diagnosis? Urine

4.5

Mrs OL is a 70 year old lady that has been brought to the Emergency department. She

has had a fall.

She complains of pain in her right leg and right wrist.

Question 1:

What signs may be found if her neck of femur is fractured? [2]

External rotation and leg shortening.

Question 2:

Outline some tests that should be conducted prior to surgery?[3]

X-Rays (especially lateral views), bloods (FBC and U&E).

Look at this investigation.

Question 3:

What is the name and resulting deformity of this common fracture in postmenopausal

women?[2]

Colles fracture and dinner fork deformity.

Question 4:

Aside from increasing age name 4 other causes of osteoporosis? [4]

Anorexia, alcoholic, steroid use, Cushings disease, hyperparathyroidism,

thyrotoxicosis, myleoma, primary biliary cirrhosis, rheumatoid arthritis.

154

Page 155: finals2009.wikispaces.com · Web viewPleural effusion which is secondary to hypoalbunimaemia Q4. What initial investigations would be implemented to confirm this diagnosis? Urine

Question 5:

Name, aside for osteoporosis, two causes of pathological fractures?

Pagets disease (increased bone turnover associated with hypernucleated

ostroclasts, causing bone remodelling, enlargement and weakness) and secondary

deposits (common cancers include breast, lung, thyroid, kidney, prostate, cervix,

bowel, and from multiple myeloma).

Extended Question:

What is the blood supply to the femoral head and why is this relevant in this case?

Lateral circumflex femoral artery and medial circumflex femoral artery and the artery

to the femoeal head which is useless in older people. That is why a fracture can

disrupt the arterial supply and lead to avascular necrosis of the femoral head.

N.B: Bits I couldn’t fit in HRT and menopause, intertrochanteric and intracapsular fractures.

4.5 Fractures-the Elderly

By the end of Phase II students should be able to:

• recognise those fracture patterns particular to the elderly

• recognise the underlying aetiological factors predisposing to fracture in the elderly

• request appropriate radiographic examinations

• recognise the possibility of significant underlying disease which may influence definitive

treatment of the fracture and initiate appropriate investigation

155

Page 156: finals2009.wikispaces.com · Web viewPleural effusion which is secondary to hypoalbunimaemia Q4. What initial investigations would be implemented to confirm this diagnosis? Urine

4.7         

27 year old male presents in A&E having sustained a compound fracture of the tibia

bone following an RTA.

1. What is his immediate management?

Airway- check airway patent. If not, try jawthrust or headtilt and chin lift. Insert geudal

airway if needed or other airways (ET, tracheostomy) if it is severely compromised.

Breathing- check RR, pulse oximeter to check O2 sats.

Circulation- check BP, pulse, capillary refill. 2x IV cannula insertd and infuse fluid

(type of fluid depends on hospital and severity of the patient) if needed.

2. a) Outline the treatment/ management of compound fractures.

Emergency theatre within 6 hours. The wound should be covered (and compressed)

with an antiseptic dressing and the limb immobilised using a splint (whilst waiting for

theatre). (Injury should be photographed so repeated examination is avoided). If

there is gross deformity, manipulation under sedation may be needed before

splinting. The neurovascular status must be checked before and after this is carried

out. In theatre, the wound is thoroughly washed out and any contaminated or dead

tissue is debribed. This is to avoid infection and better union for healing. The fracture

is assessed and stabilised (usually by internal or external fixation)

b) Which 2 drugs should you consider

broad spectrum antibiotics (such as cefazolin, or clindomycin if there

is pencillin allergy)

tetanus booster should be given

3. Eight hours later he is complaining of continued pain in his lower leg and his

posterior tibialis and pedis dorsalis pulses are absent. Write a note on your

diagnosis and management.

Compartment syndrome. As pressure rises within a fascial compartment in the limb

(due to e.g. haemorrhage), the capillary blood flow to the tissues decreases.

156

Page 157: finals2009.wikispaces.com · Web viewPleural effusion which is secondary to hypoalbunimaemia Q4. What initial investigations would be implemented to confirm this diagnosis? Urine

Ischaemia occurs when the capillary pressure is less than the compartment/ tissue

pressure. Management: Elevate the limb, remove all bandages and split the cast all

the way to the skin. If this fails to relieve the pain, remove the plaster. If symptoms

continue then a fasciotomy maybe needed to relieve the pressure.

4. What are the immediate, early and late complications of a compound fracture? Immediate Haemorrhage

Neurovascular and visceral damage

Early Compartment syndrome

Infection (worse if associated with metal work)

Late Problems with union (delayed, non- and malunion)

Thromboembolic events (prolonged immobilisation)

Avascular necrosis

Sudek’s atrophy / reflex sympathetic dystrophy

Myositis ossificans

Joint stiffness

Growth disturbance

5. Write a note on the physiological process of bone healing.

5 stages:

Bleeding into the fracture

An inflammatory reaction is set up and granulation tissue forms

The cells proliferate and early bone and cartilage form – callus

This then consolidates – as woven bone which then transforms to stronger lamellar

bone

The bone then remodels to the normal stresses its placed under

157

Page 158: finals2009.wikispaces.com · Web viewPleural effusion which is secondary to hypoalbunimaemia Q4. What initial investigations would be implemented to confirm this diagnosis? Urine

4.7 Open Fractures

By the end of Phase II the student should be able to:

• recognise an open fracture

• apply the principles of immediate stabilisation and wound coverage

• prescribe tetanus toxoid and antibiotic appropriately

• have an understanding of the importance of debridement

• act as part of the team in the definitive care for these fractures

• recognise the possibility of complications such as chronic osteomylitis

158

Page 159: finals2009.wikispaces.com · Web viewPleural effusion which is secondary to hypoalbunimaemia Q4. What initial investigations would be implemented to confirm this diagnosis? Urine

4.7

A 32-year-old man is brought by paramedics to A&E after being involved in a

motorcycle accident; impact occurred at 65 miles per hour.

(ABCs) assessment, x-rays of the lateral C-spine, chest, and pelvis have been

obtained. No injuries are detected.  The patient is hemodynamically stable.

Upper limb examination normal.

Examination of the left hip, knee, and ankle is normal, and the distal neurovascular

exam is intact.

Examination of the right lower extremity reveals a 12-cm break in the skin, with

exposed bone and periosteum on the anteromedial aspect of the leg. There is an

obvious deformity of the leg.

An anterior-posterior and lateral x-ray of the tibia and fibula is taken. The patient has a

midshaft tibia/fibula fracture with moderate comminution.

How would you dress the wound?

A sterile dressing such as dilute Betadine should be applied in A&E and should not

be removed until the patient arrives in surgery. Multiple examinations of the wound

should be avoided to prevent nosocomial contamination and infection.

Would you start him on antibiotics or wait until he goes to surgery?

Antibiotics should be started as soon as possible. At our institution, a first-generation

cephalosporin such as cefazolin 1 g every 8 hours and an aminoglycoside such as

gentamicin 80 mg every 8 hours are the antibiotics of choice. At other institutions,

cefazolin, vancomycin, and gentamicin, alone or in combination, are used.

 The patient's last tetanus booster was 7 years ago and he completed the initial

159

Page 160: finals2009.wikispaces.com · Web viewPleural effusion which is secondary to hypoalbunimaemia Q4. What initial investigations would be implemented to confirm this diagnosis? Urine

series as a child. The current wound appears dirty.

What would you do now?

In this case, a tetanus booster is warranted (Td 0.5 cc IM).

What is the most important aspect of caring for open fractures?

Debridement of all necrotic, nonviable tissues including skin, subcutaneous tissue,

muscle, tendon, and bone are crucial to the successful management of open

fractures. If tissues are devitalized, antibiotics and nutrients, which are important in

fighting bacteria and in wound healing, will not reach them. The devitalized tissues

will become necrotic and become a nidus for infection.

What are the two forms of fixation avaliable to stabilise a fracture of this sort?

Exteranal fixation - An external fixator is indicated for bad soft tissue injury with or

without gross contamination, because dirt can enter the canal during nailing.

Internal fixing (intramedullary nail)- An intramedullary nail is ideal because it will not

cause further soft tissue damage. The downside of a nail is that you disrupt the

intramedullary blood supply of bone, which could have an effect on the healing of the

fracture.

4.7 Open Fractures

By the end of Phase II the student should be able to:

• recognise an open fracture

• apply the principles of immediate stabilisation and wound coverage

• prescribe tetanus toxoid and antibiotic appropriately

• have an understanding of the importance of debridement

• act as part of the team in the definitive care for these fractures

• recognise the possibility of complications such as chronic osteomylitis

160

Page 161: finals2009.wikispaces.com · Web viewPleural effusion which is secondary to hypoalbunimaemia Q4. What initial investigations would be implemented to confirm this diagnosis? Urine

4.8

Tom is 35-year-old man who presents to A&E with an injured knee. He was tackled

whilst playing football, and fell twisting his knee. It started hurting immediately, and

has since become swollen; he is finding it difficult to weight bear.

i) Describe the structures that help to stabilise the knee.

Anterior - the tendon of quadriceps femoris

Lateral - the ilio-tibial tract + tendon of biceps femoris

Posterior - popliteus + semimembranosus

Medial - tendons of sartorius, gracilis + semitendonosus.

Or

Static stabilisers = intrinsic static stabilisers are the anterior + posterior cruciate ligaments.

Extrinsic static stabilisers are the medial + lateral collateral ligaments.

Dynamic stabilisers = Quadriceps – rectus femoris

Vastus lateralis/medialis/intermedius

Hamstrings – Biceps femoris

Semitendonosus

Semimembranosus.

ii) In an injury resulting from a side impact on the knee, how might the soft

tissues of the knee joint be affected?

May result in an ‘unhappy triad’ of injuries, namely a torn anterior cruciate ligament, a

torn medial/tibial collateral ligament and a torn medial meniscus.

iii) What features of the menisci and ligaments make the above set of injuries more

likely?

The medial collateral ligament is firmly attached to the medial meniscus, which

means that frequently tearing of the ligament also results in tearing or detachment of

the meniscus.

Meniscal tears more commonly involve the medial meniscus as it is more firmly fixed to

the tibia, compared to the lateral meniscus.

161

Page 162: finals2009.wikispaces.com · Web viewPleural effusion which is secondary to hypoalbunimaemia Q4. What initial investigations would be implemented to confirm this diagnosis? Urine

iv) You examine the knee; name two tests that you would use to investigate

whether an effusion was present in Tom’s knee.

Patella tap test – if there is a substantial effusion then the patella will spring back

against your fingers.

Bulge sign – useful for smaller effusions.

v) You continue your examination with the following tests – describe in the case

of Tom, what you might expect to find on examination?

McMurray’s test? If there is a cartilage tear, its engagement between the tibia and

femur during this manoeuvre leads to severe pain, and sometimes locking.

Anterior draw test? Rupture of the anterior cruciate ligament will allow the tibia to

slide anteriorly on the femur to a greater extent than on the unaffected side.

Posterior draw test? As the posterior cruciate ligament in unlikely to be affected in

this injury, a positive posterior draw test is unlikely.

vi) You decide that Tom has indeed ruptured is anterior cruciate ligament, and torn

his medial meniscus – which surgical procedure may he benefit from? Explain

to him what it involves and what the complications may be.

Trimming or repair of the torn meniscus is essential; this is done via an arthroscopy.

Surgical intervention reduces the recurrent pain, locking and swelling but not the risk

of secondary osteoarthritis.

An arthroscopy involves a general anaesthetic, and the examination of the inside of

the knee joint via a camera inserted through a small incision. During the procedure

repair and trimming of the cartilage can be undertaken using ‘keyhole’ surgical

techniques. The complications include bleeding, infection, knee swelling, recurrence

of symptoms, and reaction to the anaesthetic.

vii) What is the definition of a strain, and a sprain?

Sprain – injury to a ligament, caused by sudden overstretching. As the ligament is

not severed it gradually heals but may take several months. Should be treated by

cold compress.

Strain - Excessive stretching of a muscle resulting in pain and swelling.

What are the treatment options for patients diagnosed with a sprain or strain?

162

Page 163: finals2009.wikispaces.com · Web viewPleural effusion which is secondary to hypoalbunimaemia Q4. What initial investigations would be implemented to confirm this diagnosis? Urine

RICE – rest, ice, compression, elevation

Analgesia/anti-inflammatories

Use of supports

vii) A patient presents with a painful swollen knee, apart from trauma what are some of

the differential diagnoses?

Septic arthritis

Osteoarthritis

Gout

Osteomyelitis

Haemathrosis

What investigation(s) could you perform to differentiate between causes of knee

swelling?

pyrexial?

FBC + Clotting screen

Blood cultures – often positive in septic arthritis

Joint x-ray

Joint aspirate – microscopy + cultures; crystals in gout, gram stain + culture

may isolate infective organism

Serum urate – usually raised in gout >600umol/L

viii) How may a joint become infected and what factors may make an individual

more prone?

direct injury

blood-borne infection from an infected skin lesion, or other site by

haematogenous spread.

Chronically inflamed joints are more prone to infection e.g. RA, as are artificial joints

Immunosuppressed patients – drug induced/AIDS

Elderly

Alcohol + drug abuse.

163

Page 164: finals2009.wikispaces.com · Web viewPleural effusion which is secondary to hypoalbunimaemia Q4. What initial investigations would be implemented to confirm this diagnosis? Urine

4.8 Soft Tissue Injuries

By the end of Phase II the student should be able to:

• distinguish clinically between strain, sprain, and rupture of ligaments and muscles

• assess traumatic knee effusions

• initiate investigations in patients with soft tissue injuries

• initiate management of soft tissue injuries

• recognise the common underlying aetiological factors predisposing to infection in soft

tissue injuries and wounds

• identify infection in soft tissue injuries

• request appropriate radiographic examinations and other investigations as required to

assess the extent of soft tissue infections

• recognise the possibility of a gas-forming infective organism being present, the

significance of this to the patient's health and the principles of investigation and

management of this infection

• assess skin lacerations, including the recognition of foreign bodies, perform simple

suturing under local anaesthesia and dress wounds

• initiate surgical management of soft tissue injuries

164

Page 165: finals2009.wikispaces.com · Web viewPleural effusion which is secondary to hypoalbunimaemia Q4. What initial investigations would be implemented to confirm this diagnosis? Urine

4.9

Mr Willow, 25 years old presented in A&E having sustained a chain saw injury to his

left hand.

a) What further questions would you ask?

When, where, how?

Dominant hand?

Occupation?

Hobbies?

Previous hand injuries and disability?

b) On examination list 3 features you would look for on;

i. Inspection

Hand posture, where the lacerations are, what is underneath it (tendons, vessels,

nerves), is hand/finger perfused? Is there soft tissue loss?

ii. Palpation

ulnar and radial pulses present,

is hand/fingers cold,

is sensation normal distal to injury.

range of movement

From the examination you suspect he has injured a flexor tendon in his hand.

c) Name the two tendons you are concerned about?

flexor digitorum superficialis

flexor digitorum profundus

d) How could you test the integrity of these tendons?

FDS inserts into middle phalanx, FDP inserts into distal phalanx.

Failure to flex the DIP joint against resistance indicates divided FDP. If this is intact

165

Page 166: finals2009.wikispaces.com · Web viewPleural effusion which is secondary to hypoalbunimaemia Q4. What initial investigations would be implemented to confirm this diagnosis? Urine

but flexion at PIP joint is affected then division of FDS.

You have also noticed he has a laceration at the wrist.

e) Which nerves are you concerned he may have damaged?

Median

ulnar

f) Give the clinical findings for each nerve stated if injured.

median – unable to oppose thumb (paralysis of thenar muscles), fine control of 2nd

and 3rd is impaired (first two lumbricals impaired). APE HAND on trying to make a

fist. Sensory lost over thumb and adjacent two and half digits.

ulnar – difficulty making a fist because of paralysis of most intrinsic hand muscles

(these muscles put fingers into a position of flexion at MCP and extension at

interphalangeal joints) and unopposed long flexors. CLAW HAND . Sensory loss over

medial one and half digits.

g) What clinical signs would suggest a vascular injury?

Pulselessness, temperature, colour, capillary refill, venous return, swelling. Note

absent distal pulsation or pulsating haematomas.

Consider the management of this patient

h) What are the indications for surgical repair? Name 3

tendon injury (open). Nb. Closed rupture of for example extensor tendon (mallet

finger) can often simply be splintered.

nerve injury

i) Outline the need for hand rehabilitation to the patient,

166

Page 167: finals2009.wikispaces.com · Web viewPleural effusion which is secondary to hypoalbunimaemia Q4. What initial investigations would be implemented to confirm this diagnosis? Urine

Rehabilitation involves protected mobilization allowing early movement without

excessive stress on the tendon repair. Wear a splint. It restores gliding function,

minimising adhesions and increases tensile strength. Greatly aiding restoration of

function.

4.9 Limb Injuries

By the end of Phase II the student should be able to:

• diagnose nerve and tendon injuries in the hand

• identify injuries needing operative repair

• discuss rehabilitation in broad outline with the patient with the injured hand

• identify vascular injury in the upper and lower limbs

• identify division of tendons in the upper limb

167

Page 168: finals2009.wikispaces.com · Web viewPleural effusion which is secondary to hypoalbunimaemia Q4. What initial investigations would be implemented to confirm this diagnosis? Urine

4.10

You are a member of an air ambulance team attending to a 28 year old man with

multiple injuries at the site of a road traffic accident.

1. State, in order of priority your measures to ensure patient survival until transfer to a

trauma centre

Ensure airway patency and adequate ventilation by administering supplementary

oxygen.

Intubate and ventilate manually if necessary

Immobilise cervical spine and place patient on a long spine board.

Restore circulatory blood volume by crystalloids, plasma expanders and colloids as

necessary with pulse, BP, JVP monitoring

Commence head injury observations.

Alleviate pain with narcotic analgesics

(5 marks)

2. Whilst you are attempting to stabilize the patient, he becomes increasingly

breathless.

i) State the two probable causes for his progressive respiratory failure

Tension Pneumothorax

Haemothorax

(2 marks)

ii) List the clinical signs you would elicit to confirm your diagnoses.

Cyanosis

Tracheal shift

Hyper-resonance or dull to percussion, with diminished or absent breath sounds in

the chest

Evidence of injury to chest wall on gently springing the rib cage.

(4 marks)

iii) What would be your immediate management of a tension pneumothroax?

168

Page 169: finals2009.wikispaces.com · Web viewPleural effusion which is secondary to hypoalbunimaemia Q4. What initial investigations would be implemented to confirm this diagnosis? Urine

Wide bore cannula incision in the mid clavicular line, 2nd intercostals space on the

affected side.

(2 marks)

iii) State one blood investigation that would be particularly useful to monitor his

condition (if you had available facilities).

Arterial Blood Gases

(1 mark)

3. The patient is trapped in the wreckage of his vehicle and his transfer is delayed by

three hours.

i) Discuss your supportive measures.

Prevent hypothermia, administer oxygen and support ventilation.

Set up iv infusion and maintain normal blood pressure

Maintain constant verbal and tactile communication and reassure the conscious

patient.

Treat life and limb threatening emergencies as facilities permit.

(4 marks)

ii) What is meant by “the golden hour”

The golden hour is the 60 minutes after the initial trauma when fairly severe injuries

can still be treated and lives saved.

(2 marks)

Extended question:

Draw and label a chest drain.

(5 marks)

169

Page 170: finals2009.wikispaces.com · Web viewPleural effusion which is secondary to hypoalbunimaemia Q4. What initial investigations would be implemented to confirm this diagnosis? Urine

4.10 Multiple Trauma

By the end of Phase II the students should be able to:

• identify the basic principles of resuscitation in the multiply traumatised patient according

to ATLS guidelines

• be aware of the injury severity grading systems

• describe the importance of the 'golden hour'

• identify the range of investigative procedures in a patient with multiple trauma such as

radiographs, peritoneal lavage and urethography and be aware of the indications for

these

• identify the indications for immediate life support procedures such as endotracheal

intubation, needle thoracocentesis or insertion of chest drain and drainage of cardiac

tamponade

170

Page 171: finals2009.wikispaces.com · Web viewPleural effusion which is secondary to hypoalbunimaemia Q4. What initial investigations would be implemented to confirm this diagnosis? Urine

4.11

Mr Segaren, a 25 year old jack the lad, is brought to the A&E department having

sustained a closed head injury from a blow to the back of the head outside Evolve in

Leamington.

1. i) What is meant by “coup” and “contrecoup” injury with reference to closed head

injuries?

Coup injury is damage to the brain at the site of the blow.

Contrecoup injury represents damage to the brain on the opposite side to the blow

due to brain movement within the rigid skull.

(2 marks)

ii) What is the difference between primary and secondary brain injury?

Primary brain injury is damage that occurs to the brain immediately as the result of

trauma and is directly related to the amount of force applied to the head.

Secondary brain injury results from complications associated with the initial trauma.

(2 marks)

iii) List four complications that can contribute to secondary brain injury.

Respiratory complications eg hypoxia, hypercapnia, airway obstruction)

Hypovolaemic shock

Intracranial bleeding

Cerebral oedema

Epilepsy

Infection

(4 marks)

2. i) you asses his level of consciousness. He opens his eyes only in response to pain,

he is making random words but no conversational exchange and he flexes his limbs on

painful stimulus. What does he score on the Glasgow coma scale?

3 points for motor response – flexor response to pain

3 for inappropriate speech

171

Page 172: finals2009.wikispaces.com · Web viewPleural effusion which is secondary to hypoalbunimaemia Q4. What initial investigations would be implemented to confirm this diagnosis? Urine

2 for eye opening in response to pain

total of GCS= 8.

ii) Assuming he is unconscious, list the physical signs you would elicit to establish the

extent of the intracranial injury

Blood pressure, pulse and respiration

Pupillary sizes and reflexes

Reflexes – Babinski

Fundoscopy for papilloedema

(4 marks)

iii) List the types of intracranial bleeding that may be present and state one

investigation that would demonstrate the lesion.

Extradural / Subdural / Subarachnoid / intracerebral

CT/MRI head scan.

(2 marks)

3. Mr Segaren is found to have an extra-dural haematoma. Write short notes on your

management of him. (5 marks)

Protect the airway

Exclude cervical spine injury

Quarter hourly obs of BP and pulse

Intracranial pressure monitoring

Daily ABG/FBC/U&E

Mannitol infusion preoperatively

Surgical evacuation of haematoma with placement of intra-cranial pressure

monitoring device.

Postoperative antibiotic therapy.

(5 marks)

4.11 Head injury

By the end of Phase II students should be able to:

172

Page 173: finals2009.wikispaces.com · Web viewPleural effusion which is secondary to hypoalbunimaemia Q4. What initial investigations would be implemented to confirm this diagnosis? Urine

• make an initial assessment of the patient taking into account the mechanism of injury

• calculate the Glasgow coma scale

• participate in the management of head injury and recognise the importance of

prevention of secondary brain ischaemia

173

Page 174: finals2009.wikispaces.com · Web viewPleural effusion which is secondary to hypoalbunimaemia Q4. What initial investigations would be implemented to confirm this diagnosis? Urine

4.12

Whilst working in the Paediatric Assessment Unit, a 5 year old boy attends with

extensive scalding on his lower limbs and buttock area.

How would you estimate the extent of the burn?

Chart (Lund & Browder) as appropriate for age. (1)

Briefly describe the appetence of 1st, 2nd and 3rd degree burns. (3)

1st – minor, painful erythema

2nd – painful, blistering, partial thickness, non-blanching

3rd – White, brown or black. Absent sensation. No blisters.

They are extensive 3rd degree burns in this child.

Name 2 lab investigations you wish to carry out (2)

FBC

174

Page 175: finals2009.wikispaces.com · Web viewPleural effusion which is secondary to hypoalbunimaemia Q4. What initial investigations would be implemented to confirm this diagnosis? Urine

U & E

Glucose

Group and Save/ X-match

Urinalysis

Coagulation

Name 3 things to be included in your initial management (3)

Analgesia e.g. morphine

Anti- emetic e.g. Cyclizine

Ensure airway patent/protected

Oxygen

IV fluids

Antibiotics

Tetanus prophylaxis

Urinary catheter

Mention 3 complications of serious burns in any patient (3)

Hypovolaemic

Renal failure

Sepsis/toxic shock

Stress Ulcers

Pulmonary dysfunction

Cardiac arrest

Post-traumatic stress

Scarring/Keloid formation

3 Days after admission, the patient become pyrexial, hypotensive and complains of

nausea.

What is the likely reason for this? (1)

Toxic shock syndrome/Sepsis

What is the organism most commonly responsible? (1)

Staphylococcus aureus

Extended Question

What may make you suspect non-accidental injury in a child in terms of burns?

Mention one (1)

175

Page 176: finals2009.wikispaces.com · Web viewPleural effusion which is secondary to hypoalbunimaemia Q4. What initial investigations would be implemented to confirm this diagnosis? Urine

Explanation unlikely

Late presentation

Other injuries also present

Scalds on the buttocks, lower limbs and hands

Small, circular (cigarette burns)

4.12 Burns

By the end of Phase II students should be able to:

• assess the extent and depth of a burn

• plan appropriate investigations for a patient with severe burns

• initiate immediate management of a patient with burns, recognising the possible

complications of burns, including airways obstruction and the occurrence of toxic shock

syndrome in children

• be aware of burns as a form of non-accidental injury

• discuss with patients the physical and mental effects of burns

• discuss management of burns with physiotherapists and occupational therapists

176

Page 177: finals2009.wikispaces.com · Web viewPleural effusion which is secondary to hypoalbunimaemia Q4. What initial investigations would be implemented to confirm this diagnosis? Urine

4.13

A 78 year old lady presents with hip pain which she has had for many years and has

worsened over the past few months. She has pain in the hip, which is typically worse

at the end of the day and after doing her housework and other activities. She also

finds that she has stiffness in the morning but this is better by the end of the day.

These problems also severely affect her mobility and gait and have made her prone to

several falls in the past.

1) The following radiological pictures show the classical features of a particular

condition. Name the condition and describe the major radiological changes that occur

with this condition (5 marks).

Osteoarthritis

Reduced joint space

Marginal osteophytes

Sclerotic bone changes

Cyst formation

2) What signs are found on examination?

Joint tenderness

Crepitus on movement

177

Page 178: finals2009.wikispaces.com · Web viewPleural effusion which is secondary to hypoalbunimaemia Q4. What initial investigations would be implemented to confirm this diagnosis? Urine

Limitation of range of movement

Joint instability

Joint effusion and variable levels of inflammation

Bony swelling

Wasting of muscles

3) Briefly outline the pathphysiological changes associated with the condition (8

marks).

The essential feature is degeneration of the cartilage:

Fibrillation (cracks in the surface cartilage)

Loss of cartilage matrix

Chondrocyte proliferation and death

Fibrous repair tissue fills the cracks in the cartilage

Subchondral bone cysts form from exposure to synovial fluid pressure

Loss of cartilage with eburnation (ivory like thickening and polishing) of bone surface

Osteophyte formation (Heberden’s nodes – osteophytes at the DIP)

Synovitis caused by fragments of cartilage in joint space.

4) What typical investigations would you do for this disease?

Blood tests. There is no specific test; the ESR and CRP are normal. Rheumatoid

factor only when the damage is advanced.

X-rays. As above. These are only abnormal when the damage is advanced.

MRI. This demonstrates early cartilage and subchondral bone changes.

Arthroscopy. This reveals early fissuring and surface erosion of the cartilage.

5) Describe the major treatment options available?

Physical measures

Weight loss and exercise for strength and stability.

Walking aid held in contralateral hand in hip arthritis and in ipsilateral hand if it’s the

knee.

Regular exercises (e.g. quadriceps exercises in knee OA) and keep active.

178

Page 179: finals2009.wikispaces.com · Web viewPleural effusion which is secondary to hypoalbunimaemia Q4. What initial investigations would be implemented to confirm this diagnosis? Urine

Medication

Should only be used with severe disease. Simple analgesia first e.g. paracetamol

and if no good NSAIDs but prescribe with adequate explanation of side effects and

can be left off on good days.

Surgery

Total hip replacement arthroplasty for severe OA, reduces pain and stiffness and

greatly increases function.

179

Page 180: finals2009.wikispaces.com · Web viewPleural effusion which is secondary to hypoalbunimaemia Q4. What initial investigations would be implemented to confirm this diagnosis? Urine

Q1) The clinical and radiological pictures shown in picture 1a and 1b) and picture 2a

and 2b) of hands show two different conditions.

Picture 1a

Picture 1b

180

Page 181: finals2009.wikispaces.com · Web viewPleural effusion which is secondary to hypoalbunimaemia Q4. What initial investigations would be implemented to confirm this diagnosis? Urine

Name the condition in picture 1a) and 1b) above and describe some of the salient

features.

Osteoarthritis of the hands

Hand joint involvement showing bony thickening of the DIP (Heberden nodes) and

PIP joints (Bouchard nodes).

Picture 2a Picture 2b

The above hands (2a and 2b) are those of a 65 year old lady who has had this problem

with her hands for many years and describes how her hands are often very swollen,

painful and stiff, especially in the morning. This eventually involved her larger joints

i.e. ankles, knees, hips, elbows, shoulders, cervical spine and TMJ. She is thus very

debilitated with this condition. She also suffers with breathing difficulties and has

problems with her eyes.

Q2) Name the condition in picture 2a) and 2b) above and describe some salient

features that you find in the hands with this condition?

Rheumatoid arthritis of the hands

181

Page 182: finals2009.wikispaces.com · Web viewPleural effusion which is secondary to hypoalbunimaemia Q4. What initial investigations would be implemented to confirm this diagnosis? Urine

Sausage shaped fingers and MCP joint swelling. Later on, ulnar deviation and volar

subluxation (partial dislocation) at MCP joints Boutonniere and swan-neck deformity

of the fingers or z-defromity of the thumbs. The wrist subluxes and the radial head

becomes prominent (piano-key). Extensor tendons in the hand may rupture and

adjacent muscles waste.

Findings on xrays: Soft tissue, juxta-articular osteoporosis; ↓joint space. Later on:

bony erosions (+/- subluxation) and (+/- complete carpal destruction).A

Q3) Briefly outline the pathophysiology of this condition.

The essential feature in joints is chronic nonsuppurative inflammation of the synovial

membrane that can destroy the articular cartilage, bone, and adjacent tissues.

Chronic inflammatory synovitis

Synovial membrane is thickened with villous outgrowth (hypertrophic villous

synovitis)

Proliferating synovial tissue extends over the surface of the articular cartilage as

“pannus”.

Pannus covers the cartilage, eroding into cartilage and bone

Neutorphils are prominent in the joint fluid

B and T lymphocytes (especially CD4 helper cells) are found in the synovial stroma

Synovial antigen presenting cells (dendritic cells and macrophages) are activated

Pannus comes to resemble a piece of highly vascular, active, cellular lymphoid tissue

Osteoclastic activity in underlying bone results in osteoporosis and bone erosions

Inflammation of tendon sheaths and other synovium-lined tissues.

Q4) Outline the major investigations for this condition.

182

Page 183: finals2009.wikispaces.com · Web viewPleural effusion which is secondary to hypoalbunimaemia Q4. What initial investigations would be implemented to confirm this diagnosis? Urine

No one specific laboratory test is diagnostic of RA. The diagnosis is made primarily

on the clinical features (an arthritis involving small peripheral joints) but blood test

can be done with the following features:along with:

↓ HB (Normochromic or hypochromic anaemia, and other indicators of a chronic

systemic acute phase response e.g. elevated globulins, decreased albumin).

ESR

CRP

Rheumatoid factor (often negative in the start and becoming positive in 80%). NB

also raised with other conditions.

↓WCC

ANA (+ve 30%)

Radiology: Plain radiographs, occasionally CT and/or MRI

Biopsy: Synovium, skin, nodules, vessels, lymph nodes – rarely necessary.

Q5) Outline the treatment options associated with this condition

The aim is to minimize symptoms and improve prognosis

Multidisciplinary approach

Regular exercise

Physiotherapy

Occupational therapy

Household aids and personal aids (orthoses) e.g. wrist splints.

Intra-lesional steroids (joint injection)

Surgery – to improve function, not for cosmetic effect.

Simple analgesics or NSAIDs

( no affect on long term outcomes)

2nd Line DMARDs (disease modifying anti-rheumatic drugs)

Start these if there is persisting synovitis for more than 6 weeks. Sulfasalazine is the

183

Page 184: finals2009.wikispaces.com · Web viewPleural effusion which is secondary to hypoalbunimaemia Q4. What initial investigations would be implemented to confirm this diagnosis? Urine

first choice in the UK

Sulfasalazine: s/e nausea, headaches, ↓marrow, sperm count and hepatitis.

Methotrexate: s/e nausea, diarhoea, cough, SOB, cirrhosis.

Ciclosporin: s/e nausea, tremor, gingival hypertrophy, hypertension.

Gold: s/e ↓marrow, proteinuria.

Azathioprine: s/e ↓marrow, nausea, LFTs, oncogenic.

Penicillamine: s/e ↓marrow, proteinuria.

Hydroxychloroquine: s/e permanent retinopathy with high doses, tinnitus, headache.

Adjunctive therapy with corticosteroids

For severe systemic illness, intermittent and troublesome mono/pauciarticular

inflammation +/- vasculitis. Steroids may be given as intermittent pulses

e.g.methylprednisolone 80-120mg im or 500-1000mg iv or can injected into inflamed

joints. Or can be used until the DMARDs take effect. Have numerous s/e especially

osteoporosis and potential for opportunistic sepsis.

Biological agents

Target pro-inflammatory cytokines e.g. TNF and IL1

Chimeric monoclonal antibodies (infliximab)

Fc fusion proteins with soluble TNF receptor (etanercept)

Expensive but highly effective

More recently recombinant IL-1 receptor antagonists are also an effective agent.

184

Page 185: finals2009.wikispaces.com · Web viewPleural effusion which is secondary to hypoalbunimaemia Q4. What initial investigations would be implemented to confirm this diagnosis? Urine

4.13 Investigations

By the end of Phase II students should be able to:

• interpret the major radiological changes of osteoarthritis of the hip and of rheumatoid

arthritis of the hand

• request and interpret routine investigations for auto-immune disease

• relate these changes to the underlying pathology

185

Page 186: finals2009.wikispaces.com · Web viewPleural effusion which is secondary to hypoalbunimaemia Q4. What initial investigations would be implemented to confirm this diagnosis? Urine

4.14

A 75 year old gentleman presents in A&E complaining of an excruciatingly painful left

knee. The pain came on suddenly the previous night and he has never suffered from

similar symptoms before. There is no recent history of trauma to the knee.

1. What is your differential diagnosis?

Crystal arthropathy (e.g. gout, pseudogout)

Septic arthritis

Monoarthritic presentation of a polyarticular disease (e.g. OA, RA)

2. On examination he has a red, hot, swollen knee with significantly reduced

range of movement due to pain. You decide to aspirate joint fluid. What 3 other

investigations would you perform on him?

Take blood for: FBC (looking for ↑WCC), ESR/CRP (inflammatory markers), U&E

(renal status important in gout), serum urate level

Blood cultures (if sepsis is possible)

Knee X-ray – look for erosions, calcification, widening/loss of joint space, changes

in underlying bone (e.g. sclerotic areas, osteophytes)

3. Joint aspiration reveals turbid, yellow synovial fluid, rich in neutrophils. No

organisms have been identified in culture. Microscopy reveals negatively

birefringent needle-shaped crystals. What is the diagnosis and how would you

manage this condition in the acute setting?

Goutut.

NSAIDs (e.g. ibuprofen, naproxen) – analgesic/anti-inflammatory effect [If NSAIDs

are contra-indicated, e.g. due to peptic ulcer, give colchicine PO]

Joint aspiration (reduces pressure)

186

Page 187: finals2009.wikispaces.com · Web viewPleural effusion which is secondary to hypoalbunimaemia Q4. What initial investigations would be implemented to confirm this diagnosis? Urine

NB: NSAIDs & colchicine are problematic in renal failure – steroids can be very effective

but not without side-effects!

4. What are the predisposing factors for the above condition?

chronic renal disease

drugs (e.g. thiazide diuretics, low-dose aspirin)

hypertension

lead toxicity

alcohol

genetic (e.g. glucose-6-phosphate deficiency)

idiopathic gout

increased purine turnover (myeloproliferative disorders, e.g. polycythaemia vera –

lymphoproliferative disorders, e.g. leukaemia – others, e.g. carcinoma)

genetic (e.g. Lesch-Nyhan syndrome)

5. How would you prevent similar attacks in the future?

Avoid purine-rich foods (e.g. offal, oily fish), obesity & alcohol excess

Consider long-term allopurinol (↓ serum urate) but not until 3 wks after an attack

know allopurinol’s mode of action, dose, side-effects!

Control hypertension, avoid diuretics and aspirin

E. If microscopy of the synovial fluid had revealed positively birefringent rhomboid-

shaped crystals what would the diagnosis be? What are the risk factors for this

condition?

Pseudogout.

Risk factors:

Old age

Dehydration

Hyperparathyroidism

DM

Acromegaly

187

Page 188: finals2009.wikispaces.com · Web viewPleural effusion which is secondary to hypoalbunimaemia Q4. What initial investigations would be implemented to confirm this diagnosis? Urine

Haemochromatosis

Wilson’s disease

↓ PO4/ ↓ Mg

A 35-year old gentleman presents in A&E with an acutely painful knee. The pain came

on over the last 24 hours and he is also feeling feverish and generally unwell. His

doctors suspect that he might be suffering from septic arthritis.

1. What are the possible micro-organisms responsible for this condition?

(in order of decreasing prevalence)

Staph aureus

Streptococci (mainly – Strep pneumoniae, Group A strep)

Gram –ve bacilli (mainly – H. influenzae, E. coli)

Neisseria gonorrhoea

Anaerobes

2. What are the 3 possible routes by which the joint becomes colonised?

Direct inoculation (e.g. during joint surgery/aspiration)

Spread from infected peri-articular tissue (e.g. spread from osteomyelitic focus)

Haematogenous spread (due to transient/persistent bacteraemia, e.g. skin/lung/bladder

infection)

3. Name 3 host factors that can predispose to non-gonococcal septic arthritis

Increasing patient age

Decreased immunocompetence (e.g. diabetes)

Pre-existing joint disease (e.g. RA patients)

4. You decide to aspirate synovial fluid from the patient’s knee and send it for

culture. How would you treat his condition while awaiting culture results?

Ask for advice from orthopaedic surgeons and microbiologists!

188

Page 189: finals2009.wikispaces.com · Web viewPleural effusion which is secondary to hypoalbunimaemia Q4. What initial investigations would be implemented to confirm this diagnosis? Urine

Start empirical antibiotic treatment depending on suspected organism (e.g. for staphs

– flucloxacillin & benzylpenicillin IV until sensitivities are known)

Rest affected joint (no bearing of weight)

Encourage early joint mobilisation as symptoms subside – vigorous physio!

4.14 Acute onset joint pain

By the end of Phase II the students should be able to:

• recognise the symptoms of infection of bone or soft tissue

• initiate diagnostic procedures and treatment of the condition

• recognise the possibility of septic arthritis

• recognise the indications for surgery

• discuss the prognosis and rehabilitation with the patient or parents

• recognise acute and chronic gout and initiate management

189

Page 190: finals2009.wikispaces.com · Web viewPleural effusion which is secondary to hypoalbunimaemia Q4. What initial investigations would be implemented to confirm this diagnosis? Urine

4.15

Mr L is a 65 year-old man who presented to his GP with unremitting chest pain that is

particularly worse at night. His worried wife, who accompanied him, has also noticed

that he has lost weight recently. His significant past medical history comprises of

successful management of prostatic carcinoma by brachytherapy 3 years ago.

a. What is brachytherapy?

Radiotherapy administered by implanting

radioactive wires or grains into or close to a

tumour

b. Name 3 differential diagnoses for his presenting complaint

rib tumour, primary or metastatic

muscular strain

rib fracture

The GP was concerned that Mr L may have bone metastases from his prostatic

carcinoma.

c. Which investigations would you carry out to confirm or refute the presence of

bone cancer?

Serum alkaline phosphatase

Calcium

Prostate specific antigen

d. Name 2 types of primary malignant neoplasms of bone

Osteosarcoma

Fibrosarcoma

Chondromas

e. Name three treatment options for Mr L.

190

Page 191: finals2009.wikispaces.com · Web viewPleural effusion which is secondary to hypoalbunimaemia Q4. What initial investigations would be implemented to confirm this diagnosis? Urine

Analgesics

Anti-inflammatory drugs

Local radiotherapy

Hormonal therapy

Internal fixation of pathological fracture

4.15 Bone and Soft Tissue Tumour

By the end of Phase II the student should be able to:

• identify the broad pathological types of bone and soft tissue tumours which affect the

musculo-skeletal system

• recognise the possibility of underlying bone or soft tissue tumour in patients presenting with

musculoskeletal symptoms

• detect major abnormalities on examination suggestive of bone and soft tissue tumour

• recognise the importance of staging and how this may be performed

• be aware of the concepts of treatment of bone and soft tissue tumours

191

Page 192: finals2009.wikispaces.com · Web viewPleural effusion which is secondary to hypoalbunimaemia Q4. What initial investigations would be implemented to confirm this diagnosis? Urine

Mrs JR, a 76 year old female, presents to your GP clinic with a 2 day history of a painful

left knee.

1. List four possible causes of a painful solitary large joint

trauma

Septic arthritis

Gout/pseudogout

Gonococcal arthritis

Haemophilia

Rheumatoid arthritis

The following is found on examination

Left knee examination shows:

overlying skin red and warm

swollen, painful left knee joint

pain on all left knee movement – with diminished movements due to pain and

spasm

2. What additional features would you expect on general examination that would

suggest a infective aetiology

Swinging pyrexia

Tachycardia

3. From the clinical findings what is the most likely diagnosis

Septic arthritis

4. What predisposing factors need to be considered in any patient with this

condition?

debilitating disease

infection elsewhere such as an infected leg ulcer

192

Page 193: finals2009.wikispaces.com · Web viewPleural effusion which is secondary to hypoalbunimaemia Q4. What initial investigations would be implemented to confirm this diagnosis? Urine

rheumatoid arthritis

immunosuppression/corticosteroids

diabetes mellitus

recent joint surgery

joint prostheses

penetrating injury

5a) What is your immediate management?

Admit to hospital as risk of rapid joint damage

5b) What investigations should be ordered?

blood cultures

full blood count for leucocytosis

erythrocyte sedimentation rate

C-reactive protein

aspiration of synovial fluid - usually purulent with a neutrophil count above

50,000 per mm3, and low glucose concentration

anti-streptolysin O titre

Plain X-ray of left knee

What radiological findings would expect?

None at this stage, first radiological changes appear after 2-3 weeks.

5. What radiological features may you find in 2-3 weeks?

Narrowed joint space, and patchy rarefaction

6. What aetiological agents are likely to be isolated from your investigations name

2.

Staphylococcus aureus is the most common agent.

Others may include:

Streptococcus pyogenes

pneumococcus

193

Page 194: finals2009.wikispaces.com · Web viewPleural effusion which is secondary to hypoalbunimaemia Q4. What initial investigations would be implemented to confirm this diagnosis? Urine

Haemophilus influenzae, although most likely in those under three years

gonococcus

gram negative rods, such as E. coli, Pseudomonas

All the above are primary causes, but may be secondary to osteomyelitis of an

adjacent bone.

7. What immediate antibiotic agents would you expect to be administered?

Should check local protocol, and most recent BNF, but likely agents include

clindamycin alone or flucloxacillin and fusidic acid

if Haemophilus influenzae then give amoxicillin or cefuroxime

8. What complications could arise from this condition?

chondrolysis

septic dislocation

avascular necrosis

shortening

late degenerative change

Objective: painful solitary large joint

194

Page 195: finals2009.wikispaces.com · Web viewPleural effusion which is secondary to hypoalbunimaemia Q4. What initial investigations would be implemented to confirm this diagnosis? Urine

4.16

A 29 year old woman presents to medical outpatients with a 3 month history of painful,

swollen proximal interphalangeal joints associated with early morning stiffness and

malaise and lethargy.

What is the likely cause of her arthopathy? (1 mark)

Rheumatoid arthritis

Name 3 investigations that you may wish to conduct and the likely finding if it is the

diagnosis suggested in question 1? (6 marks)

Blood count – low Hb, high platlets,

Rheumatological marker- high rheumatoid factor,

Inflammatory markers – high ESR, high CRP

X-ray – periarticular erosion, soft tissue swelling, angular deformatity, osteoperosis

In the categories below list an associated feature which may occur in this disorder 1)

eyes, 2)skin 3) respiratory 4) cardiovascular 5)renal:

Sjogrens syndrome (dry eyes and mouth), scleromalacia

Rheumatoid nodules , vasculitic rash

Pleural effusion, rheumatoid nodules, pulmonary fibrosis

Pericarditis, pericardial effusion

Chronic renal failure, secondary to amyloidosis or drugs

Outline the pharmaceutical medical management of this condition

Analgesia – paracetamol, NSAIDS

Anti-inflammatory – NSAIDS

Corticosteroids- prednisolone

195

Page 196: finals2009.wikispaces.com · Web viewPleural effusion which is secondary to hypoalbunimaemia Q4. What initial investigations would be implemented to confirm this diagnosis? Urine

DMARDS (Disease Modifying Agents of Rheumatological Diease) D-penacillamine, gold

salts, sulphazalazine

Immune modulatory – anti TNF (Infliximab)

Extended

Breifly outline the role of 2 members of the MDT in the care of this patient

Physician -patient education of disease, medical management, referral management

Physiotherapist – maintain joint mobility through exercise

Occupational therapist – evaluate impact of disease on life, advise easier ways to

perform daily tasks and design and prescribe assistive devices

Orthotist- provide and fit splints and other aids

Surgeon- perform joint replacement

Others you can mention:

Social worker,

Specialist nurse,

GP

196

Page 197: finals2009.wikispaces.com · Web viewPleural effusion which is secondary to hypoalbunimaemia Q4. What initial investigations would be implemented to confirm this diagnosis? Urine

An obese 67y old woman presents to her GP with a 4 month history of

increased swelling , pain and stiffness in her right knee. She suffered a

comminuted fracture of her distal right femur in a road traffic accident

15 years ago?

What is the likely diagnosis?

osteoarthritis.

List 3 risk factors for the development of this condition

Family history of OA

Herediatry disorders (Ehler Danlos)

Metabolic conditions – haemachromatosis, agromegaly

Concominant bone disorders (Pagets, Perthes)

Trauma – previous fractures

Obesity

Repeatative trauma/ occupation (footballers, atheletes, ballet

dancers)

For each of the following features place either RA (for rheumatoid

arthritis) or OA (for osteoarthritis) – for which ever condition it is

commonly associated with.

Number 10 is done for you.

1. Loss of joint space on xray 6. Bouchards node

2. Feltys syndrome 7. Initial synovial inflammation

3. HLA DR1 8. Swan neck deformity

4. Z deformity of the thumb 9. Subchondral cysts

197

Page 198: finals2009.wikispaces.com · Web viewPleural effusion which is secondary to hypoalbunimaemia Q4. What initial investigations would be implemented to confirm this diagnosis? Urine

5. Osteophyte formation 10. Ulnar deviation RA

Answers below:

OA-1,5,6,9

RA- 2,3,4,7,8,10

Briefly outline the management of this condition

Conservative/supportive:

Patient education (explanation of disease and its management).

Lifestyle changes (weight loss, exercise)

Physiotherapy (maintain joint moment, increase muscle strength),

Occupational therapy to supply aids and appliances to aid mobility

Medical:

Pain relief and NSAIDS. Steroid injections

Surgical:

Debridement (removal of dead tissue within the joint)

Artherodesis (fusing a joint)

Osteotomy

Arthroplasty

4.16 Chronic Inflammatory and Degenerative Joint Disease

By the end of Phase II the student should be able to:

• recognise rheumatoid arthritis and distinguish active from inactive disease

• describe the immunological basis of rheumatoid arthritis

• recognise the multi-system manifestations of many auto-immune processes

• diagnose and assess the severity of degenerative disease of the hip and knee

• initiate appropriate investigations

• initiate appropriate management, especially the relief of pain

• discuss with patients the range of aids to daily living that are available

198

Page 199: finals2009.wikispaces.com · Web viewPleural effusion which is secondary to hypoalbunimaemia Q4. What initial investigations would be implemented to confirm this diagnosis? Urine

• describe the referral pathways for patients to physiotherapy and occupational therapy

• recognise the need for surgical assessment

• be able to discuss with patients the treatment options available, the role of rehabilitation and

possible complications of surgical treatment

4.18

A 23 year old man who had previously fractured his leg is now on crutches. He has

noticed that since the use of his crutches, he has experienced loss of sensation and

weakness in his hands. On examination a diagnosis of radial nerve palsy is made.

Q) which actions of the upper limb would he find difficult to perform?

extension of fingers, wrist and elbow

Q) What area of sensation would be lost?

Dorsum of hand – lateral 3 ½ digits with sparing of finger tips

Q) How did the radial nerve become affected?

Compression of nerve in the axilla by incorrect use of crutches

A 42 Year old female is seen by her GP complaining of pain and numbness in her

hands. She notices that her symptoms are classically worse at night and during the

day, during prolonged flexion of the wrist e.g typing or knitting.

Q) What is the diagnosis ?

Carpal tunnel syndrome

Q) What are features on examination?

199

Page 200: finals2009.wikispaces.com · Web viewPleural effusion which is secondary to hypoalbunimaemia Q4. What initial investigations would be implemented to confirm this diagnosis? Urine

Wasting of thenar eminence, weakness of thumb movements

Q) What test can be done to confirm the diagnosis?

Tinnels Test (tapping over median nerve at the wrist to reproduce the

symptoms)

Q) What are the causes of this condition?

Median nerve compression as it passes under flexor retinaculum (in carpal tunnel).

Much more common in females than in males and associations are pregnancy,

rheumatoid arthritis, hypothyroidism, acromegaly, trauma and idiopathic in post

menopausal women ?

Q) What is the treatment for this condition?

Conservative – using splints

Medical – local steroid injections

Surgical – decompression of the flexor retinaculum

4.18 Nerve Compression

By the end of Phase II the student should be able to:

• recognise nerve compression in the upper limb

• initiate investigations for a patient with nerve compression

• be aware of the role of surgery and radiotherapy in the relief of nerve compression

200

Page 201: finals2009.wikispaces.com · Web viewPleural effusion which is secondary to hypoalbunimaemia Q4. What initial investigations would be implemented to confirm this diagnosis? Urine

 4.19

During a neonatal health check, performed the first day after birth, a baby is diagnosed

with developmental dysplasia of the hip (DDH)

Q) Which clinical tests would be carried out to diagnose this?

Ortolani and Barlow Manoeuvre

Q) Name 3 risk factors for DDH?

Breech birth,

+ve family history,

increase in birth weight,

post maturity,

Oligohydramnios

Q) What imaging technique can be used as a screening tool for DDH?

Ultrasound

Q) Briefly outline the treatment of this condition?

Use double nappies – reassess in 3 wks with ultrasound

If still problem – splint hips in abduction

6 – 18 mths closed reduction – frame /gallows

before 6 yrs – open reduction (surgery)

4.19 Congenital Problems

By the end of Phase II the students should be able to:

• detect scoliosis, fixed flexion, kyphosis, varus and valgus deformities

• detect deformity present in the knee, hip, shoulder and the small joints of the hand

• consider the possibility of congenital dislocation of the hip and talipes equinovarus

• perform the diagnostic procedures available at birth and early life to detect these

conditions

201

Page 202: finals2009.wikispaces.com · Web viewPleural effusion which is secondary to hypoalbunimaemia Q4. What initial investigations would be implemented to confirm this diagnosis? Urine

• outline the diagnosis, management and prognosis to the family

202

Page 203: finals2009.wikispaces.com · Web viewPleural effusion which is secondary to hypoalbunimaemia Q4. What initial investigations would be implemented to confirm this diagnosis? Urine

4.21

Q1. What steps can Mrs S take to help prevent herself getting osteoporosis?

203

Dietary – Calcium supplementation, Vitamin D supplementationRegular Weight bearing exercise (30 mins 3 times per week)Smoking cessation

Avoid excess alcoholHip protectors to reduce fall risk Q2. Give four risk factors for Osteoporosis.

Female sex, increasing age, family history of osteoporosis, early menopause, smoking, excessive alcohol, immobility, lack of exercise, malabsorption, prolonged steroid therapy, Cushing’s disease, Hyperparathyroidism, Myeloma, Hypogonadism in men.

4 marks Q3. Give four treatment options for Osteoporosis.

Oestrogen therapy via HRT in postmenopausal womenBiphosphonatesVitamin D (Calcitriol)Calcium supplementationCalcitoninAndrogens for Hypogonadal men

4 marks 4 marks

Q4. Osteomalacia and Osteoporosis may both present with subclinical fractures. What blood tests would help differentiate osteomalacia from osteoporosis?

In osteoporosis, Serum alk phos, plasma calcium and serum phosphate are all normal.

In osteomalacia, serum alk phos is high, plasma calcium is low or low normal and serum phosphate is low.

Mrs S is a 47 year old woman that presents to her GP. Her mother has suffered from osteoporosis and Mrs S wants advice on how she can help prevent osteoporosis.

Page 204: finals2009.wikispaces.com · Web viewPleural effusion which is secondary to hypoalbunimaemia Q4. What initial investigations would be implemented to confirm this diagnosis? Urine

4.21 Metabolic bone disease

By the end of Phase II students should be able to:

• recognise the circumstances where osteoporosis is likely to occur

• be able to give advice to patients about preventing osteoporosis and outline treatment

strategies

• recognise the circumstances when osteomalacia may occur

• initiate investigation of osteomalacia

• recognise the circumstances when hypercalcaemia may occur

• initiate the management of hypercalcaemia

204

Page 205: finals2009.wikispaces.com · Web viewPleural effusion which is secondary to hypoalbunimaemia Q4. What initial investigations would be implemented to confirm this diagnosis? Urine

4.22

4.22 Other conditions

By the end of Phase II students should be able to:

205

A 62 year old lady presents to her GP. She has been feeling non-specifically unwell for nearly 3 months. She has been suffering pain and stiffness in her shoulders, neck, hips and lumbar spine (limb girdle pattern) which is worse first thing in the morning upon waking and makes getting out of bed difficult. For the past 2 days she has suffered a severe constant headache affecting her right temporal scalp. She has also noticed tenderness in this same area.

Q1. What is the likely diagnosis?

Polymyalgia Rheumatica with concurrent Temporal Arteritis (Giant cell arteritis/Cranial arteritis).

2 marks

Q2. Which investigations should be carried out and what is the definitive diagnostic test for this condition?

FBC may demonstrate mild anaemiaESR characteristically very elevatedCRP raisedLFT’s – Serum alk phos and γ-glutamyltransferase may be raised

Definitive diagnostic test is Temporal Artery Biopsy 4 marks

Q3. How should this condition be initially managed?

High dose Corticosteroids PO 2 marks

Q3. Name 3 other conditions associated with vasculitis.

Rheumatoid ArthritisSLESclerodermaPolymyositis/DermatomyositisGoodpastures SyndromeInflammatory Bowel DiseaseParaneoplastic

Subacute Infective Endocarditis 3 marks

Page 206: finals2009.wikispaces.com · Web viewPleural effusion which is secondary to hypoalbunimaemia Q4. What initial investigations would be implemented to confirm this diagnosis? Urine

• recognise, investigate and initiate management of cranial arteritis

• recognise and refer cauda equina compression

• recognise progressive cervical myelopathy

• recognise common dermatological manifestations of rheumatic disease

206

Page 207: finals2009.wikispaces.com · Web viewPleural effusion which is secondary to hypoalbunimaemia Q4. What initial investigations would be implemented to confirm this diagnosis? Urine

3.1 & 5.2

Mr Bogg is an unfortunate 65 year old man who has attended the ward prior to a right

hemicolectomy for a longstanding colonic tumour. He has a longstanding medical

history. He has had diabetes mellitus for the past 48 years for which he is currently

using insulin and notes that he has good control and has no known complications. He

also suffers from hypertension that he diligently checks at home and at his GP’s and

notes is within the ‘normal range’. He is otherwise well on the day of admission.

(a) Given his medical history what ASA grade would you assign him and state what

features in his history suggest this particular grade. What does the ASA grading

system predict? (3 marks)

Class II: “a patient with mild to moderate systemic disease that does not limit the patient’s activities in any way”. Known hypertensive and diabetic both of which are well controlled and have no significant complications. Predicts perioperative mortality.

(b) Name 3 pre-operative investigations that would be appropriate in this patient (3

marks)

Bloods (FBC, U&E, Cr, BM, HbA1c)

PA CXR

12-Lead ECG

(c) What steps would take to optimise his pre-operative glucose levels. Give a detailed

account (exact numerical details are not required). (7 marks)

Liaise with surgeon and anaesthetist

Put patient first on list

Refer to local policy

Stop LA insulin the night before

Setup a sliding scale (50U of SA insulin with 50mL N/saline)

Check BM every hour and adjust sliding scale

Aim for a BM of between 7 – 11mmol/L during surgery

Continue sliding scale until px eats second meal – check BM every 2

hours

Switch back to normal regimen

On the day of his operation, Mr Bogg was transferred from the ward to the anaesthetic

207

Page 208: finals2009.wikispaces.com · Web viewPleural effusion which is secondary to hypoalbunimaemia Q4. What initial investigations would be implemented to confirm this diagnosis? Urine

room. Following pre-oxygenation and initial iv induction with propofol the medical

student was asked to ventilate him by bag and mask. He was having trouble maintaining a seal and therefore the anaesthetist asked him to put in an oral airway.

(d) How do you size a guedel airway in an unconscious patient? (1 mark)

An estimate of the airway size required is given by comparing the airway length with

the vertical distance between the patient’s incisor teeth and the angle of the jaw

(e) Describe the process of inserting an oral airway (2 marks)

Start with airway upside down

Place into oropharynx in this position, advance to the hard palate

Rotate the airway 180o

Fully insert until opening lies in front of teeth

(f) What is the main problem that one must warn the patient of during propofol

administration – especially if they are of an anxious disposition? (1 mark)

Often administered with local anaesthetic due to pain on injection

He was then paralysed prior to intubation. The anaesthetist used rocuronium to

facilitate this.

(e) What is the mechanism of action of rocuronium and how could its effects be

reversed if the surgeon broke a fingernail and decided to cancel the procedure. (2

marks)

Rocuronium is a non-depolarizing competitive neuromuscular blocking agent that

competes with acetylcholine for post-synaptic binding sites. It is commonly reversed

using an anticholinesterase such as neostigmine to overcome the block.

(f) Name 2 methods to confirm that the ETT was in the correct place. Give 4

complications of intubation. (6 marks)

To confirm correct placement either:

ventilate the chest and watch it rise symmetrically

auscultate over the stomach during ventilation and hope not to hear bubbles with

ventilation.

208

Page 209: finals2009.wikispaces.com · Web viewPleural effusion which is secondary to hypoalbunimaemia Q4. What initial investigations would be implemented to confirm this diagnosis? Urine

Complications:

(Any from below)

Oesophageal intubation

Right main-stem bronchus intubation

Gastric aspiration

Airway trauma

Dental trauma

Cervical cord injury

(g) How does one prevent gastric aspiration during induction of anaesthesia? How

does it work? What is the complication that may arise from forgetting to do this

(especially in a trauma situation)? Where would such a patient go in the post-operative

period? (4 marks)

Use cricoid pressure

Cricoid is the only complete ring of cartilage in the larynx therefore occludes

oesophagus with application of direct anterior pressure

Aspiration pneumonia

Send to ITU for monitoring and management

5.1 Pre-operative assessment for anaesthesia

By the end of Phase II students should be able:

• to order appropriately pre-operative blood tests, chest radiographs and ECGs.

• to recognise patients who have an increased risk of peri-operative morbidity and mortality

• to participate in the pre-operative work-up regimes for high risk patients

• to liaise with the anaesthetist in the management of a diabetic patient having surgery

5.2 General anaesthesia

By the end of Phase II students should be able to:

• recognise the properties of some of the commoner drugs used in anaesthesia and their side

effects

• perform venous cannulation competently

• recognise some of the complications which may occur during an anaesthetic including,

hypertension, bradycardia and tachycardia, myocardial infarction, aspiration of stomach

contents

• care for an unconscious patient

209

Page 210: finals2009.wikispaces.com · Web viewPleural effusion which is secondary to hypoalbunimaemia Q4. What initial investigations would be implemented to confirm this diagnosis? Urine

5.3

Question 1

A post-operative patient is in recovery.

State 3 risk-factors that increase the chance of post-operative nausea and vomiting.

(3 marks)

Female gender

Obesity

Young age

Positive history

Surgery to middle ear, posterior cranial fossa, intra-abdominally, hysterectomy

Severe pain

Administration of opioid analgesics, nitrous oxide (ketamine, isoflurane)

Hypoglycaemia, hypoxia, uraemia, hypotension.

Gastric dilatation e.g. from bag and mask ventilation

State 2 different anti-emetic drug groups and the generic name for 1 drug from each.

(2 marks)

Anti-histamine e.g. chlorphenamine, loratidine, cyclizine.

Dopamine antagonist e.g. metoclopramide, domperidone.

5HT3 antagonist e.g. granisetron, ondansetron.

Anti-cholinergic e.g. hyoscine, atropine

Where in the body can the receptors for these drugs be found?

(2 marks)

Histamine (H1) receptors, found mainly in the vestibular labyrinth.

Dopamine receptors are found in chemoreceptor trigger zone

5HT3 receptors are predominantly found in the GI tract.

Cholinergic receptors are found in the vestibular labyrinth and chemoreceptor trigger

zone

Question 2

Post-surgery a patient is sent to the ward on patient-controlled analgesia (PCA).

Name 2 advantages of PCA over infusion or i.m. injection.

(2 marks)

210

Page 211: finals2009.wikispaces.com · Web viewPleural effusion which is secondary to hypoalbunimaemia Q4. What initial investigations would be implemented to confirm this diagnosis? Urine

Increased patient satisfaction.

Increased ability to match the dose with the patient’s perceived pain level.

Reduced work-load for nursing staff.

Elimination of painful repeated i.m. injections.

Greater certainty of plasma opiate levels as given i.v.

Name 2 disadvantages of PCA over infusion or i.m. injection.

(2 marks)

Initial cost of pump

Not suitable for children under 5-7 years, confused, frail or elderly patients.

Potential for overdose remains.

What instructions would you give to the nursing staff regarding safe monitoring of

PCA?

(3 marks)

Regular observations, especially sedation score (neuro) and respiratory rate.

Pain score.

Written-up prescription for naloxone.

Check syringe volume with actual consumption – stop PCA if discrepancy.

Nurse on open ward rather than side room.

Prescription for breakthrough anti-emetic.

Later, you check the patient, but they are unresponsive, have a respiratory rate of 7

breaths per minute and pin-point pupils. What do you do?

(4 marks)

Call for help

Stop the PCA

ABCs giving O2 via bag-and-mask ventilation

Give naloxone

State the generic name and dose of the reversal agent given in this situation.

(2 marks)

Naloxone, 0.8-2mg i.v. every 2-3 mins until respiratory rate is adequate, to a

maximum of 10mg.

5.3 Postoperative problems

By the end of Phase II students should be able to:

211

Page 212: finals2009.wikispaces.com · Web viewPleural effusion which is secondary to hypoalbunimaemia Q4. What initial investigations would be implemented to confirm this diagnosis? Urine

• treat post-operative nausea and vomiting

• appreciate the likely causes and initiate management of post-operative confusion

• recognise and initiate management of post-operative respiratory failure

• understand the importance of effective post-operative analgesia

• manage i.v. patient controlled analgesia and epidural analgesia and their side

effects

• prescribe strong, intermediate and minor analgesics in the post-operative period

having regard to the operation and to appropriate doses, routes of administration,

side effects and contra-indications

• perform intravenous, intramuscular and subcutaneous injections competently

212

Page 213: finals2009.wikispaces.com · Web viewPleural effusion which is secondary to hypoalbunimaemia Q4. What initial investigations would be implemented to confirm this diagnosis? Urine

5.4

Question 1

Pulse oximetry is commonly used to monitor hospital patients.

Describe 2 advantages and 3 limitations of this technique.

(5 marks)

Advantages:

Small and portable

Continuous monitoring

Non-invasive

Simple to use

Fast results

Limitations:

Does not measure C02 levels

Carboxyhaemoglobin gives falsely high readings

Methaemoglobin gives falsely low readings

In respiratory failure a high FiO2 may give misleadingly high SpO2%

Inadequate pulse e.g. hypoperfusion, hypovolaemia, hypothermia, peripheral

vasoconstriction.

Excessive patient movement can disturb

Nail varnish, nicotine and henna can prevent reading

Methylene blue (thyroid surgery) gives false low readings

In anaemia the SpO2 may be high, but there may be inadequate tissue delivery of

oxygen

Below 10kPa, a small drop in PaO2 produces a large drop in saturation reading due

to the haemoglobin dissociation curve

Readings <70% are inaccurate (machines are calibrated against healthy human

volunteers – they don’t let them get this hypoxic apparently!)

Question 2

Name 2 vessels appropriate for introducing a central venous catheter.

(2 marks)

Internal jugular vein

Subclavian vein

Basilic vein at antecubital fossa (only 60% chance of success)

213

Page 214: finals2009.wikispaces.com · Web viewPleural effusion which is secondary to hypoalbunimaemia Q4. What initial investigations would be implemented to confirm this diagnosis? Urine

State 4 complications that can potentially arise from central venous catheterisation.

(4 marks)

Air embolus

Pneumothorax

Scepticaemia

Local infection at catheterisation site

Arterial puncture and haemorrhage

Venous thrombosis

Thoracic duct injury (left side)

Injury to brachial plexus, phrenic or recurrent laryngeal nerve

5.4 Monitoring

By the end of Phase II students should be able to:

• recognise the need for central venous cannulation and the associated complications

• measure a patients CVP using a water manometer

• recognise the need for pulmonary artery catheterisation and its uses

• use and interpret pulse-oximetry

• recognise the need for cardiac monitoring

214

Page 215: finals2009.wikispaces.com · Web viewPleural effusion which is secondary to hypoalbunimaemia Q4. What initial investigations would be implemented to confirm this diagnosis? Urine

5.5

While working as a house officer on the cardiology ward, Mr S a 75-year-old man with a

history of ischaemic heart disease, mistakenly arrives at the ward for his outpatient

appointment. While you are showing him to the outpatient department he collapses in

the corridor with central crushing chest pain and quickly becomes unresponsive.

Using simple manoeuvres you open his airway but find he is not breathing and has no

pulse. After calling for the resuscitation team you return to him to begin basic life

support until help arrives.

1) According to the 2005 resuscitation council guidelines, at what ratio should chest

compressions to rescue breaths be given to an adult? (1)

30:2

Help quickly arrives and a defibrillator is attached to assess his heart rhythm and IV access is

gained.

A) B)

C) D)

2) Assuming they are all pulseless, name the ECG rhythms A-D (4)

A) Pulseless electrical activity (PEA) / electromechanical dissociation (EMD)

B) Asystole

C) Ventricular fibrillation (VF)

D) Ventricular tachycardia (VT)

3) Mr S is found to be in the same rhythm shown in C. According to the advance life

support algorithm 2005 how should this be treated? (4)

1 x defibrillator shock

215

Page 216: finals2009.wikispaces.com · Web viewPleural effusion which is secondary to hypoalbunimaemia Q4. What initial investigations would be implemented to confirm this diagnosis? Urine

2mins of CPR (30:2)

Adrenaline 1mg IV each cycle

Reassess rhythm

4) What other rhythm is treated this way (use the letter A-D as above)? (1)

D

5) Name one drug other than adrenaline that can be used in order to correct rhythm C

and your answer to question 4 (1)

Amiodarone

6) Name four reversible causes of cardiac arrest (4)

Hypoxia

Hypovolaemia

Hypothermia

Hyper/hypokalaemia (metabolic)

Tension pneumothorax

Thromboembolism

Toxic

Tamponade, cardiac

Eventually the resuscitation attempts are successful and Mr S returns to sinus rhythm

but does not regain consciousness. He is taken to the intensive care unit due to his

poor condition. His condition continues to deteriorate and a decision is needed on

whether to attempt to resuscitate Mr S, should he experience anther cardiac arrest.

7) Under what circumstances is it appropriate to issue a do not attempt resuscitation

order (DNAR) in an incompetent / unconscious patient? (3)

When CPR will to restart the patient’s heart and breathing - Futile

When there will be no benefit gained by restarting the patient’s heart and breathing –

Patient will die now or next time they arrest

216

Page 217: finals2009.wikispaces.com · Web viewPleural effusion which is secondary to hypoalbunimaemia Q4. What initial investigations would be implemented to confirm this diagnosis? Urine

When the expected benefits of CPR are outweighed by the burdens – co-morbidity

will cause poor or no quality of life

8) Who is ultimately responsible for decision-making regarding resuscitation orders for

incompetent / unconscious patients? (1)

The consultant / GP in charge of the patients care (Where care is shared consensus

is needed)

The resuscitation council 2005 guidelines can be accessed here.

5.5 Cardio-pulmonary resuscitation

By the end of Phase II students should be able to:

• Diagnose and initiate management of asystole, ventricular

fibrillation, pulseless ventricular tachycardia and electro-mechanical

dissociation according to established guidelines

• Operate a defibrillator competently

• Discuss the ethical dilemmas surrounding resuscitation and how

they may be resolved

217

Page 218: finals2009.wikispaces.com · Web viewPleural effusion which is secondary to hypoalbunimaemia Q4. What initial investigations would be implemented to confirm this diagnosis? Urine

5.8

Miss S, a 20 year old women attends A&E and admits to taking an overdose of pills.

1. What essential questions must you ask at this point?

What have you taken? Including drugs and alcohol.

How much have you taken?

When did they take them?

Why did they take them?

2. She tells you that she has taken 16 paracetamol tablets, no alcohol, roughly an hour

and a half ago. What is your immediate management plan?

IV access

Bloods – FBC, U&E, LFT, Clotting/INR

Paracetamol levels taken at 4 hours post OD

ABG

Activated charcoal – according to local protocol

Senior advise

3. What is the specific antidote/treatment for paracetamol overdose?

N-acetyl cysteine (Trade name is Parvolex)

4. When would you decide to use it?

Refer to Paracetamol OD chart found in all A&E departments

Table shows two treatment lines for plasma-paracetamol concentration against time

(in hours) since the OD.

Most patients will receive N-aceyl cysteine if their plasma-paracetamol level is

above the normal treatment line.

5. There are two treatment lines on the paracetamol overdose chart, one for

normal/low-risk patients and one for high-risk patients. When would you use the high-

risk treatment line?

Malnourished patients e.g. anorexia, alcoholism, HIV positive

Patients taking enzyme-inducing drugs e.g. carbemazepine, phenobarbitone,

phenytoin, rifampicin, alcohol.

218

Page 219: finals2009.wikispaces.com · Web viewPleural effusion which is secondary to hypoalbunimaemia Q4. What initial investigations would be implemented to confirm this diagnosis? Urine

6. What would you like to ask Miss S to determine her suicide risk?

What happened today to make her take the OD?

Was the self-harm/attempted suicide premeditated or opportunistic?

Did she leave a note?

Did she intend to kill herself?

Does she regret her actions or would she do this again?

Any history of psychiatric disorders? E.g. depression, schizophrenia.

Previous suicide attempts?

Assess support network.

7. What are the long term consequences of paracetamol overdose?

Liver failure & encephalopathy

Renal failure

5.8 Acute poisoning

By the end of Phase II students should be able to:

• recognise the commoner inhalants and ingestants responsible for accidental and

deliberate poisoning

• initiate immediate resuscitation if appropriate

• participate in the management of the commoner poisonings

• make an assessment of the underlying social and psychiatric factors in deliberate self-

harm and the likelihood of a repetition

219

Page 220: finals2009.wikispaces.com · Web viewPleural effusion which is secondary to hypoalbunimaemia Q4. What initial investigations would be implemented to confirm this diagnosis? Urine

5.9

You are the Surgical HO on call. You are asked by the nursing staff to assess Mrs B,

as she has an increasing Mews Score. Mrs B had a right hemicolectomy two days ago

for a colonic carcinoma.

1. Define shock?

Circulatory failure resulting inadequate organ perfusion.

2. List four types of shock and a specific example, not including neurogenic shock.

Cardiogenic shock – post-MI

Hypovolaemic shock – bleeding from trauma, ruptured AAA, ruptured ectopic

pregnancy

fluid loss from vomiting, diarrhoea, burns, third

space losses

Anaphylatic shock – Type I hypersensitivity reaction

Septic shock – Endotoxins released from Gram negative bacteria e.g. E.coli

Endocrine shock – Addisonian crisis, hypothyroidism

Iatrogenic shock – from anaesthetics, antihypertensives

3. What signs would suggest shock in a patient? List six.

Systolic BP <90 mm Hg

heart rate/tachycardia

Capillary refill

respiratory rate

urine output/oliguria

Pallor

Faint/collapse

Confusion

Mrs B has the following observations:

Temp: 38.4C

Heart rate: 108bpm

BP: 92/74 mm Hg

O2 Sats: 97%

220

Page 221: finals2009.wikispaces.com · Web viewPleural effusion which is secondary to hypoalbunimaemia Q4. What initial investigations would be implemented to confirm this diagnosis? Urine

On examination she has warm peripheries and capillary refill is <2 seconds. The pulse

is rapid but bounding.

4. What type of shock is Mrs B likely to be suffering from?

Septic shock

5. What other observation would be useful to aid management?

Urine output

Fluid balance

6. What would be your immediate management plan?

IV access, if not already got it

IV fluid, colloid or crystalloid

Blood, urine and wound swab cultures

Antibiotics: Cefuroxine 1.5g TDS IV, Metronidazole 500mg QDS IV, Gentamicin

5mg/kg OD IV (blind treatment)

Senior review

5.9 Shock

By the end of Phase II students should be able to:

• recognise and initiate immediate investigation and management of hypovolaemic

shock

• recognise and initiate immediate investigation and management of septic shock and its

complications

• recognise the pathophysiological disturbances which accompany shock

• monitor and assess the response to treatment

221

Page 222: finals2009.wikispaces.com · Web viewPleural effusion which is secondary to hypoalbunimaemia Q4. What initial investigations would be implemented to confirm this diagnosis? Urine

5.11

Mrs D, an 80 year-old lady is brought to the Emergency department by her daughter

who is very concerned about her mother’s sudden onset of acute confusion. Her

mother lives alone independently but recently complained of a fever and has spent the

last few days in bed.

List 3 possible differential diagnoses for her acute confusional state (3)

Infection - LRTI

UTI

Septicaemia

Metabolic - Hypoglycaemia of hyperglycaemia

Hyponatraemia

Alcohol

Uraemia

Hypothyroidism or hyperthyroidism

CNS - Meningitis

CVA

Post-ictal

Drugs - Tranquillizers

Lithium, Digoxin

Benzodiazepines

You are unable to obtain a history from Mrs D. On examination Mrs D is very confused,

pyrexial 38ºC with a respiratory rate of 32 breaths/min, SATS 85% on air, BP 150/60

mmHg. Bronchial breathing is noted at the right lower lobe. Abdomen examination is

unremarkable.

What is the most likely diagnosis? (1)

LRTI

List 3 immediate investigations you wish to do. (3)

CXR

ABG

Blood cultures

Blood tests (urea, WCC, CRP)

Bm

222

Page 223: finals2009.wikispaces.com · Web viewPleural effusion which is secondary to hypoalbunimaemia Q4. What initial investigations would be implemented to confirm this diagnosis? Urine

An ABG comes back for you to interpret it

paO2 7 KPa

paCO2 4KPa

pH 7.47

HCO3- 20

BE 1

What does it show? (2)

Type I RF

What is your immediate management? (5)

High flow (60%) oxygen

IV access

IV fluids

IV antibiotics

Analgesia

Monitor ABGs regularly over the next few hours

Inform anaesthetist

Mrs D subsequently improves over the next few hours and is moved to a ward. What

potential complications could result from pneumonia? (2)

Lung abscess

Empyema

End of Question

Total 16 Marks

5.11 Acute confusional state

By the end of Phase II students should be able to:

• recognise and distinguish the common causes

• initiate investigation and management of the commoner causes

223

Page 224: finals2009.wikispaces.com · Web viewPleural effusion which is secondary to hypoalbunimaemia Q4. What initial investigations would be implemented to confirm this diagnosis? Urine

5.13:

Whilst on your respiratory rotation, you are called by the nurses to see a 26 year old

man, with a known history of asthma, because he is complaining of shortness of

breath! When you reach his bed he is agitated and confused. He is tachpnoeic (RR =

24). His most recent blood gas results are shown below.

 

ABG's:

 

pH = 7.36 kPa

pO2 = 7.8 kPa

pCO2 = 3.2 kPa

HCO3 = 24 mmol/l

BE = -2.0 mmol/l

 

SaO2 = 92% on air

Temperature = 38.1oC

 

1. What is your assessment of this patient?

 

ABG's show hypoxia. All other values are within normal parameters.

? Exaccerbation of asthma. Patient is suffering from Type 1 respiratory failure.

Causes include:

Pneumonia

Pulmonary oedema

Pulmonary embolism

Asthma

Emphysema

Fibrosing alveolitis

ARDS

 

 

2. What is your management?

 

A,B,C etc.....

Auscultate - wheezing?

Start highflow oxygen (35 - 60%) by face-mask to correct hypoxia

224

Page 225: finals2009.wikispaces.com · Web viewPleural effusion which is secondary to hypoalbunimaemia Q4. What initial investigations would be implemented to confirm this diagnosis? Urine

Salbutamol nebuliser

Position patient comfortably

Check hydration, pulse, BP, temperature etc.

Look for underlying cause of respiratory failure:

 

Order CXR

FBC; U and E's; CRP

Repeat ABG's.

Sputum and blood cultures (esp. if febrile)

Check medication and time of last dose.

Look up the BTS guidelines on step-wise approach to asthma management in adults (Beta 2

agonist, Inhaled steroid, Lk receptor agonist, Aminophylline etc).

 

Don't forget to get Senior help early, particularly if there is a chance his condition could

deteriorate! i.e. anaesthetists/ITU

 

Having done the above the patient improves so you go back to bed. However, 3 hours

later you are called back to the same ward because another patient has "gone off". You stroll merrily back!

On arrival, the patient is a 56 year old man with an 80 pack-year smoking history and a

known COPD sufferer. He is cyanosed, sleepy and has a bounding pulse. His most

recent ABG results are as follows:

 

pH = 7.32

pO2 = 7.1 kPa

pCO2 = 7.0 kPa

HCO3 = 26 mmol/l

BE = 4

 

SaO2 = 89% on 2l O2 via nasal cannulae

 

3. What is you assessment?

 

ABG's show Respiratory acidosis with Type 2 respiratory failure.

Type 2 respiratory failure is defined as Hypoxia (PaO2 < 8kPa) with Hypercapnia

(PCO2 > 6.5kPa). This is caused by alveolar hypoventilation with or without V/Q

mismatch. Causes include:

225

Page 226: finals2009.wikispaces.com · Web viewPleural effusion which is secondary to hypoalbunimaemia Q4. What initial investigations would be implemented to confirm this diagnosis? Urine

Pulmonary disease (COPD, asthma, pneumonia, pulm. fibrosis, obstructive sleep

apnoea)

Reduced respiratory drive (sedative drugs, CNS tumour, trauma)

Neuromuscular disease (cervical cord lesion, diaphragm paralysis, polio, Guillain-

Barre syndrome)

Thoracic wall disease (kyphoscoliosis, flail segment).

 

4. What is your management?Some debate on this one!

 

- Most consultants say......

A,B,C's as normal.

Start high-flow oxygen as hypoxia will kill you quicker than hypercapnia.

This is because high-flow oxygen will "knock-off" the patient's respiratory drive which

is driven by hypoxia instead of hypercapnia. (The reason for this is becasue the

patient has been suffering from a chronic resp. disease, resulting in higher-than-

normal blood CO2 levels. The central respiratory centre is therefore relatively

insensitive to CO2 levels, "resets" to a higher CO2 level and as a result hypoxia

drives respiration. By giving lots of O2, pO2 levels rise and respiration reduces. This

raises CO2 levels to dangerous levels and causes acidosis. In addition, hypoxia will

also occur due to the lack of respiration).

CALL ANAESTHETISTS/ITU CONSULTANT!!!

 

- The OHCM says......

Treat underlying cause

Controlled O2 therapy: start at 24%

re-check ABG's after 20 mins. If PCO2 is steady or lower, increase O2 to 28%. If

PCO2 has risen > 1kPa and the patient is still hypoxic, consider a respiratory

stimulant e.g. Doxapram 1.5 - 4 mg/min IV or assisted ventilation e.g NIPPV.

If this fails consider intubation and ventilation i.e. CALL ANAESTHETISTS/ITU

CONSULTANT!!!

 

Guide to colours of Venturi valves:

Venturi valve Flow rate Oxygen delivered colour (l/min) (%)

Blue                 2                     24

226

Page 227: finals2009.wikispaces.com · Web viewPleural effusion which is secondary to hypoalbunimaemia Q4. What initial investigations would be implemented to confirm this diagnosis? Urine

White               4                     28

Yellow              6                     35

Red                  8                     40

Green              12                    60

 

Which mask for which patient?

Nasal cannulas:

Patients with normal vital signs—for example, postoperative, slightly low oxygen

saturations, long term treatment with oxygen at home

Simple face masks and masks with a reservoir bag:

Higher concentrations of oxygen are required and controlled oxygen is not necessary

—for example, severe asthma, acute left ventricular failure, pneumonia, trauma, or

severe sepsis. (Masks should always be set to a minimum of 5 l/min because

significant rebreathing of CO2 can occur when exhaled air is not adequately flushed

from the mask)

Venturi masks:

Controlled treatment with oxygen required in people with chronic respiratory failure—

for example, COPD

 

5.13 Respiratory failure

By the end of Phase II students should be able to:

• make a rapid assessment and initiate management in the patient with acute respiratory

distress

• recognise a patient in respiratory failure

227

Page 228: finals2009.wikispaces.com · Web viewPleural effusion which is secondary to hypoalbunimaemia Q4. What initial investigations would be implemented to confirm this diagnosis? Urine

• assess the cause, severity and type of respiratory failure and initiate management

• interpret arterial blood gas results

• prescribe oxygen and oxygen delivery systems appropriately

228

Page 229: finals2009.wikispaces.com · Web viewPleural effusion which is secondary to hypoalbunimaemia Q4. What initial investigations would be implemented to confirm this diagnosis? Urine

6.1 + 6.2

Miss P.V Bleed presented to her G.P with long standing menorrhagia. She went on to

explain that recently she had been feeling tired all the time and on occasions becomes

quite breathless. On examination the G.P noted she was pale, tachycardic and had

spoon shaped nails. He decided to undertake some routine blood tests, one of these

being a full blood count.

Which 2 components of the full blood count are we most interested in when classifying

Miss P.V Bleeds anaemia? (1)

Haemaglobin

MCV

Miss P.V Bleeds full blood count results were as follows.

Haemoglobin 9.8g/dL

Mean Corpuscular volume (MCV) 72fl

Mean Corpuscular Hb (MCH) 23 pg

From these results, how would you classify Miss P.V Bleeds anaemia? (1)

Microcytic anaemia

Given her history and above blood results, Miss P.V Bleed is most likely to be suffering

from Iron deficiency anaemia as a result of her chronic blood loss.

List three other causes of Iron deficiency anaemia (3).

malabsorption

low dietary Fe intake

Pregnancy (increased requirement)

Hypothyroidism

chronic gastrointestinal bleed

What two following blood tests could you request the lab to run on Miss P.V Bleeds

blood related to her condition of Iron deficiency anaemia to confirm your diagnosis?

229

Page 230: finals2009.wikispaces.com · Web viewPleural effusion which is secondary to hypoalbunimaemia Q4. What initial investigations would be implemented to confirm this diagnosis? Urine

(2)

Serum ferritin

Serum Iron

What is the appropriate treatment for Miss P.V Bleed given her Hb level is 9.8 g/dL (1)

Oral Fe, ferrous sulphate, I.M Fe if unable to tolerate oral.

How would you initially manage her if the Hb level was 4.8 g/dL? (1)

Blood transfusion

List 3 hazards for this initial treatment of correcting a low Hb level (3).

Transfusion reactions (cross reaction between donor and recipient)

transmission of infection, Hep B, C, HIV

Circulatory overload leading to heart failure, electrolyte disturbances from large

volumes in transfusion.

Following her Gynae outpatient appointment Miss P.V Bleed’s menorrhagia was treated

and her iron deficiency anaemia corrected itself with the above treatments.

Unfortunately 6 years later she developed a gastric carcinoma and required a

gastrectomy.

After her operation she returned to her G.P with similar symptoms as before, pallor,

exhaustion and lethargy. Her Hb was 9g/dL.

How would the red cells appear on a blood film taken from Miss P.V Bleeds blood? (1)

Macrocytic

Based on her history, what form of anaemia is Miss P.V Bleed most likely suffering

from now? (1)

230

Page 231: finals2009.wikispaces.com · Web viewPleural effusion which is secondary to hypoalbunimaemia Q4. What initial investigations would be implemented to confirm this diagnosis? Urine

Pernicious anaemia.

What is required for Vitamin B12 to be adequately absorbed by the terminal ileum? (1)

Intrinsic factor

The Schillings test is often used to confirm a diagnosis of Vitamin B12 deficiency

anaemia, briefly describe this test (4)

Patient given a loading dose of 1000mg of Vit B12 intramuscularly, followed by

small doses of radioactive vit B12 and excretion is then measured in the urine.

Vitamin B12 malabsorption is corrected by giving intrinsic factor with labelled

vitamin in pernicious anaemia, but persists despite the use of intrinsic factor in

intestinal disease.

How would you wish to manage Miss P.V Bleed if the schillings test was positive for Vit

B12 deficency anaemia and her Hb was around 9 g/dL? (2)

IM hydroxycobalamin 3 times weekly for 2 weeks and then once every 3 months for

life.

Name 4 other causes of macrocytic anaemia.

Alcohol

Folate deficiency

Liver disease

Drugs , e.g. azathioprine

Reticulocytosis

hypothyroidism

231

Page 232: finals2009.wikispaces.com · Web viewPleural effusion which is secondary to hypoalbunimaemia Q4. What initial investigations would be implemented to confirm this diagnosis? Urine

6.1 Investigations

By the end of Phase II students should be able to:

• obtain a blood sample by venepuncture

• interpret a full blood count report

• explain to patients the nature and value of a bone marrow aspiration and biopsy

6.2 Anaemia

By the end of Phase II students should be able to:

• recognise the significance of a low haemoglobin taking into account the age and sex of

the patient

• distinguish the type of anaemia from the blood count and together with the clinical

information from the patient determine the likely cause

• evaluate the cause of iron-deficiency anaemia

• treat iron-deficiency anaemia

• distinguish between iron-deficiency anaemia, the anaemia of chronic disease and

thalassaemia trait and either treat or make appropriate referral

• distinguish between macrocytic and megaloblastic anaemia

• determine the cause of macrocytic anaemia clinically and with appropriate laboratory

investigation

• use laboratory investigation to identify the cause of megaloblastic anaemia and to

institute treatment with appropriate urgency

• initiate investigation of normochromic, normocytic anaemia and make appropriate

referrals

• recognise the possibility of and likely causes of haemolytic anaemia from laboratory

investigation and clinical evidence and make appropriate referrals

• recognise and institute management of acute complications of sickle cell anaemia

232

Page 233: finals2009.wikispaces.com · Web viewPleural effusion which is secondary to hypoalbunimaemia Q4. What initial investigations would be implemented to confirm this diagnosis? Urine

6.3

Miss F is an 18 year old female who presented with acute shortness of breath. On

further questioning she has a 3- 4 month history of malaise and night sweats which

has worsened over the last two weeks. She also complains of frequent nose bleeds,

heavy periods and easy bruising. On examination the positive findings are pallor, a

temperature of 380c, pulse of 105, BP: 135/79, respiratory rate of 18 per minute. You

suspect a Leukaemia.

A) Name four features you would expect to see on a blood test.

Any four from:

Anaemia

Leucopenia

Thrombocytopenia

Hyperuricaemia (due to break down of cells)

Reduced clotting times

B) Apart from a blood test, name four other investigations you would carry out.

Any four from:

Peripheral blood film

Bone marrow aspirate/trephine

Immunophenotyping (identify cell type by looking at cell surface)

Cytogenetics (to look for Philadelphia Chromosome which is found in Acute

Lymphoblastic and Chronic Myeloid Leukaemia).

Cytochemical studies (to identify cell types by looking at enzymes in cells).

Chest X-ray (to rule out lymphoblastic lymphoma which would cause a mediastinal

mass which would be identified on the chest x- ray).

C) As part of her treatment you give her a blood transfusion. Name two complications

of a blood transfusion.

Immunological complications:

Haemolytic transfusion reaction.

Non- haemolytic transfusion reaction (febrile reaction)

Alloimmunisation

233

Page 234: finals2009.wikispaces.com · Web viewPleural effusion which is secondary to hypoalbunimaemia Q4. What initial investigations would be implemented to confirm this diagnosis? Urine

Incompatibility

Transfusion related acute lung injury (TRALI)

Anaphylaxis (plasma protein incompatibility)

Non immunological complication:

Transmission of infection

Circulatory failure because of volume overload

Iron overload, with repeated transfusions

Electrolyte change (K+ particularly) with massive transfusion

Thrombophlebitis

Air embolism

D) The studies show she has acute lymphoblastic leukaemia and you begin

chemotherapy to induce remission. During chemotherapy there is a risk of Tumour

Lysis Syndrome. What is this and how can it be prevented?

It is the rapid necrosis of tumour cells due to the chemotherapy which results in

hyperuricaemia and raised potassium and phosphate levels. It can cause renal

failure due to the deposition of uric acid and calcium phosphate crystals in the renal

tubules. It is often exacerbated by a low intravascular volume.

It can be prevented by ensuring the patient has normal uric acid levels by prescribing

Allopurinol and by ensuring diuresis is established with I.V. fluids.

E) Apart from Tumor Lysis Syndrome name three short term and three long term

effects of chemotherapy:

Short term: (any three)

Myelosuppression

Nausea and Vomiting

Alopecia

Oral ulceration

Long term: (any three)

Infertility

Second malignancy

Cardiac Arrhythmias and cardiomyopathy

Pulmonary fibrosis

Cataracts

234

Page 235: finals2009.wikispaces.com · Web viewPleural effusion which is secondary to hypoalbunimaemia Q4. What initial investigations would be implemented to confirm this diagnosis? Urine

F) Leukaemic blast cells can infiltrate the brain and lungs resulting in coma and

respiratory failure respectively. If the blast cell count in the peripheral blood is very

high (> 100x 109/L) that patient may need leukophoresis. What is leukophoresis?

Blood is collected from a vein and centrifuged to remove the leukaemic cells. The red

blood cells and plasma are then returned to the patient via another vein. It prevents

sludging of blast cells in the capillary beds.

6.3 Haematological malignancies

By the end of Phase II students should be able to:

• recognise the possibility of a malignant disorder of the lympho-haemopoietic system on

clinical grounds and from the blood count

• make an appropriate estimate of the urgency of referral

• undertake investigations to confirm myeloma as a cause of a raised plasma viscosity

• describe to patients in outline the potential benefits and possible side effects of

radiotherapy, chemotherapy and hormonal therapy in lymphoma

235

Page 236: finals2009.wikispaces.com · Web viewPleural effusion which is secondary to hypoalbunimaemia Q4. What initial investigations would be implemented to confirm this diagnosis? Urine

6.4 (answers not provided for this question)

Clinical Scenario:

You receive the following lab data at the end of clinic and turn up these values:

Haematocrit: 49%

RBC; 6.4 x 106 microlitres

MCV: 76fL

WBC 14 000 per microlitre

Platelet count 480 000 per microlitre

The white cell differential is normal.

The patient is a middle aged woman who has come in for a yearly check up, she does

not smoke or drink. She appeared well and physical examination was normal.

1. Are these results normal?

2. Are these separate abnormalities or part of a patter

3. What separate abnormalities would cause similar results?

4. If you consider the data together, what disease comes to mind?

5. What would be your plan at this stage?

The patient returned in 2 months and re examination revealed a spleen tip.

6. What is your thought now?

This same patient returns with a swollen left leg. Doppler studies have revealed she

has a femoral vein thrombosis. Her laboratory results return as follows:

HCT: 50%

WBC: 18 000 per microlitre

Platelet count: 850 000 per microlitre

MCV: 72 fL

7. There are many causes of venous thrombosis. Does any possibility seem

obvious from this data?

8. What treatment would you offer her?

6.4 Polycythaemia

236

Page 237: finals2009.wikispaces.com · Web viewPleural effusion which is secondary to hypoalbunimaemia Q4. What initial investigations would be implemented to confirm this diagnosis? Urine

By the end of Phase II students should be able to:

• recognise an abnormally high haemoglobin

• use clinical examination and a full blood count to determine the cause

• investigate and refer appropriately

• participate at an appropriate level in venesection treatment

237

Page 238: finals2009.wikispaces.com · Web viewPleural effusion which is secondary to hypoalbunimaemia Q4. What initial investigations would be implemented to confirm this diagnosis? Urine

6.5 (answers not provided for this question)

A 12-year-old girl visits the GP following concerns about her first menstrual period.

The flow was very heavy and persisted for two weeks. On further discussion she

reported having always bruised easily and suffered with nose bleeds as a young child.

What other information should be sought if a bleeding disorder is suspected?

You suspect this is a congenital bleeding disorder. What blood tests will you need to

carry out for the haematologist?

If she is diagnosed with a bleeding disorder what 3 interventions should her GP avoid?

Bleeding disorders are rarely congenital and are more commonly acquired, complete

the following table regarding common disease states and the coagulation deficit they

cause:

Disease state Pathophysiology

Malabsorption Vitamin K deficiency

Decreased synthesis of coagulation factors

Bacterial septicaemia

Thrombotic disorders:

Mr RB is a 58-year-old businessman who has come to see his GP with some pain and

swelling in his left calf. He has recently been on a long haul flight to New York and is

worried about deep vein thrombosis (DVT)

What else do you need to ask about in the history?

If you suspect a DVT what would you look for on examination?

After a full history and examination are carried out Mr RB is sent for investigations that

confirm a DVT and anticoagulation therapy is commenced.

238

Page 239: finals2009.wikispaces.com · Web viewPleural effusion which is secondary to hypoalbunimaemia Q4. What initial investigations would be implemented to confirm this diagnosis? Urine

As the F1 on call you are asked to start Mr RB on warfarin, what dose will Mr RB be

started on, and how will the treatment be monitored?

What else does Mr RB need to know about starting anticoagulation therapy?

What is the target international normalised ratio for treatment of a one off DVT or

pulmonary embolism?

Mr RB returns to the GP after a minor episode of bleeding following a tooth extraction

during the treatment period, the results of a blood test show an INR of 5, he is also

complaining of bleeding gums. What 3 measures are recommended management

strategy for this type of patient?

6.5 Disorders of bleeding and thrombosis

By the end of Phase II students should be able to:

• recognise a bleeding tendency on history and examination

• manage at an appropriate level an acute haemorrhagic state

• interact appropriately with the haematologist in the management of patients with

chronic problems of haemostasis

• recognise the possibility of a pro-thrombotic state on clinical grounds and make an

appropriate referral

• initiate and monitor anticoagulant therapy with warfarin and heparin according to

published guidelines

• counsel patients on anticoagulant therapy

• recognise over-anticoagulation on clinical and laboratory grounds and be able to initiate

management after appropriate discussion

239

Page 240: finals2009.wikispaces.com · Web viewPleural effusion which is secondary to hypoalbunimaemia Q4. What initial investigations would be implemented to confirm this diagnosis? Urine

6.8

Jane Smith is a 52 year old lady receiving chemotherapy for cancer. She has

undergone 2 cycles with an interval of 6 weeks between them.

She finished the latest chemotherapy 2 weeks ago.

Reviewing her blood results it is clear that she has the following abnormalities:

Anaemia

Thrombocytopaenia

Leucopenia

Q1) what has happened and why?

EXPERIENCING BONE MARROW FAILURE ?SECONDARY TO

CHEMOTHERAPY.

Q2) what are the risks associated with this picture?

INCREASED BLEEDING, INCREASED SUSCEPTIBILITY TO INFECTION

Q3) how do chemotherapy drugs work?

AFFECT DNA SYNTHESIS OR REPAIR

PROMOTES CELLULAR APOPTOSIS

The rest time between the cycles of chemotherapy allow the normal cells such

as bone marrow and the GI tract to recover from the toxicity.

Q4) what precautions should be taken (haematological) before commencing

chemotherapy?

REVIEW PERIPHERAL BLOOD FILM

Q5) what can be given to accelerate the recovery of the white cell count after

chemotherapy

GROWTH FACTOR G-CSF

240

Page 241: finals2009.wikispaces.com · Web viewPleural effusion which is secondary to hypoalbunimaemia Q4. What initial investigations would be implemented to confirm this diagnosis? Urine

Q6) A cytotoxic drug (such as Alkylating drug) when used in combination with

extensive irradiation, can result in what possible haematological disorder?

Increased incidence of ACUTE NON-LYMPHOCYTIC LEUKAEMIA

Q7)Give two factors that can increase the incidence of blood dyscrasias in a patient?

INREASING AGE

DOSE OF DRUG

DURATION OF DRUG USE

6.8 Drug interactions

By the end of Phase II students should be able to:

• recognise a haematological problem as being a possible consequence of concomitant

drug therapy especially in patients on chemotherapy

• recognise patients who are at risk of severe sequelae to drugs because of underlying

haematological diseases

241

Page 242: finals2009.wikispaces.com · Web viewPleural effusion which is secondary to hypoalbunimaemia Q4. What initial investigations would be implemented to confirm this diagnosis? Urine

7.2 & 7.3

Barry, a previously well sixty two

year old butcher from Barton on

the Beans presents with a 3 day

history of swollen ankles and

shortness of breath. He is a

poorly controlled diabetic and has

been so for at least 18 years, and

has poorly controlled

hypertension. During his work up

he has the above PA x-ray taken

and an ECG.

1. Give 3 pathological findings you can see on this x-ray?

Cardiomegaly: The cardio thoracic ratio is greater than 50%.

Pulmonary Oedema: Bat wing appearance (Caused by the initial narrowing of the

lower lobe vessels and widening of the upper lobe vessels).

Pulmonary Effusion (blunting of the costophrenic angles)

*** Also to look for in real life are KERLEY B LINES: 1-2 cm long horizontal

lines which meet the pleura at right angles. They are usually seen up the

side of the lungs beginning at the costophrenic angle. They are due to

interlobular lymphatics which have been distended by fluid or tissue

classically secondary to left ventricular failure or mitral stenosis. ***

2. Why is it important to know if this is a PA or AP film and which is most useful in this

case?

242

Page 243: finals2009.wikispaces.com · Web viewPleural effusion which is secondary to hypoalbunimaemia Q4. What initial investigations would be implemented to confirm this diagnosis? Urine

When a large heart shadow isn’t necessarily cardiomegaly: In an AP x-rayIn expiration:A pregnant patient.One with ascites.An obese patient A disobedient patient

In these cases patients may not be able to take a full inspiration which might make the heart look bigger than it actually is. Eight to nine posterior ribs showing is a good inspiration.

When x-rays enter the chest anteriorly and are detected posteriorly, the heart, which

is anterior in the chest, casts a bigger shadow than in PA films. PA films are thus more

accurate in the assessment of cardiomegaly.

3. Considering the history, what are the likely causes each of the 3 pathological

findings?

Cardiomegaly: Caused by hypertrophy of the myocardium as a result of years of

uncontrolled hypertension

Pulmonary Oedema: Caused by fluid leaking into the interstitium due to ↑

hydrostatic pressures as a result of left ventricular failure and pulmonary venous

congestion.

Pleural Effusion: A Transudative pleural effusion also caused by raised hydrostatic

pressure as above.

243

Page 244: finals2009.wikispaces.com · Web viewPleural effusion which is secondary to hypoalbunimaemia Q4. What initial investigations would be implemented to confirm this diagnosis? Urine

One of WMS’s medical students suggested to the Consultant cardiologist that an ECG

might be useful. Below is the patient’s ECG [Sorry the ECG does not fit the case perfectly;

I couldn’t find one that did]

4. Where and what are the abnormalities, and hence where and what is the cause?

Bradycardia of 50 bpm (Often associated with MI)

S-T segment changes: ST elevation in leads II, III, and aVF suggesting inferior wall

MI.

Reciprocal changes (ST segment depression) in leads V1 and V2.

In addition there is ST elevation in lead V6 suggesting lateral wall involvement.

Lateral infarction produces changes in leads I, avL and V5/6.

T Waves: Tall, (>75% of R waves) especially in Lead III and AVF which can be

caused by myocardial ischemia.

Inverted T waves in V1, V2 V3

These changes suggest an infarction affecting the inferior aspect of heart.

5. What is the likely diagnosis? Be specific about where in the heart the problem is.

244

Page 245: finals2009.wikispaces.com · Web viewPleural effusion which is secondary to hypoalbunimaemia Q4. What initial investigations would be implemented to confirm this diagnosis? Urine

Inferior lateral MI (Silent)

6. From your basic understanding of ECG’s, explain why is there S-T depression in

leads V1 and V2, and S-T elevation in leads II, III and aVF?

The ST depression is basically S-T elevation when viewed from the other side of the

heart. S-T Elevation represents MI in the inferior aspect of the heart. When viewed

from the V1 and V2 leads, the opposite picture is seen.

E

7. Why might our patient not have any chest pain despite the likely diagnosis?

A long term Diabetic, especially one who is poorly controlled may have diabetic

neuropathy. The damage to nerves caused by hyperglycaemia may mask the usual

symptoms of infarction, causing a silent MI.

Florence, a 72 year old women with a 15 year history of COPD presents to you with an

acute exacerbation of shortness of breath. She also has fevers and rigors and pain

associated with breathing and coughing. Her cough is productive of rusty coloured

sputum.

She has a chest x-ray

shown below:

8. What can you see?

How does this support

the likely diagnosis?

245

Page 246: finals2009.wikispaces.com · Web viewPleural effusion which is secondary to hypoalbunimaemia Q4. What initial investigations would be implemented to confirm this diagnosis? Urine

The right upper lobe (RUL) is showing an increased density.

There are linear lucencies in the right upper lobe (air bronchogram).

This is due to consolidation consistent with a lobar pneumonia.

Explanation: On a normal CXR, we can visualize the air-filled trachea, mainstem bronchi

and initial portions of the lobar bronchi. Further branches should only be seen with the

injection of contrast during a bronchogram. When air can be visualized in the more

peripheral intrapulmonary bronchi, this is known as the ‘air-bronchogram sign’. This

abnormality is usually caused by an infiltrate/consolidation that surrounds the bronchi.

Dave Is a 66 year old man who has been getting progressively more breathless over a

period of years. He is a life long non smoker, is phobic of pigeons and only tried slate

mining once for a few minutes as a teenager (and he did not inhale). He has no

medical history except for long standing and severe rheumatoid arthritis.

9. Describe what you can see in

this x-ray that may be related to

Dave’s breathlessness. State in

general terms the diagnosis:

Fine nodular and streaky

linear shadowing

(reticularnodular pattern)

which is worse in the lung

basis. Poor definition of

cardiac border (Due to

adjacent lung fibrosis)

Fibrosis

Blunted costophrenic angle indicative of pleural effususion.

246

Page 247: finals2009.wikispaces.com · Web viewPleural effusion which is secondary to hypoalbunimaemia Q4. What initial investigations would be implemented to confirm this diagnosis? Urine

A spirometry is requested to investigate Dave’s lung function.

A Leicester medical student who is writing up Dave for a portfolio intercepts the results

and finds that Dave’s FEV1:FVC ratio is 0.88. Having a vague recollection of the

importance of the ratio being over 0.70 He asks you if he can tell Dave the good news,

that all is well.

10. What do you say? Explain your answer, especially the relevance of a ratio being >

than 0.75.

No you can’t. A ratio of >0.75 indicates that there is no obstructive airway disease, so

we can only rule out diseases such as COPD and Asthma. To interpret the results

fully you will need the rest of the results.

11. When you get the results you find that Dave’s Vital capacity is only 55% of the

predicted for his age and height. What can you conclude about the pattern of lung

disease now?

This is a restrictive pattern of disease. The vital capacity is reduced by fibrosis, and

the FEV1: FEV ratio is normal because there is no obstruction.

E

12. Years later Dave presents with worsening breathlessness and swollen ankles.

What is the likely cause?

Dave’s rheumatoid related lung fibrosis has caused right heart failure (cor-

pulmonale). Pulmonary vasoconstriction due to alveolar hypoxia or blood acidemia

and/or anatomic compromise of the pulmonary vascular bed in lung disease

eventually causes pulmonary hypertension, which eventually causes right heart

failure.

A women presents to A+E acutely unwell: Her arterial blood gases are measured, and

the results are shown below:

Normal Values

247

Page 248: finals2009.wikispaces.com · Web viewPleural effusion which is secondary to hypoalbunimaemia Q4. What initial investigations would be implemented to confirm this diagnosis? Urine

pH 7.25 7.35-7.45

pCO2 kPa 7.12 4.7-6

pO2 kPa 9.1 >10.6

HCO3 mmol/l 25 22-28

13. What do they indicate? (State respiratory/metabolic alkalosis/acidosis)

Respiratory acidosis

14. Suggest a Cause:

Severe asthma

COPD

Decreased GCS (CVA)

19 year old Sarah presents to you, her GP, with shortness of breath. She has had an

irritating cough at night for a month. You decide to get her to do a peak flow.

15. Explain how you would explain to her what to do:

Make sure the peak flow dial is at zero. Stand up if possible and take in as deep a

breath as you can. Then place your mouth around the mouthpiece of the meter and

form a tight seal with your lips. Blow out through your mouth as hard and fast as you

can until you can not blow out any longer. This will move the main flow indicator up

the scale and the number listed beside the indicator is your peak flow.

While chatting to Sarah she tells you that she recently spent 6 months working and

travelling in the Indian sub continent. She stayed with a family for three months in a

remote village. She left in a rush and burnt all her belongings when she found out that

2 of the adults had TB, and she is worried she has caught it. Sarah has had all her due

vaccinations.

16. Would a tuberculin test such as the mantoux test be useful in ruling out TB

248

Page 249: finals2009.wikispaces.com · Web viewPleural effusion which is secondary to hypoalbunimaemia Q4. What initial investigations would be implemented to confirm this diagnosis? Urine

infection? Explain.

No. If she has had a BCG vaccination she should react to the tuberculin protein

regardless of whether she has contracted TB. [Incidentally, the vaccination only

provides 80% protection, and only for the first 15 years after the vaccination, so she

could have TB despite having a BCG]

E.

17. What would be the most useful investigation to rule out pulmonary TB, especially

in someone with a non productive cough?

Chest X-ray. (In TB Typically shows patchy or nodular shadows in the upper lobes,

loss of volume and fibrosis with or without cavitation) An abnormal chest x-ray is often

found in asymptomatic patients, but the reverse is very rare.

Alan, a 56 year old Social Science lecturer presents with central crushing chest pain on

exertion. The pain only comes when he is exercising and has been getting slightly

worse over a period of 2 years. He smokes 20/day and has a 5 year history of

hypertension, which he attributes to a particularly stressful module he teaches at his

university’s medical school.

18 Which two important investigations would you do at this early stage?

ECG

Cardiac Enzymes

Both these tests were normal. His blood tests are normal except cholesterol which

comes back as 7.9 mmol/l.

19. What other non-invasive test would you like to do?

Stress electrocardiography.

249

Page 250: finals2009.wikispaces.com · Web viewPleural effusion which is secondary to hypoalbunimaemia Q4. What initial investigations would be implemented to confirm this diagnosis? Urine

20. How would you explain this test to Alan?

Stress electrocardiography is an measurement of your hearts electrical activity taken

while you are exercising. The purpose is to analyse the changes in your hearts

electrical pattern as it does more work. Some patterns which indicate a lack of

oxygen reaching your heart are not visible until the heart is stressed.

Below is Alan’s stress ECG

21. Is the

exercise test

positive or

negative?

Explain.

There

is S-T

segment depression, indicating a positive exercise test and probable IHD. (5% may

not actually have IHD). In particular the segment of S-T depression is flat or even

slightly down sloping which is much more likely to be due to IHD than up sloping S-T

depression (see Oxford Hand book).

22. What are Alan’s risk factors for IHD?

Smoking,

Hypertension,

Hyperlipidaemia,

sex,

age,

250

Page 251: finals2009.wikispaces.com · Web viewPleural effusion which is secondary to hypoalbunimaemia Q4. What initial investigations would be implemented to confirm this diagnosis? Urine

stress.

23. Alan is diagnosed with Angina. What would you prescribe for him to take when he

experiences the same chest pain on exertion again.

Sublingual glyceryl trinitrate tablet (0.5 mg) or GTN spray.

24. In the longer term how would you treat him? Give life style and medical

management.

Weight loss

Stop Smoking

Take gentle exercise

Diet (low fat, low salt)

Relaxation

Decrease alcohol (if appropriate)

Medical:

Asprin (75-150mg /24hrs PO unless contraindications)

Beta blockers (atenalol 50-100mg/24hr PO)

Nitrates (GTN spray or sublingual tablets up to every ½ hour)

(as prohylaxis 10-30mg oral nitrates BD such as isosorbide

mononitrate)

Calcium antagonists (amlodipine 5mg/24hr, diltiazem-MR 90-180mg/12 hr PO)

Statin (Aim to reduce cholesterol to <5mmol/l)

25. Alan is scheduled for coronary angiography. Explain to him what this entails and

how it may benefit him.

This involves insertion of a catheter into the heart via a blood vessel in your leg.

Contrast can be introduced into the coronary allowing visualisation of the anatomy -

and thus patency - of the coronary arteries.

Obstructed blood vessels can be unblocked in various ways in the same procedure if

appropriate.

251

Page 252: finals2009.wikispaces.com · Web viewPleural effusion which is secondary to hypoalbunimaemia Q4. What initial investigations would be implemented to confirm this diagnosis? Urine

Objectives:

7.2 Investigations

By the end of Phase II students should be able to:

• recognise the basic radiological features of cardiac enlargement

and pulmonary oedema and relate them to the underlying

pathophysiology

• perform an ECG

• understand the basic principles of electrocardiography and interpret

major abnormalities.

• be able to explain stress electrocardiography, echocardiography,

and coronary and femoral angiography to a patient.

• recognise the basic radiological features of consolidation, pleural

fluid, lung collapse, carcinoma and lung fibrosis, seeking advice

where necessary and relate them to the underlying pathology

• obtain and interpret a spirograph and recognise the changes in

obstructive and restrictive lung disease

• interpret blood gas abnormalities

• use and explain the use of a peak flow meter

• interpret tuberculin tests

252

Page 253: finals2009.wikispaces.com · Web viewPleural effusion which is secondary to hypoalbunimaemia Q4. What initial investigations would be implemented to confirm this diagnosis? Urine

7.3 Angina pectoris

By the end of Phase II students should be able to:

recognise angina

initiate investigations to confirm the diagnosis and assess the

severity of underlying coronary artery disease

recognise the risk factors in individual patients

initiate immediate management

253

Page 254: finals2009.wikispaces.com · Web viewPleural effusion which is secondary to hypoalbunimaemia Q4. What initial investigations would be implemented to confirm this diagnosis? Urine

7.3

Daisy, an 80 year old lady presents to her GP following a few episodes of chest pain

she has experienced whilst out walking. This is particularly noted on colder days and

the pain is relieved by sitting on a nearby park bench.

The GP informs her that she may have a condition known as ‘Angina Pectoris’.

Question> What is the definition of Angina pectoris (also known as stable angina)?

Central chest pain (caused by myocardial ischaemia) which is brought on by exertion

and relieved by rest.

Question> The GP mentions that a cold environment, exercise, eating a large meal and

emotional stress can play a role in initiating an angina pectoris attack. As well as these

triggers, list 6 risk factors associated with coronary artery disease?

Hypertension

diabetes mellitus

smoking

hyperlipidaemia

high BMI

family history

exercise (lack of)

age, sex of the individual

ethnicity

socio-economic status

alcohol.

Question> The GP after taking a detailed history, begins an examination (with a

chaperone). In particular, he looks for signs of anaemia, thyrotoxicosis and

hyperlipidaemia. He then auscultates the chest for heart sounds. In particular, he

listens for murmurs associated with aortic stenosis and aortic regurgitation.

What role would aortic stenosis play in someone with angina pectoris?

If the individual exerts themselves, the cardiac output would increase. However, if

there is an obstruction to left ventricle emptying, then the coronary arteries will not

receive the perfusion they are now requiring with exertion, thus exacerbating angina

254

Page 255: finals2009.wikispaces.com · Web viewPleural effusion which is secondary to hypoalbunimaemia Q4. What initial investigations would be implemented to confirm this diagnosis? Urine

pectoris.

Can you draw the features of an aortic stenosis murmur on this diagram?

HS I----------II------------I

HS I111111111II------------I

(ejection pan-systolic murmur)

The GP refers Daisy for an ECG.

Question> Name 2 positive findings on an ECG that would indicate Daisy was suffering

with an acute attack of angina pectoris?

ST depression, T wave inversion and T wave flattening.

Question> Name 2 positive findings that would indicate previous ischaemic heart

disease (IHD)?

Left bundle branch block

pathological Q waves

left ventricular hypertrophy (i.e. left axis deviation, left ventricular strain, large R

waves in V5 and large S waves in V2).

Question> The GP refers Daisy for an Exercise Tolerance Test (ETT). The cardiologist

reviews the ETT-ECG for any of the ECG changes aforementioned. If there are no

positive ECG changes and no symptoms experienced whilst on the treadmill, does this

mean Daisy does not have angina pectoris?

No. Angina pectoris is a clinical diagnosis and the ETT is only 70% sensitive and

80% specific. The diagnosis of angina pectoris is a clinical one, derived after a

combination of history, examination and investigations.

With a normal ETT, the GP wants to refer Daisy for a diagnostic Coronary angiography,

where therapeutic treatment may also occur; PTCA (Percutaneous transluminal

coronary angioplasty i.e. ballooning) and/or a stent may be inserted. Depending on the

results, a distant option is a coronary artery bypass graft (CABG).

Question> In the meantime, the GP puts Daisy on a β-antagonist. What is the

255

Page 256: finals2009.wikispaces.com · Web viewPleural effusion which is secondary to hypoalbunimaemia Q4. What initial investigations would be implemented to confirm this diagnosis? Urine

mechanism of action in angina pectoris?

A β antagonist acts on the β1 receptors in the blood vessels and heart to reduce heart

rate (negative chronotropic effect) thus allowing greater filling of the coronary arteries

during diastole. β antagonists also reduce contractility of the ventricles (negative

inotropic effect) and thus reduce the oxygen demand on the heart. Non-

cardioselective β antagonists will also act on β2 receptors in the lungs and cause

broncho-constriction.

Question> Name two other classes of medication that are used to relieve symptomatic

angina pectoris?

Calcium receptor antagonists

nitrates

Potassium receptor agonist (e.g. Nicorandil).

Extended Question> Name 2 additional types of angina (other than angina pectoris)

and state their definition?

Crescendo angina. The amount of effort to invoke symptoms decreases with time,

although symptoms do not occur at rest.

Unstable angina. Symptoms appear unpredictably either with minimal exertion or at

rest.

Nocturnal angina. Symptoms appear at night and can awake the patient.

Decubitus angina. Onset of symptoms occurs when the patient is lying down.

The GP also notes that treatment will be a holistic one with an aim of reducing all

cardiovascular risk factors.

7.3 Angina pectoris

By the end of Phase II students should be able to:

• recognise angina

256

Page 257: finals2009.wikispaces.com · Web viewPleural effusion which is secondary to hypoalbunimaemia Q4. What initial investigations would be implemented to confirm this diagnosis? Urine

• initiate investigations to confirm the diagnosis and assess the severity of underlying

coronary artery disease

• recognise the risk factors in individual patients

• initiate immediate management

257

Page 258: finals2009.wikispaces.com · Web viewPleural effusion which is secondary to hypoalbunimaemia Q4. What initial investigations would be implemented to confirm this diagnosis? Urine

7.4

1) A) 65 years old Mr. E presented with acute onset of central crushing chest pain 2

hours ago. He said that the pain came on suddenly while he was sitting and

reading a book and since then the pain persisted. He looked generally unwell but

otherwise was not breathless. Apart from acute coronary syndrome (ACS), list 4

other possible causes for his symptoms:

(any 4)

Pericarditis

Dissecting aortic aneurysm

Pulmonary embolism

Gastroesophageal reflux

Musculoskeletal pain

Pancreatitis

Perforated peptic ulcer

B)

Hopefully you know you will need to do a 12 lead ECG. What ECG patterns would

indicate ST elevated myocardial infarction (STEMI) that fits the criteria for

thrombolysis?

More than 1 mm elevated ST segment in 2 or more contiguous limb leads, or

More than 2 mm ST elevated in two or more contiguous chest leads.

C) You have diagnosed him as having ST elevated myocardial infarction (STEMI) and

decided to prescribe thrombolytic drugs. List five contraindications to the prescription

of thrombolytic drugs.

(any 5 from below)

History of haemorhagic stroke at anytime in the past

Active or recent gastric ulcer or any active internal haemorrhage

Suspected aortic dissection

258

Page 259: finals2009.wikispaces.com · Web viewPleural effusion which is secondary to hypoalbunimaemia Q4. What initial investigations would be implemented to confirm this diagnosis? Urine

Recent major surgery with the last 6 weeks e.g laparotomy

Head injury

Previous allergic reaction to fibrinolytic agents

Non haemorrhagic stroke within the past 1 year

Previous streptokinase exposure

Pregnancy or post partum

C) what other immediate investigations will you do for Mr. E ?

Bloods- FBC, U&Es, LFTs, D- dimer, glucose, lipid profile, cardiac enzymes (CK,

Troponin T 12 hours after the onset of chest pain)

Chest xray

ABG

Later management( arrange Echo)

D) At the mean time, you will need to control his pain. what analgesia, apart from GTN,

will you use, what dose and what route?

Diamorphine IV 2.5-10mg

C)

C) what are the three factors that might contribute to thrombus formation in Mr. E’s

coronary artery?

Virchow’s triad:

Damage in the intimal surface of the vessel

changes in the pattern of blood flow

hypercoagubility

D) Few hours later Mr. E had a cardiac arrest and you were called in by the nurses. The

cardiac nurses were doing the chest compression and a Guedal airway was in place.

Another nurse now showed you the following ECG pattern. What is your diagnosis?

What do you do next?

259

Page 260: finals2009.wikispaces.com · Web viewPleural effusion which is secondary to hypoalbunimaemia Q4. What initial investigations would be implemented to confirm this diagnosis? Urine

Diagnosis: ventricular fibrillation

Attach defibrillator and defibrillate (joules depending on the machines) and check

pulses in between.

Give adrenaline every 3 mins accordingly, IV 1 mg.

7.4 Acute myocardial infarction

By the end of Phase II students should be able to:

• recognise acute myocardial infarction and use appropriate investigations to confirm the

diagnosis

• act appropriately to ensure that those patients likely to benefit receive thrombolysis as

quickly as possible

• control the pain of myocardial infarction

• recognise ventricular fibrillation and carry out immediate management

• recognise the need for further active management in the medium to long term

• describe the features of the underlying pathophysiological changes which can be used

to develop treatment and prevention strategies

260

Page 261: finals2009.wikispaces.com · Web viewPleural effusion which is secondary to hypoalbunimaemia Q4. What initial investigations would be implemented to confirm this diagnosis? Urine

A fifty-five year old man, Mr. Askew, presents in A&E with sudden onset of severe chest

pain persisting for one hour. He describes his chest pain as “crushing”, with no

radiation. He admits to having similar chest pains lasting for around 15 minutes over

the last 5 months. These pains usually occur when he is exercising and are relieved by

rest. Rest does not relieve his chest pain today. He feels sweaty, breathless and

nauseous. He does not smoke.

1 What features of the history suggest a cardiac origin for his chest pain?

Central, crushing chest pain with sweating, breathlessness and nausea.

2 What chest pain features would be suggestive of aortic dissection?

Sudden tearing chest pain, may radiate to the back.

3 Give two likely cardiac causes of Mr. Askew’s chest pain.

Unstable angina.

Acute myocardial infarction.

4 Which pathological process commonly underlies these two diagnostic possibilities?

Thrombus formation on an atherosclerotic plaque.

5 What biochemical markers would you assay to distinguish between your two

diagnostic possibilities given in question 3?

Cardiac enzymes – creatine kinase (MB), aspartate transferase (AST), lactate

dehydrogenase (LDH).

Cardiac troponins (troponin T and/or I).

6 Mr. Askew’s ECG shows ST elevation and development of pathological Q waves in

leads V1-6, I and aVL. What diagnosis do these ECG findings suggest?

Acute anterior myocardial infarction.

7 What would you expect to find at cardiac catheterisation in this patient?

Lesion causing blockage in/narrowing of left anterior descending coronary artery.

8 Mr. Askew is given 300mg oral aspirin. What is the mode of action of aspirin?

Irreversible acetylation of platelet cyclo-oxygenase enzyme, preventing thromboxane

A2 formation, leading to inhibition of long-term platelet activity/aggregation.

261

Page 262: finals2009.wikispaces.com · Web viewPleural effusion which is secondary to hypoalbunimaemia Q4. What initial investigations would be implemented to confirm this diagnosis? Urine

9 Give the four phases of clot formation.

Vasoconstriction

Platelet activation

Clotting

Clot lysis and scarring

10 Name three other substances you would prescribe, in addition to aspirin, as part of

Mr. Askew’s immediate management.

Oxygen

Diamorphine/morphine

Glyceryl trinitrate

11 What non-modifiable risk factors for coronary heart disease does Mr. Askew have?

Male sex

Age

12 List five modifiable risk factors for coronary heart disease.

Smoking

Hypertension

Hypercholesterolaemia

Diabetes mellitus

Obesity

13 Mr. Askew has a body mass index of 26.9kg/m2. What does this indicate?

Patient is overweight.

14 Name two drugs that may be used in obesity treatment to achieve weight reduction.

Orlistat.

Sibutramine.

15 Mr. Askew receives thrombolysis. List five contraindications to thrombolysis.

Coagulation defects, including anticoagulant treatment.

Stroke within last six months.

Possible aortic dissection.

Acute/recent gastrointestinal bleeding.

Surgery/major trauma within last two weeks.

16 One UK National Service Framework states patients suitable for thrombolysis

262

Page 263: finals2009.wikispaces.com · Web viewPleural effusion which is secondary to hypoalbunimaemia Q4. What initial investigations would be implemented to confirm this diagnosis? Urine

should receive this within 60 minutes of symptoms onset. Name the process that could

be used to evaluate whether current practice is meeting this standard.

Clinical audit.

A few hours later, Mr. Askew suddenly feels unwell. His ECG is shown below.

(Source: http://www.ecglibrary.com)

17 What does this ECG show?

Ventricular fibrillation.

18 Name two drugs that you might need to use as part of your immediate management

of this condition.

Adrenaline

Amiodarone

19 List five early complications of myocardial infarction.

Arrhythmias

Sudden death

Cardiogenic shock

Ruptured papillary muscle or chordae tendinae

Pericarditis

20 Mr. Askew has a total blood cholesterol of 8.1mmol/l. Which type of drug would you

consider prescribing?

Statin.

263

Page 264: finals2009.wikispaces.com · Web viewPleural effusion which is secondary to hypoalbunimaemia Q4. What initial investigations would be implemented to confirm this diagnosis? Urine

21 Give two side effects of this type of drug.

Myalgia

Myopathy

22 What lifestyle advice would you give Mr. Askew as part of secondary prevention?

Diet – decrease saturated fat intake, decrease salt intake, increase fibre intake (fruit

and vegetable intake).

Exercise – advise programme of cardiac rehabilitation leading to at least 30 minutes

of aerobic exercise three times per week.

Smoking – remain abstinent.

23 Another patient dies 3 days after having a myocardial infarction. What histological

features would you expect to find in the myocardium at post mortem?

Marked necrosis and inflammation is seen 2-4 days post-MI.

24 The relatives of the deceased patient are grieving. Name and outline one

psychological model which attempts to explain this process.

Worden’s four-stage model of grieving:

Stage 1- Accept the reality of the loss

Stage 2- Experience the pain of grief

Stage 3- Adjust to an environment without the deceased person

Stage 4- Withdraw emotional energy from the deceased person and reinvest this in

other relationships

Topic “ACUTE CORONARY SYNDROME”

7.3 Angina pectoris

By the end of Phase II students should be able to:

•recognise angina

•initiate investigations to confirm the diagnosis and assess the severity of underlying coronary

artery disease

•recognise the risk factors in individual patients

•initiate immediate management

7.4 Acute myocardial infarction

By the end of Phase II students should be able to:

264

Page 265: finals2009.wikispaces.com · Web viewPleural effusion which is secondary to hypoalbunimaemia Q4. What initial investigations would be implemented to confirm this diagnosis? Urine

•recognise acute myocardial infarction and use appropriate investigations to confirm the

diagnosis

•act appropriately to ensure that those patients likely to benefit receive thrombolysis as

quickly as possible

•control the pain of myocardial infarction

•recognise ventricular fibrillation and carry out immediate management

•recognise the need for further active management in the medium to long term

•describe the features of the underlying pathophysiological changes which can be used to

develop treatment and prevention strategies.

265

Page 266: finals2009.wikispaces.com · Web viewPleural effusion which is secondary to hypoalbunimaemia Q4. What initial investigations would be implemented to confirm this diagnosis? Urine

7.5

Rose, is an 83 year old lady, who presents to the same GP, requesting a ‘medical’. Rose

needs a medical certificate to satisfy the requirements for a local charity bungee-jump.

One of the first actions performed by the GP was to take Rose’s blood pressure.

Question> What are the diastolic and systolic values of a blood pressure reading,

above which, constitute hypertension?

Controversial numbers but essentially 140(systole) / 90 (diastole).

The GP notes that Rose has a large arm but the GP only has a small blood pressure

cuff, as the room had previously been used for a paediatric clinic.

Question> If the GP were to progress with the blood pressure recording using this

equipment, what effect would it have on the blood pressure reading, and why?

Blood pressure reading would be inaccurate and the reading (both systolic and

diastolic) would be higher than the actual blood pressure. The small cuff would not

occlude the brachial artery even at a high cuff pressure, thus giving a false high

reading.

In search of the correct equipment, the GP comes across an automated oscillometric

blood pressure device (e.g. ‘Dinamap’).

Question> State one limitation of this device?

Does not measure diastolic pressure (it calculates it using systolic pressure and

mean arterial pressure).

The machine may not be automated.

Need electrical/battery supply.

Requires user education.

Requires blood pressure recording to be done twice consecutively for a more

accurate measurement.

Is less accurate with arrhythmias.

The GP thinks Rose may have hypertension, so advises her to return to the surgery on

2 more occasions.

266

Page 267: finals2009.wikispaces.com · Web viewPleural effusion which is secondary to hypoalbunimaemia Q4. What initial investigations would be implemented to confirm this diagnosis? Urine

If Rose expresses anxiety associated with attending the doctor’s surgery, the GP may

want to investigate her blood pressure using a 24 hour blood pressure monitor.

Question> With a diagnosis of hypertension, the GP wants to assess end-organ

damage.

He looks into her eyes with a fundoscope. List the 4 stages of hypertensive retinopathy

outlining a characteristic feature of each.

Grade 1. Silver wiring; tortuous retinal arteries with increased reflectiveness.

Grade 2. Grade 1 plus arterio-venous (A-V) nipping; thickened retinal arteries

passing over retinal veins.

Grade 3. Grade 2 plus haemorrhages and soft exudates (i.e. cotton wool spots);

small infarcts.

Grade 4. Grade 3 plus papilloedema (blurring of optic disc margins).

Question> Apart from the eyes, list 3 other organs potentially at risk from uncontrolled

hypertension?

Brain; six times more likely to suffer with a Stroke.

Heart; 3 times more likely to have coronary artery disease.

Kidneys; renal failure.

Peripheral blood vessels; twice as likely to develop PVD. At an increase risk of aortic

dissection.

Question> Although in over 90% of cases, the cause of hypertension is unknown

(‘essential hypertension’), the GP would like to exclude any secondary causes of

Rose’s hypertension. Can you name 4 causes of secondary hypertension?

Renal causes: diabetic nephropathy, chronic glomerulonephritis, polycystic disease,

tubulointerstitial nephritis and renovascular disease (e.g. renal artery stenosis).

Endocrine causes: conn’s, adrenal hyperplasia, phaeochromocytoma, cushing’s and

acromegaly.

Congenital causes: coarctation of the aorta.

Medication: oral contraceptive, NSAID’s, steroids, vasopressin, carbenoxolone,

liquorice derivatives, and sympathomimetics.

Pregnancy.

267

Page 268: finals2009.wikispaces.com · Web viewPleural effusion which is secondary to hypoalbunimaemia Q4. What initial investigations would be implemented to confirm this diagnosis? Urine

Question> The GP initiates Rose’s hypertension management by giving her some

lifestyle advice. List 4 such interventions that will aid Rose’s hypertension and reduce

her risk of complications.

Weight reduction (BMI<25 ideally).

Low fat (especially saturated fat) diet.

Low sodium intake (<6g/day). Smoking cessation.

Limit alcohol consumption to advisable units (<14 (women) and <21 (men)/week).

Partake in exercise.

Eat fruit, vegetables and fish regularly.

Question> Despite this good advice, it is clear Rose may still need medication to

reduce her blood pressure. List 4 categories of first line medication that can be used to

achieve a reduction in blood pressure?

ACE Inhibitor

β antagonist

Calcium channel blocker

Diuretic.

Angiotensin II receptor antagonist.

α antagonists.

Extended Question> What is Malignant Hypertension and name 2 symptoms that may

occur in someone with malignant hypertension.

A recent and severe rise in blood pressure that, if left untreated, could be life

threatening (>220/120mmHg). It carries an 80%+ mortality rate at 1 year if left

untreated.

Symptoms include: severe headache, visual disturbances, seizures, transient loss of

consciousness, reduced urine output, blood in the urine, symptoms of heart failure

(i.e. frothy white sputum cough, dyspnoea, swollen ankles, orthopnoea, paroxysmal

nocturnal dyspnoea, anorexia, fatigue and nausea).

Rose wasn’t too bothered when the GP advised against performing the bungee jump, she had

done 10 jumps already.

268

Page 269: finals2009.wikispaces.com · Web viewPleural effusion which is secondary to hypoalbunimaemia Q4. What initial investigations would be implemented to confirm this diagnosis? Urine

7.5 Hypertension

By the end of Phase II students should be able to:

• appreciate the importance of recognising and treating hypertension to reduce the risk of

morbidity and mortality from its complications

• diagnose hypertension having regard to the variability in blood pressure and possible

errors in its measurement

• look for possible remediable causes

• recognise end organ damage

• manage a patient with hypertension having regard to modification of other factors, the

place of non-drug methods, the risks and benefits of pharmacological intervention and

problems of compliance

269